You are on page 1of 179

1119 PROGRESSION

Edexcel GCSE (91)

Mathematics
Higher
Practice, Reasoning and Problem-solving Book
Confidence Fluency Problem-solving Reasoning

ALW AY S L E A R N I NG i
1119 PROGRESSION

Edexcel GCSE (91)

Mathematics
Higher
Practice, Reasoning and Problem-solving Book
Confidence Fluency Problem-solving Reasoning

ALW AY S L E A R N I NG i
Published by Pearson Education Limited, 80 Strand, London A note from the publisher
WC2R 0RL. In order to ensure that this resource offers high-quality
support for the associated Pearson qualification, it has
www.pearsonschoolsandfecolleges.co.uk
been through a review process by the awarding body.
Copies of official specifications for all Edexcel qualifications This process confirms that; this resource fully covers the
may be found on the website: www.edexcel.com teaching and learning content of the specification or part
of a specification at which it is aimed. It also confirms
Text Pearson Education Limited 2015
that it demonstrates an appropriate balance between the
Edited by ProjectOne Publishing Solutions, Scotland
development of subject skills, knowledge and understand-
Typeset and illustrated by Tech-Set, Gateshead
ing, in addition to preparation for assessment. Endorse-
Original illustrations Pearson Education Limited 2015
ment does not cover any guidance on assessment activities
The rights of Bola Abiloye, Gemma Batty, Phil Boor, Cath- or processes (e.g. practice questions or advice on how to
erine Murphy and Claire Powis to be identified as authors of answer assessment questions), included in the resource nor
this work have been asserted by them in accordance with does it prescribe any particular approach to the teaching
the Copyright, Designs and Patents Act 1988. or delivery of a related course. While the publishers have
made every attempt to ensure that advice on the qualifica-
First published 2015
tion and its assessment is accurate, the official specification
18 17 16 15 and associated assessment guidance materials are the only
10 9 8 7 6 5 4 3 2 1 authoritative source of information and should always be
referred to for definitive guidance. Pearson examiners have
British Library Cataloguing in Publication Data not contributed to any sections in this resource relevant
A catalogue record for this book is available from the British to examination papers for which they have responsibility.
Library Examiners will not use endorsed resources as a source of
ISBN 9781292190242 material for any assessment set by Pearson. Endorsement
of a resource does not mean that the resource is required to
Copyright notice achieve this Pearson qualification, nor does it mean that it
All rights reserved. No part of this publication may be repro- is the only suitable material available to support the quali-
duced in any form or by any means (including photocopy- fication, and any resource lists produced by the awarding
ing or storing it in any medium by electronic means and body shall include this and other appropriate resources.
whether or not transiently or incidentally to some other use
of this publication) without the written permission of the
copyright owner, except in accordance with the provisions
of the Copyright, Designs and Patents Act 1988 or under the
terms of a licence issued by the Copyright Licensing Agency,
Saffron House, 610 Kirby Street, London EC1N 8TS (www.
cla.co.uk). Applications for the copyright owners written
permission should be addressed to the publisher.
Acknowledgements
We would like to thank Glyn Payne for his work on this book.
The publisher would like to thank the following for their
kind permission to reproduce their photographs:
Cover images: Front: Created by Fusako, Photography by
NanaAkua
Every effort has been made to contact copyright holders
of material reproduced in this book. Any omissions will be
rectified in subsequent printings if notice is given to the
publishers.
Contents

1 Number 1 6.2 More linear graphs 46


1.1 Number problems and reasoning 1 6.3 Graphing rates of change 47
1.2 Place value and estimating 1 6.4 Real-life graphs 49
1.3 HCF and LCM 2 6.5 Line segments 51
1.4 Calculating with powers (indices) 3 6.6 Quadratic graphs 52
1.5 Zero, negative and fractional indices 4 6.7 Cubic and reciprocal graphs 53
1.6 Powers of 10 and standard form 5 6.8 More graphs 54
1.7 Surds 6 Problem-solving 56
Problem-solving 7 7 Area and volume 58
2 Algebra 9 7.1 Perimeter and area 58
2.1 Algebraic indices 9 7.2 Units and accuracy 59
2.2 Expanding and factorising 9 7.3 Prisms 60
2.3 Equations 10 7.4 Circles 61
2.4 Formulae 11 7.5 Sectors of circles 63
2.5 Linear sequences 12 7.6 Cylinders and spheres 64
2.6 Non-linear sequences 13 7.7 Pyramids and cones 65
2.7 More expanding and factorising 15 Problem-solving 66
Problem-solving 15 8 Transformations and constructions 68
3 Interpreting and representing data 17 8.1 3D solids 68
3.1 Statistical diagrams 1 17 8.2 Reflection and rotation 68
3.2 Time series 19 8.3 Enlargement 70
3.3 Scatter graphs 20 8.4 Translations and combinations of
3.4 Line of best fit 21 transformations 72
3.5 Averages and range 23 8.5 Bearings and scale drawings 73
3.6 Statistical diagrams 2 25 8.6 Constructions 1 74
Problem-solving 26 8.7 Constructions 2 75
8.8 Loci 77
4 Fractions, ratio and percentages 29
Problem-solving 78
4.1 Fractions 29
4.2 Ratios 30 9 Equations and inequalities 80
4.3 Ratio and proportion 31 9.1 Solving quadratic equations 1 80
4.4 Percentages 32 9.2 Solving quadratic equations 2 81
4.5 Fractions, decimals and percentages 33 9.3 Completing the square 81
Problem-solving 34 9.4 Solving simple simultaneous equations 82
9.5 More simultaneous equations 83
5 Angles and trigonometry 35
9.6 Solving linear and quadratic
5.1 Angle properties of triangles and
simultaneous equations 84
quadrilaterals 35
9.7 Solving linear inequalities 85
5.2 Interior angles of a polygon 37
Problem-solving 86
5.3 Exterior angles of a polygon 38
5.4 Pythagoras theorem 1 39 10 Probability 88
5.5 Pythagoras theorem 2 40 10.1 Combined events 88
5.6 Trigonometry 1 41 10.2 Mutually exclusive events 89
5.7 Trigonometry 2 42 10.3 Experimental probability 90
Problem-solving 44 10.4 Independent events and tree diagrams 91
10.5 Conditional probability 92
6 Graphs 45
10.6 Venn diagrams and set notation 94
6.1 Linear graphs 45
Problem-solving 95

iii
11 Multiplicative reasoning 97 17 More algebra 148
11.1 Growth and decay 97 17.1 Rearranging formulae 148
11.2 Compound measures 98 17.2 Algebraic fractions 149
11.3 More compound measures 99 17.3 Simplifying algebraic fractions 149
11.4 Ratio and proportion 100 17.4 More algebraic fractions 150
Problem-solving 101 17.5 Surds 151
17.6 Solving algebraic fraction equations 152
12 Similarity and congruence 102
17.7 Functions 152
12.1 Congruence 102
17.8 Proof 153
12.2 Geometric proof and congruence 103
Problem-solving 154
12.3 Similarity 104
12.4 More similarity 106 18 Vectors and geometric proof 155
12.5 Similarity in 3D solids 107 18.1 Vectors and vector notation 155
Problem-solving 108 18.2 Vector arithmetic 155
18.3 More vector arithmetic 156
13 More trigonometry 110
18.4 Parallel vectors and collinear points 158
13.1 Accuracy 110
18.5 Solving geometric problems 159
13.2 Graph of the sine function 110
Problem-solving 160
13.3 Graph of the cosine function 112
13.4 The tangent function 113 19 Proportion and graphs 162
13.5 Calculating areas and the sine rule 115 19.1 Direct proportion 162
13.6 The cosine rule and 2D trigonometric 19.2 More direct proportion 162
problems 116 19.3 Inverse proportion 163
13.7 Solving problems in 3D 117 19.4 Exponential functions 165
13.8 Transforming trigonometric graphs 1 118 19.5 Non-linear graphs 166
13.9 Transforming trigonometric graphs 2 119 19.6 Translating graphs of functions 168
Problem-solving 121 19.7 Reflecting and stretching graphs of
functions 169
14 Further statistics 123
Problem-solving 171
14.1 Sampling 123
14.2 Cumulative frequency 124
14.3 Box plots 125
14.4 Drawing histograms 127
14.5 Interpreting histograms 127
14.6 Comparing and describing populations 129
Problem-solving 130
15 Equations and graphs 131
15.1 Solving simultaneous equations
graphically 131
15.2 Representing inequalities graphically 132
15.3 Graphs of quadratic functions 134
15.4 Solving quadratic equations
graphically 136
15.5 Graphs of cubic functions 137
Problem-solving 139
16 Circle theorems 141
16.1 Radii and chords 141
16.2 Tangents 142
16.3 Angles in circles 1 143
16.4 Angles in circles 2 144
16.5 Applying circle theorems 146
Problem-solving 147

iv
Welcome to Edexcel GCSE (9-1) Mathematics Higher
Practice, Reasoning and Problem-solving Book

This Practice Book is packed with extra practice on all the content of the Student Book
giving you more opportunities to practise answering simple questions as well as
problem-solving and reasoning ones.

Icons alongside the


questions show
their level of
difficulty. Questions
in this book will
range from 6 to
12 .

Exam-style
questions
are included
throughout to help
you prepare for
your GCSE exam.
The letters P
and R are used
to show where a
question requires
you to problem- There is a section
solve or reason relating to every
mathematically mastery lesson in
essential skills for the Student Book.
your GCSE.

QR codes link to worked


examples from the Student Book
which will help you with the
question they are alongside.

v
Problem-solving section

These are strategies that you have A QR code is given in the problem-solving section
learned so far in the Student Book. They where you learn a new strategy in that unit in the
build up as you work through the book. Student Book. Scan it to see the worked example
Consider whether they could help you to and remind yourself of the strategy.
answer some of the following questions.

The R symbol indicates questions


where you are required to reason
mathematically. Questions in this
section do not have the P symbol as
they are all problem-solving.

vi
1 NUMBER
a Write down the different orders in which
1.1 N
 umber problems they can finish first, second, third and
and reasoning fourth.
b How many different ways can the prizes be
1 A T-shirt manufacturer offers a choice of three awarded if there are
different coloured shirts red, green or blue, i 5 people and 5 prizes
and two different patterns spots or stripes. ii 10 people and 10 prizes
a Write down all possible combinations of iii 12 people and 12 prizes?
colours and patterns. 4 P A shop alarm needs a 5-character code.
b How did you order your list to make sure a How many different codes can be created
that you didnt miss any combinations? using
The manufacturer decides to offer yellow i 5 numbers
T-shirts as well. ii 5 letters
c How many possible combinations are iii 4 letters followed by 1 number?
there now? The shop owners decide not to repeat a digit
d Copy and complete. or letter.
3 colours and 2 patterns: b How many ways are possible in parts i to
combinations iii now?
4 colours and 2 patterns:
combinations
n colours and m patterns: 1.2 P
 lace value and
combinations estimating
Another T-shirt company offers 4 colours,
2patterns and 3 different sizes. 1 Work out
e How many possible combinations are a 56 12 b
28 24
there now? c 14 48 d
7 96
2 Exam-style question 2 5.4 7.39 = 39.906
A bank asks for a 3-character PIN code. Use this fact to work out the calculations
The first character is a letter. below.
The other two characters are digits between Check your answers using an approximate
0 and 9 inclusive. calculation.
a How many choices are possible for each a 54 7.39
character of the code? b 5.4 0.00739
b What is the total number of 3-character c 0.0054 0.739
PIN codes possible? d 39.906 7.39
Amy chooses a PIN code beginning with e 3990.6 73.9
the letter A.
f 399.06 739
c How many different PIN codes are
possible for Amy? (5 marks) 3 R 18.4 9.07 = 166.888
a Write down three more calculations that
Q2 hint Inclusive means that the end have the same answer.
numbers are also included. b Write down a division that has an answer
of 18.4.
3 Four people, Anne, Brian, Colin and Danni c Write down a division that has an answer
enter a competition. Between the four of of 0.907.
them, they win first, second, third and fourth d Prue says that 18.4 907 = 16888.08.
prizes. Explain why Prue must be wrong.

R = Reasoning P = Problem-solving 1.2 Place value and estimating 1


__ ___
4 a Write down the value of___ and
9 ___ ___
.
16 13 PTiles measure 15cm by 20cm.
b Estimate
___ the value of 10, 11, 12and
Each tile costs 1.09.
13. a Estimate the cost of tiles required for
Round each estimate to 1 decimal place. these rectangular spaces.
c Use a calculator to check your answers to i 90cm by 120cm
part b. ii 2m by 4m
iii 3.1m by 2.6m
Q4b hint Use a number line to help. b Use a calculator to work out each answer.
9 10 16 14 A spreadsheet is used to record the numbers
of books on 10 shelves in the library.
The numbers are in cells A1 to J1.
The mean number is in cell K1.
5 Estimate the value to___
the nearest tenth.___
__ A B C D E F G H I J K
a ___
6 b ___
30 c ___
29 1 13 18 22 11 16 19 23 24 32 15 23.2
d 40 e 59 f 70

a Use estimates to show that the mean
6 P A logo contains 300 square pixels. shown in cell K1 is wrong.
The logo has a total area of 600mm2. b Work out the correct mean to the nearest
a Estimate the side length of a pixel. tenth.
b Use a calculator to check your answer.
7 a Write down the value of 92 and 102.
b Estimate the value of 9.22 and 9.72. Round 1.3 HCF and LCM
each estimate to the nearest whole number.
c Use a calculator to check your answers to 1 a Copy and complete this factor tree for 54.
part b. 54
8 Estimate to the nearest whole number.
a 4.12 b 5.22 c 2.62
d 9.6 e
2 3.4 f
2 10.32 27

9 a Estimate answers _____


to these.
i 12.5 48.416.27

9
ii 2.91 (12.4 2.87)2
2
31.4______
iii _____________

18.5+1.986
___________
15.432.91
b Write 54 as a product of its prime factors.
iv _____________


(3.42.091)2 54 = =
b Use your calculator to work out each
answer. Q1b hint Circle the prime factors in your
Give your answers correct to 1 decimal factor tree.
place.
2 Write 24 as a product of its prime factors.
10 The sum of these two values is 9.
____
14+121 3 Gail and Hermione are asked to find 90 as a
_________ 82
___ ______ __ product of its prime factors.
25
9

Gail begins by writing 90 = 9 10
Work out the missing number. Hermione begins by writing 90 = 3 30
11 PA cubic block has a side length of 12.3cm. a Work out a final answer for Gail.
Estimate the surface area of the block. b Work out a final answer for Hermione.
12 PThe area of a square is 120cm2. c Start the prime decomposition of 50 in
Estimatethe perimeter of the square. two different ways: 2 25 and 5 10.

2 R = Reasoning P = Problem-solving 1.2 Place value and estimating


4 Write each number as a product of its prime 15 a Daryl says the prime factors of 4 appear in
factors in index form. the prime factor decomposition of 3000,
a 35 b 20 c 33 so 3000 is divisible by 4.
d 48 e 56 f 100 Is 3000 divisible by 12, 75 or 21?
b Use prime factors to show that 945 is
5 180 can be written as a product of its prime divisible by 27.
factors in the form 2m n2 p.
c Is 945 divisible by 25?
Work out m, n and p. Explain your answer.
6 Find the HCF and LCM of Example d Is 945 divisible by 35?
a 12 and 20 b 15 and 40 Explain your answer.
c 18 and 24 d
7 and 12 16 In prime factor form, 360 = 23 32 5 and
e 30 and 42 f
9 and 60 2700 = 22 33 52
a What is the HCF of 360 and 2700?
7 P Two lighthouses flash their lights at Give your answer in prime factor form.
different intervals. One flashes every b What is the LCM of 360 and 2700?
6seconds, the other every 9 seconds. Give your answer in prime factor form.
They have both just flashed together. c Which of these are factors of 180 and
Howlong before they next flash together? 2700?
i 2 2 3 ii 25
8 PMiranda needs 144 paper cups for a party.
iii 9 iv 22 3 5
The cups come in packs of 15, 20 or 24.
d Which of these are multiples of 180 and
Which single-size packs should she buy to
2700?
ensure she has exactly the right number?
i 23 34 52
9 PThe HCF of two numbers is 3. ii 2 32 52
Write down three possible pairs of numbers. iii 23 32 53
10 PThe LCM of two numbers is 15.
One of the numbers is 15. 1.4 C
 alculating with
a Write down all the possibilities for the powers (indices)
other number.
b Describe the set of numbers you have 1 Work__ out ___
created.
a 8 b 8
3 3

____ _____
c 125 d
216
3 3
11 54 = 2 33 and 36 = 22 32
Write down, as a product of its prime factors,
2 Work out these.
a the HCF of 54 and 36
Use _______
a calculator to check your answers.
___________
b the LCM of 54 and 36.
252+27
3
a 52+122
b
__ ___ ____ ___
c 83+25 d 27
3 3
12 Exam-style question __25+42
______________
___
84
3
_____
Given that A = 23 52 7 and e 27+
8+118 f ________
B = 2 53 72 ____ ___
(2)2
____
100 _____16 2752
3
write down, as a product of its prime factors, g _____
3 ___ h
3 ___ _________
3 _____

a the HCF of A and B 1 8 125
b the LCM of A and B. (2 marks)
Q2e hint The square root applies to the
whole calculation. Work out the calculation
13 Write 96 as a product of its prime factors. inside the square root first.
14 PThe prime factor decomposition of 3000 is
23 3 53 3 Work out
Write down the prime factor decomposition a [(18 22) 2]3
of b 54 [21 3 + (15 9___ 2)]

a 8 b 25 c 24 d 75 c [73 (3 42) + 5] 16

R = Reasoning P = Problem-solving 1.4 Calculating with powers (indices) 3


4 Work____out ____ b Keiko divides two powers of 7.
a __________ ________
4 4
256 b 625 7 7 = 75
c 100000000 0.00001
4 5
d What could the two powers be when
i both numbers are greater than 720
5 a Work out
ii one power is double the other power?
i 53 54 ii 57
iii 5 5
6 3 iv 59 13 Work out these.
b How can you work out the answers to Write each answer as a single power.
parta by using the indices of the powers a 35 38 32 b 94 92 97
you are multiplying? 237
3 7274
c ______
4 d
_______
3

c Check your rule works for 3 7
i 53 57 ii 56 5 14 The number of pixels on a computer screen is
iii 5 5
3 1
219. A company logo uses 212 pixels.
6 Write each product as a single power. a How many logos can be displayed on the
screen?
a 62 68 b 43 45 c 97 93
b Another company logo needs 215 pixels to
7 Find the value of n. display all its data.
a 3n 34 = 36 What fraction of the screen is blank?
b 94 9n = 92 15 Copy and complete.
c 72 7n = 27 a (53)6 = 53
8 Write these calculations as a single power. = 5
Give your answers in index form. b (22)4 = = 2
a 8 24 = 2 24 = 2 c (94)7 =
b 125 53 = 9
c 27 81
16 Write as a single power
d 2 32
a (52)4 b (93)4 c (41)5 d
(72)3
e 9 9 9
f 10 1000 100000 17 P Write each calculation as a single power.
3333333 48
___ 842
_____
9 R a i Work out ___________________
a 9 27 27 b c 3
333 16 2
by cancelling.
Write your answer as a power of 3.
ii Copy and complete. 37 33 = 3 1.5 Zero, negative and
b Copy and complete.
555555 fractional indices
56 52 = ________________

= 5
55
c Work out 89 83 1 Work out the value of n.
a 18 = 2 3n b 2n 2n = 210
10 Work out _
c 7 7 = 7 d
2n n 4 13 3n = 35
a 47 43 b
53 52 c
71 74
2 a Use a calculator to work out
11 Find the value of t.
i 31 ii 21 iii 1001 iv 41
a 75 72 = 7t
b Write your answers to part a as fractions.
b 59 5t = 53
c Use a calculator to work out
c 4t 44 = 48
i 32 ii 52 iii 1002 iv 62
12 Pa Ying multiplies three powers of 8 d Write your answers to part c as fractions.
together. e Work out
) ii ( __
i ( __ )
1
8 8 8 = 815 1 1 2
What could the three powers be when 4 3
i all three powers are different
ii all three powers are the same?

4 R = Reasoning P = Problem-solving 1.4 Calculating with powers (indices)


3 a Match the equivalent cards. 9 Work out _12

( 49)
_1
_
1
16
___
1 5 1 a 36 2 b

9 2 c

4 65 43
43 10 Work out
Example
a 1253
_2
2
1 1
b 1000005
_2
25 5 56 3
65 _

(___
)
16 2
c 92 d
_3

25
4 1 1 e 162 f
_3
10003
_2
29 5 92 56
11 Work out _1 1
_

( 25) ( 8)
b Write a matching card for the two cards _
1
_
2
16 2 __
___ 1 3
a 25 8 b


2

3
that are left over. _41 _5
c (
10000 )
( )
______ 1 1 2
____
c Copy and complete. 100
(__
) = __
1

(b)
3 a 1 12 Find the value of n.
so __
= __ _____
5 a 100 = 10n
n
b ___ 1000 =10


4 Work out these.
=(___ )
n
1 3 27 64
Write each answer as a single power. c __
= 2n d ___
8__ 64
___
27
7677 3532 3

a 45 44 42 b _______ _______
c
4
e 2 = 2
5 n
f ( 25 ) = 5n
3 4
7
3 3
13 P / R Dan says that 83 252= 10
_1 _1
5 P a Copy and complete. 53 53 = 5
b Write down 53 as a whole number. a Show that Dan is wrong.
c 53 53 = 125 = b What mistake did he make?
d Copy and complete using parts a and c.
14 P / R Match the expressions with indices to
53 53 = 5 =
their values.
e Repeat parts a to d for 25 25.
1 1
f Write down a rule for a0, where a is any 8 
3 1 
2
number. 1 000 000
27 16
6 Work out
1
a 51 b 32 c 105 25 2 3
4
d ( __
) e
(__
) f
2 1
(
2_12)
1 4 1
36 2
2 5
g ( ) h ( ) i
4 2
__ 1 3
___ (0.3)1 
3 
3
5 10 1 2 1 5
8
j (0.25)3 k (21)0 l (8 4)1 10 000 32
7 a Use a calculator to work out 1
_

iv ( __
)
1 2
i 42 ii 1002 iii 1442 2
_1 _1 _1

4 9 8 
3 5
b Copy and complete. 4 27 6
a2is the same as the _________
_1

____________ of a.
c Work out 31
_

iii ( ) iv 83
1
___
1.6 Powers of 10 and
_1 _
1 _1
i 83 ii 643



27
d Copy and complete.
a3is the same as the _________
_1 standard form
____________ of a.
e Copy and complete. 1 Write these prefixes in size order, starting
_1
i 81 = 3 so 81 4 = with the smallest.
_1
ii 1024 = 5 so 1024 5 = pico, milli, nano, tera, kilo, deci, micro, giga,
centi, mega
8 Evaluate 21
_ 21
_

( ) d ( )
_
1
_
1
4
__ 25
___ 2 Convert
a 1002 b

83 c


9 _1 49 a 3Tg into grams b
8m into metres
31
_ 2 31
_

( ) g ( ) h (
1000 )
_
1
1
___ 4
__ 1
_____
e 273 f

c 3.5Gg into kg d 9000ps into seconds.
27 9

R = Reasoning P = Problem-solving 1.6 Powers of 10 and standard form 5


3 Write these measurements in grams. 10 P A space shuttle travels at 2.8 104km per
a 1 grain of rice weighs about 2mg. hour. The Earth is 3.844 105km from the
b An elephant weighs about 5.5Mg. Moon. How long would it take to travel the
c A bacterium weighs about 1.2pg. distance between the Earth and the Moon?
Give your answer in hours and minutes.
4 a Copy and complete. 39000000 = 3.9
11 P The mass of a carbon atom is
b 10000000 = 10
1.994 1023g.
c 39000000 = 3.9 10
How many atoms of carbon are there in a
5 Which of these numbers are in standard form? sample weighing 20g?
A 5.4 103 B 91 102 Give your answer in standard form to 2 d.p.
C 0.7 104 D 9.2 1013 12 a Write these numbers as ordinary numbers.
E 5.2nm F 8.6 10 i 5 103 ii 7 105
6 Write these numbers in standard form. b Work out (5 10 ) + (7 105), giving your
3

a 540000 b 1903000 answer in standard form.


c 209000 d 0.000754 13 Work out these.
e 0.9015 f 0.0007 Give your answers in standard form.
a 4.8 103 + 2.1 102
Q6a hint Write the number between 1 and b 4.2 104 3.5 103
10 first. Multiply by a power of 10 to give c 9.5 102 + 6.8 101
540000.
d 1.2 103 9 102
7 Write these as ordinary numbers. 14 Exam-style question
a 5 102 b 7.1 105 (7 10x) + (8 10y) + (9 10z) = 9.00708
c 8.04 10 d
7 9.25 103 Write down a possible set of values
e 8.04 105 f 3.001 102 for x, y and z. (3 marks)
8 R a The average distance from the Sun to
Pluto is 5906000000000m. Exam hint
i Write this number in standard form. Dont just write down the possible values
ii Enter the ordinary number in your give your working to show how you
calculator and press the = key. worked out the values.
Compare your calculator number with
the standard form number.
Explain how your calculator displays a
number in standard form. 1.7 Surds
b The diameter of a hair follicle is 0.00069m.
i Write this number in standard form. 1 Write to 2 decimal___places.
__ ___ ___
ii Enter the ordinary number in your a 6
b 11 c
28 d
37

calculator and press the = key.
Compare your calculator number with 2 a Work__out __ ___
the standard form number. i 2 5 ii 10
9 Work out these. b Work__out __ ___
Use a calculator to check your answers. i 3 7 ii 21
a (2 104) (3 108) c What do you notice about your answers to
Example
b (5 102) (7 103) parts a and b?
c (9 103) (4 105) d Fill in the missing __
numbers.
__ __
i 2 __

3=
d (8 103) (2 102) __ ___
e (12 105) (3 102) ii __

2 = 12
__ ___
f (3 102) (3 104) iii 7= 14
g (2 105)2 h (3 102)4

6 R = Reasoning P = Problem-solving 1.6 Powers of 10 and standard form


3 Find the value of the integer k to simplify 13 Rationalise the denominators. Example
these surds. Simplify your answers
___ __ __ __ __
a 72= 2=
k2 if possible.
____ __ __ __ 1__ 1 1
b 125= __ 5=
___
k5 a ___
b ___
__ c ____
___

2
8
12
c 48= k 3

1 3 4
d ____ ____ ____
___ __
d 63= k7 ___ e ___ f ___

30
18
13
4 Simplify these surds.___ 12 5
___ ___ g ____
___ h ___
__
a ____
18
b ___
75
___

c 98
16
5
3__
d 200 e 45
20

f 14 R / P Henry types ___ into his calculator.

__ 3
Q4 hint Find a factor that is also a square Hisdisplay shows 3.
__
number. 3
a Show that ___ __ =3
____
3

5 Use a calculator to work out 128 b Use your calculator to check your answers
a as a simplified surd from Q12.
b as a decimal.
__
15 The area of a rectangle
__ is 33cm2. The length
. What could the
6 a A surd simplifies to 43 cm.
of one side is 3
original surd be? Work out the length of the other side. Give
b How did you find the surd? your answer as a surd in its simplest form.
7 Simplify
__ __ ___
16 Work out the area of these shapes. Give your


5 ___ 5 17 answer as a surd in its simplest form.
a = __= b
__ ___
2 5 3
4

4
25 a b c
___ ___ ___


8 2 2
10 ___ 10 6
c ___
=____ ___
= d 1
64 64 16 6
5
2 3
8 Copy and complete the table using the 12
numbers below.
Rational Irrational 1 Problem-solving
___
5 _____ Solve problems using this Example
15
0.3120.01
__
strategy where appropriate:
___ 9

__

2
___

0.7100
____
9
3 Use pictures or lists.
25
9 Solve the equation x2 + 20 = 70, giving your 1 R Kevin cuts a 90cm length
answer as a surd in its simplest form. of wood into thirds by
estimating the places to cut.
10 Solve these equations, giving your answer as One piece is exactly a third of the total length.
a surd in its simplest form. The difference in length between the other
a 3x2 = 54 b _
13x2 = 225 two pieces is 5cm. How long is each piece?
c x2 5 = 75 d 2x2 + 2 = 42
Q1 hint Draw a picture of the three lengths.
11 The area of a square is 75cm2.
Find the length of one side of the square. 2 R 126314 tickets have been sold to a
Give your answer as a surd in its simplest concert in the park. On average, each ticket
form. holder throws away 942g of rubbish. The
rubbish will be taken away in trucks that
12 a Work out__ __ __ ___ must not carry more than 575kg each.
i 3__5 2__
3 ii 2__
7 3___
10 a Estimate the number of trucks that will be
iii 28 54 iv 62 711 needed.
b Use a calculator to check your answers to b Is this an under-estimate or an over-
parts i to iv. estimate? How can you tell?

R = Reasoning P = Problem-solving 1 Problem-solving 7


3 R Here are some clues to fill in these boxes. 7 R Fill in the boxes to make all five
=6 expressions equivalent.
Both numbers have two digits.
They round to the same number when 3 3
rounded to 1 significant figure. 23 2 3
22

They round to different numbers when


rounded to the nearest 100.
a Write a pair of numbers that fit the clues.
25 3 34
b Write the pair of numbers that fit which
have the lowest sum.
4 R At a charity quiz night, each team can
have 3, 4 or 5 people. 21 people take part. 3 3 18
a How many different possible
arrangements of teams are there?
Each team pays 8. The total raised is 40. 8 R On average, a grain of very coarse
b How many people could there be in each sand weighs 11mg and has a volume of
team? 0.00419cm3.
c What is the maximum amount of money a 1 litre = 1000cm3. How many cubic
that could be raised from this number of centimetres are in a 12 litre bucket?
people? _______________
b How many grains of very coarse sand will


2(39.8)2+7(2.14)3 fill a 12 litre bucket? Give your answer in
5 Rachel estimates that
_______________ standard form to 3 significant figures.
1.982
is about 8. The bucket will break if its contents weigh
a Without using a calculator, check if she is more than 15kg.
correct. c Can the bucket be lifted when it is half full
b Now use a calculator to arrive at the without breaking it? How can you tell?
correct answer.
c Comment on both your estimate and 9 Exam-style question
Rachels. One sheet of paper is 9 103cm thick.
6 The large gear has 24 teeth, the medium Mark wants to put 500 sheets of paper into
gears both have 20 teeth and the smallest the paper tray of his printer.
gear has 16 teeth. The paper tray is 4cm deep.
Is the paper tray deep enough for 500 sheets
16 of paper?
teeth You must explain your answer. (3 marks)
June 2013, Q15, 1MA0/1H
20 teeth 20 teeth
10 R This square has been split into four
sections. Section D has an area of 16.

24 teeth 3 2

2 2 A D

a What is the smallest number of complete


turns the small gear will have to make for
all the labels to be upright again? B C
b Every second, the small gear makes the
number of turns worked out in part a.
How many turns will the large gear make
a What is the area of the square?
i every second ii every minute?
b What is the ratio of area A:area C?

8 R = Reasoning P = Problem-solving 1 Problem-solving


2 ALGEBRA
b x5 x6 = x = x
2.1 Algebraic indices xxxxx
x5 x6 = _________________
=__

xxxxxx

1 Simplify Therefore x = __

a y4 y2 b x3 x5 c b2 b c x2 x5 = x = x
d y3 y3 y3 e m
_1
2m 6 xx
x2 x5 = ______________ =__
xxxxx

2 Simplify
Therefore x = __

a 3b2 5b5 b 2a 3a2
c 5m3 10m4 d t6 2t2 10 Simplify
e 4s t 2st f
3 2 4 3a3b2 2ab3 4ab a a2 b m4 c
c1 d
r0
3 Simplify 11 Exam-style question
a10
a x3 x2 b a5 a3 c ___ a Simplify 12x3y2 3x2y4 (3 marks)
a4
y
3 p
7 b ab a3 = a
r6
d ___
y e __5 f ______ Work out b. (1 mark)
r pp3

4 Simplify
12 Simplify
12g4 9p3
a ____
b ____
a (y3)2 b (a4)3 c (m1)1 d (w4)0
2g
3 3p
13 Simplify
c 18x 6x d
4 2 16t10 8t5
a (a3b5)2 b (x3y3)3
5 Simplify
c (3mn2)1 d (____ 3)
2
5b6
a (x5)2 b
(y3)3 c
(t4)5 d
(r3)7
2a
6 Simplify 14 Simplify
(__
)
r3
3 ___ ___ ___ ______
a (3t2)3 b
(4m3)2 c a y10 b 4x4 c 9a6 d 16x8y2
2
7 The term in each brick in the wall is found 15 R Copy and complete
1 _ _ 1 _ _ 1 1

by multiplying together the two terms in the a a4a4a4a4= a =


__ __ __ 4 __
aaaa=
4 4 4
bricks underneath.
_1
Copy and complete the wall. Therefore a4=
1 _ _ 1 _ _ 1 _ 1 1
b a5a5a5a5a5 = a =
24x8y __ __ __ __ 5 __
aaaaa=
5 5 5 5

_ 1
2x3y Therefore a5=
2x2 16 Simplify
_1
2x a (2x2q3)2 b ( 16b8)2
_1 _1
c ( 27x3y9)3 d
(16x12y16)4
Q7 hint To find the term in the yellow brick
you must work out 24x8y 2x3y
2.2 E
 xpanding and
8 Simplify factorising
( 2xy5)
3 6 2
4x y
a (3x2y3)2 b (2x3y5)3 c (5x2y)4 d _____

x13
9 R Copy and complete 1 R a Write down
a x4 x4 = x = x an expression
x4 containing brackets
x4 x4 = ___
4= 7
x for the area of
Therefore x = the rectangle.

R = Reasoning P = Problem-solving 2.2 Expanding and factorising 9


b Copy and complete this diagram to show 10 Factorise completely
the areas of the two small rectangles. a 12(a + 3)2 + 4(a + 3)
3 x b 6(m 1)2 12(m 1)
c 2(y + 7)2 4(y + 7)
d (x + 2y)2 3(x + 2y)
Area Area 5 7 e 7(a 2) + 14a(a 2)
5
f 3(x y)2 9(x y)
Example
c What do you notice about your answers to 11 R Show algebraically that the
parts a and b? product of five consecutive
numbers must be a multiple
2 R State whether each relation is an equation of 120.
or an identity.
Rewrite the identities using ;. 12 Exam-style question
a x + 1 = 1 + x b xy = yx
a Expand 2a(10b 3a) (1 mark)
2x
c 4x + 1 = 5x d ___
=0.5x b Factorise completely
4
12x2y + 18xy 2 (1 mark)
3 R By drawing rectangles show that ______
c Simplify 16x4y2 (2 marks)
a 2x(x + 1) = 2x2 + 2x
b a(2b + c) = 2ab + ac
c 2x(5 + 3x) = 10x + 6x2
4 a Expand
2.3 Equations
i2a(b + 3) ii3b(10 + 3a)
1 a Copy and complete to begin to solve the
b Use your answers to part a to expand and
equation.
simplify
4x 3 = 2x + 7
2a(b + 3) + 3b(10 + 3a)
4x 3 = 2x + 7
5 Expand and simplify x 3 =
a 3(x + 1) + 4x b 5y + 3(y + 2) b Solve the equation.
c 9(2x + 1) + 5x d 2(x + 5) + 4(x + 7)
e 4b + 3(b + 5) + 2b f 4(5x + 2) + 5(2 + 5x) 2 Solve
a 4x + 2 = 5x b 9x 5 = 7x + 1
6 Expand and simplify c x 4 = 2x 9 d 10x + 3 = 12x 5
a 2(x 4) b 3(x 5) + 8x
3 a Expand
c x(2x 4) d 2x(x + 1) x(x + 3)
i4(x 2) ii2(3x 2)
e 5x + 2x(10 x) f
5x(2 + 5x) 2(5x + 1)
b Use your answers to part a to solve
7 Find the HCF of 4(x 2) = 2(3x 2)
a 2a and 4ab b 7ab and 8b 4 a Expand and simplify 7(x 2) 3(x 3)
c 6ab and 3a d 15a2b2 and 5a b Use your answer to part a to solve
8 Factorise completely 7(x 2) 3(x 3) = 11
a 4x + 6 b 2xy + 8y 5 Solve these equations.
c 7xy + 2x d 3ab + 6ac a 3(2x 1) + 5(x + 2) = 40
e xyz + 2xy f a2 + 3a3 b 3(x + 1) (2x 5) = 12
g 10ab2 + 5ac h 4x2y2 + 6xz2
6 Solve
i 12n k 8n k j
3 2 2 15tm2 12mq3
a 3(2x + 1) + 2(x 7) = 20
9 a What is the HCF of 2(a + 3b)3 and 12(a + 3b)? b 9(2x 4) = 3(4x + 1)
b Copy and complete c 2(3x 4) = 5(10 2x)
2(a + 3b)2 12(a + 3b) d 10x 3(x + 7) = 12
= (a + 3b)[(a + 3b) ] e 9(2x 1) 2(3x + 4) = 15
= (a + 3b)(a + 3b ) f 6(10 2x) = 5(2 3x)

10 R = Reasoning P = Problem-solving 2.2 Expanding and factorising


7 Simplify these expressions by cancelling. 13 R A scientist mixes the same mass of two
12x 7y 3a 10m fluids with different densities.
a ___
__
b
c __
d ___


6 21 12 2 The resulting mixture has a density of 9g/cm3.
8 a Copy and complete to begin to solve the He uses 40cm3 of the first fluid and 5cm3 of
equation. the second fluid.
3x5
____ a Assume the mass of each fluid is xkg.
=8
2 Write down an expression for the density
3x5 of each fluid.
____ = 8
2 b Write down an expression for the density
3x 5 = of the mixture.
b Solve the equation. c Form and solve an equation to find the
9 a Copy and complete to begin to solve the value of x.
equation.
mass
___5 Q13a hint Density = _____

=2 volume
x+1
5
___ ( ) = 2 ( )
x+1
5 = x + 2.4 Formulae
b Solve the equation.
10 a By multiplying both sides of the equation 1 Write whether each of these is an expression,
3x+1 ___
____ x1 an equation, an identity or a formula.
= by 4, and cancelling,
2 4 a y = 2x + 5 b 2x 3 = 4x + 1
show that 2(3x + 1) = x 1. c 2x + 2y = 2(x + y) d 3xyz
Then solve the equation. e A = pr2 f s = u + at
b By multiplying both sides of the g m h
3 3(x + 1) 4 = 3x 1
x x 5 d
equation _ _=_by 6, and cancelling, i V = pr h
2 j s = _
2 3 6 t
show that 3x 2x = 5. 2x3
____
k (ab ) = a b l
3 2 2 6 =7x
Then solve the equation. 5
11 Solve these equations. 2 Use the formula y = x2 x3 to work out the
x+2 ______ 3x2 a a value of y when
a _____
= b ____=5 a x = 3 b x = 2
3 2 3 6
y+3 y+1 1 3
c _____ +_____
=__
d ______
=5 3 Use the formula c = (a + 1)3 b to work out
2 3 6 2x1 the value of c when
x+1 _____ x1 ___ x
e _____
+ + =2 a a = 1 and b = 75 b a = 4 and b = 50
3 5 15
12 P Find the size of the smallest angle in the 4 R The cost of posting a package is a fixed
kite. cost of 5 plus 2 per kg over 10kg.
a Work out the total cost for a 14kg
2x 1 9 package.
b Write a formula for the total cost (C) for a
package of weight pkg.
3x 2 10 3x 2 10 5 P a Write a formula, in terms of a and b, for
the perimeter, P, of the rectangle.
b

x27 b Use the formula to work out the value of


i P when a = 4 and b = 7
ii a when P = 30 and h = 7

R = Reasoning P = Problem-solving 2.4 Formulae 11


6 A credit card company charges e% monthly 12 R The formula for converting between
interest on all debts. temperatures in Fahrenheit (F) and
After n months the debt (D) for an initial Celsius(C) is
loan of R is given by the formula 9C
F=___ +32
5
D=R(_____
100 )
n
100+e

It is recommended that paint is stored
between 59F and 80F.
Mr Bloomer borrows 500 at a monthly
Convert this range of temperatures to
interest rate of 12%.
degrees Celsius.
How much will he owe after 6 months?
Give your answer to the nearest penny.

7 P The final velocity of an object can be 2.5 Linear sequences


calculated using the formula
v2 = u2 + 2as 1 Work out the 1st, 2nd, 3rd, 10th and 100th
where v = final velocity, u = initial velocity, terms of the sequence with nth term
a = acceleration and s = distance. a un = 2n 5 b un = 24 3n c un = 5
A ball is released from the top of the Empire 2 For each arithmetic sequence, work out
State Building, at a height of 380m. the common difference and hence find the
It accelerates at 9.8m/s2. 3rdterm.
1
Work out the final velocity of the ball when it a 0.21, 0.23, b _, 1,
3
hits the ground if c 12, 1, d 0.101, 0.202, ...
a the ball is dropped (assume that the initial
velocity is 0m/s) 3 R Write down the common Example
b the ball is thrown vertically down with an difference for each sequence.
initial velocity of 15m/s. a nth term 4n 5
b un = 2n 3
8 Change the subject of each Example c nth term 10 0.5n
formula to the letter given
d un = 0.5 0.2n
in the brackets.
a y = mx + c [x] 4 Write down, in terms of n, expressions for the
b E = mc2 [m] nth term of these arithmetic sequences.
2b
__ a 2, 5, 8, 11, b 19, 25, 31, 37,
c a=c [a] d C = 2pr [r]
c 18, 15, 12, 9, d 12, 24, 36,
e A = _12bh [b] f v2 = u2 + 2as [s] e 9, 9.5, 10, 10.5,
8M 5 R Which of the values in the cloud are terms
9 The formula K = __ can be used to convert
5 in the sequence un = 3n + 2?
between miles (M) and kilometres (K).
a Convert 38 miles into kilometres. 12 2
35
b Make M the subject of the formula. 4 102
c Convert 108km into miles.
17 202
10 The formula for the surface area of a sphere
is A = 4pr2 6 Exam-style question
a Make r the subject of the formula.
The first three terms of an arithmetic
b Work out the radius of a circle with surface
sequenceare
area 100cm2. Give your answer to 1 d.p.
_2
3, 1_3, 2
1
11 Exam-style question
a Write an expression in terms of n for the
a Make t the subject of the formula nth term. (2 marks)
2s = 4(3rt + 2) (2 marks)
b Is 15 a term in the sequence?
b Find t when r = 2 and s = 52. (3 marks) Explain your answer. (2 marks)

12 R = Reasoning P = Problem-solving 2.4 Formulae


7 R The nth term of the sequence 13 R When 5 is input into this function machine,
2, 15, 28, 41, ... is 13n 11. the output is 23.
a Solve 13n 11 = 200 When 1 is input into the function machine,
b Use your answer to part a to find how the output is 7.
many terms in the sequence are smaller
than 200. 3p 1q

8 R a Find the first term in the arithmetic a Work out the difference between the two
sequence 24, 31, 38, 45, which is greater inputs.
than 300. b Work out the difference between the two
b Find the first term in the arithmetic outputs.
sequence 400, 330, 260, which is less c Use your answers to parts a and b to find
than zero. the value of p in the function machine.
9 A baby girl gains 0.5lb in weight per week. d Work out the value of q.
a If the baby weighs 8.2lb at age 2 weeks, 14 R Find the values of p and q in this function
how much does she weigh when she is age machine when the inputs 1 and 12 produce
i 3 weeks outputs of 10 and 42 respectively.
ii 4 weeks
iii 5 weeks? 3p 1q
b How long will it be before she weighs more
than 2 stone?

Q9b hint 1 stone = 14lb 2.6 Non-linear sequences


1 Find the next three terms in each of these
10 Jen opens a new email account.
Fibonacci-like sequences.
In the first week she receives 12 junk emails.
a 1, 7, , , ,
Each week after that she receives 5 more junk
b 1, 1, , , ,
emails than the previous week.
c 2 , 4, , , ,
How long before she receives over 200 junk
emails a week? 2 Write down the first four terms of each
sequence.
11 R The nth term of an arithmetic sequence is n
un=3n 2 a un = __ 2 b un = 1n c un = 0.1n
n
a Write down the values of the first four
3 Write down the first five terms of these
terms, u1, u2, u3, u4.
geometric sequences.
b Write down the value of the common __
a first term = 22;
difference, d. __
term-to-term__rule is multiply by 2
c By substituting n = 0, work out the value
b first term = 3;
of the zero term, u0. 1
term-to-term rule is multiply by ___
__
12 a Find the outputs when the terms in each 3

of these arithmetic sequences are used as 4 P A paperboy is offered a payment of 1p for
inputs to the function machine. the first day, 2p for the second, 4p for the
third and so on.
42 15 How long before he is paid over 100 for the
paper round?
i 1, 5, 9, 13, 17,
ii 5, 10, 15, 20, 25, 5 The surface area of bacteria in a Petri dish
b Compare the common difference for increases by 10% each day.
each input sequence with the common a If the area of bacteria is 2cm2 on day 1,
difference for the output sequence. what will the area be
How are these related to the operations i on day 2 ii on day 3?
used in the function machine? b How long before the area is over 10cm2?

R = Reasoning P = Problem-solving 2.6 Non-linear sequences 13


6 A job has a starting salary of 22000. 12 R The triangular numbers can be found by
The company offers two different pay options. adding increasing integers.
Option 1: An increase of 5% of the current Term 1 Triangular number 1
salary each year. 2 1+2=3
Option 2: An increase of 1500 per year. 3 3+3=6
Which option would you choose? Explain why. 4 6 + 4 = 10
7 R a Write down the first six terms of the 5 10 + 5 = 15
sequence un = n2. a Copy and complete the table for the
b Work out a formula for the nth term of triangular numbers.
each sequence. Term 1 2 3 4 5 6 7 8 9
i 3, 6, 11, 18, 27,
ii 1, 2, 7, 14, 23, 34, ... Triangular
1 3
iii 2, 8, 18, 32, 50, 72, ... number
b Work out a formula for the nth triangular
8 Copy and complete this diagram to work number.
outthe next term in the sequence
3, 12, 25, 42, 13 The sequence 4, 13, 26, 43, 64 has nth
sequence 3 12 25 42 term in the form un = an2 + bn + c
a Find the second differences and show that
1st differences 19 113 117 a = 2.
b Subtract the sequence 2n2 from the given
2nd differences 14 14 sequence.
4 13 26 43 64
9 Work out the next term of each sequence.
2 2 8 18 32 50
a 2, 3, 6, 11, 18, ...
b 5, 10, 20, 35, 55, ... 2 14
c 0, 2, 6, 12, 20, c Find the nth term of this linear sequence.
10 a Copy and complete to work out the first d Write the nth term of 4, 13, 26, 43, 64
and second differences for the sequence 2n2 + n
un = n2 1
14 Find the nth term of each sequence.
sequence 0 3 8 15 24
a 1, 6, 13, 22, 33,
1st differences 13 15 17
b 2, 5, 16, 31, 50,
c 7, 14, 27, 46, 71,
2nd differences 12 d 1.5, 1, 4.5, 9, 14.5, ...

b Copy and complete for the sequence Q14 hint Use the method in Q13.
vn = 2n2 + n 5
sequence 22 5 16 31 50 15 The nth term of a sequence is un = 2n.
Show that the product of the 5th, 6th and 7th
1st differences 17 111 terms is 218.

2nd differences 14 16 Exam-style question


a Write down the first four terms in the
11 R Find a formula for the 1
Example sequence with nth term un=__n(2 marks)
nth term of each of these 2
quadratic sequences. b State the term-to-term rule. (1 mark)
a 5, 14, 29, 50, 77, c Use algebra to show that the product
of any two terms in the sequence is
b 1, 7, 17, 31, 49, 71,
also a term in the sequence. (2 marks)
c 12, 27, 52, 87, 132,

14 R = Reasoning P = Problem-solving 2.6 Non-linear sequences


9 P / R The rectangle and triangle shown have
2.7 More expanding and the same area. Find x.
factorising x13

1 Expand and simplify x x22


Example
a (x + 3)(x + 4)
2x
b (x + 3)(x 6)
c (x 2)(x + 4) 10 Copy and complete these factorisations.
d (x 2)(x 6) a 9x2 100 = (3x)2 2
= (3x )(3x + )
2 P Find the missing terms in these quadratic
b 4y2 25 = ( y)2 2
expansions.
= ( y )( y + )
a (x + )(x + 5 ) = x2 + x + 15
b (x )(x 3) = x2 8x + 11 Factorise
3 Expand and simplify a 100m2 1 b x2 y2
16c2 36 c
a (x + 1)2 b (x 1)2 12 Exam-style question
c (x + 7)2 d (x 5)2
a Factorise b2 5b + 6 (2 marks)
4 a Copy and complete to evaluate 352 152 b Expand (5r 2s)2 (2 marks)
without a calculator.
(35 15)(35 + 15) = 20 =
b Without using a calculator work out
i 552 352 2 Problem-solving
ii 1.52 0.52
Example
5 Expand and simplify Solve problems using these
a (x + 1)(x 1) b (x + 3)(x 3) strategies where appropriate:

6 Factorise
Use pictures or lists
a x2 16 b
y2 121 c
t2 64 Use smaller numbers.
7 Factorise 1 R Tom finds the original plan of his house
and sees that a rectangular extension has
a x2 + 5x + 6 b x2 + 5x + 4 Example
been built on the side. The rectangular area
c x + 6x + 5 d
2 x2 x 6
of the ground floor on the original plan is
e x2 + 2x 8 f x2 2x 3 10m long 8m wide. Theextension is the
g x + x 20 h
2 x2 + 9x + 14 full width of the house but Tom does not
i x2 3x 18 j x2 + 2x 15 know the length.
k x2 + 7x 30 l x2 + 20x + 100 a What is the area of the whole ground floor
if the unknown length is x?
8 P Charlie and James are given a mystery Expand your answer.
number.
Tom wants to widen his house by 3m.
Charlie squares the number.
b What would be the new total area of the
a Write down an algebraic expression to ground floor?
represent Charlies number. Expand and simplify your answer.
James adds 2 to the number and then After his extension is built, Tom decides to lay
squares then result. flooring on the whole of the ground floor but
b Write down an algebraic expression to not in the kitchen.
represent Jamess number. He knows that the kitchen is 4m wide but he
Jamess number is 32 larger than Charlies. does not know the length y.
c What was the original number? c What is the total area of floor to be covered?

Q8a hint Let x represent the mystery number. Q1 hint Draw a picture.

R = Reasoning P = Problem-solving 2 Problem-solving 15


2 There are x cakes in a packet, x packets in a 7 R Oliver is designing a rug with a
box, x boxes in a case, y cases in a single load Fibonacci-like sequence. He starts with two
and six loads to fill a truck. squares of side length 2cm and then adds a
a Write the total number of cakes in a load square of side length 4cm along the side of
as an algebraic expression in its simplest the first two squares.
form. a What are the sizes of the next three
Three trucks deliver equal numbers of cases squares of the rug?
to x2 stores. b Oliver wants to make the rug with
b How many cakes are delivered to each 10squares.
store? Write the total as a simplified What is the size of the tenth square?
algebraic expression. c Oliver has arranged the 10 squares in
aspiral.
3 The surface area s of a cuboid box is What is the size of the whole rug?
6200cm2.
The length is 30cm and the height is 20cm. 8 R Joshua and Charlie both expand and
What is the width? simplify (x + 2)(2x 3).
Show your working. Joshua says that the answer is 2x2 + x 6
Charlie says that the answer is 2x2 + 4x + 6
4 Exam-style question
Who is correct?
Here are the first five terms of an arithmetic Give reasons for your answer.
sequence:
5, 8, 11, 14, 17 9 Look at these simultaneous equations:
a What is the next number in this 2x + 2y = 8 and x 2y = 1.
sequence? (1 mark) Solve to find the values for x and y.
b Find an expression, in terms of n, for 10 Suki measured the distances a group of
the nth term of this sequence. (1 mark) animals travelled in one hour to find out their
c Johann says that 52 is in this sequence. average speed. She measured the distances
Is Johann correct? in millimetres.
Explain your answer. (2 marks)
Animal Distance (mm)
snail 540
5 The term-to-term rule of a sequence is
multiply by 3. The second term is 24. worm 12
Write the first five terms of the sequence. slug 22
ladybird 980
6 Find the missing terms in these quadratic
expressions. spider 110
a (x + 5)(x + ) = x2 + x + 20 a Convert each of the distances into
b (x + 8)(x ) = x2 + x 24 kilometres.
1 millimetre = 0.000001 kilometres
c (x )(x 4) = x2 + x + 8
b Write each of your answers to part a in
standard form.

16 R = Reasoning P = Problem-solving 2 Problem-solving


3 INTERPRETING AND
REPRESENTING DATA
c What is the range?
3.1 Statistical diagrams 1 d What is the median?
e Calculate the mean age of the people in
1 The pie charts show the sizes of jeans sold in the restaurant.
a shop on a Friday and Saturday.
n+1
60 pairs of jeans were sold on Friday and Q2d hint The median is the _____
th value,
2
120pairs on Saturday. where n is the total number of values.
Friday
Key
3 P The ages of participants Example
Size 10 in two different exercise
classes were recorded.
Size 12
The back-to-back stem and
Size 14 leaf diagram shows the results.
Compare the distribution of
Size 16
ages in the two classes.
Yoga Body pump
Saturday
Key 1 8 9
Size 10 8 8 7 2 3 4 4 6 6 9
Size 12 8 3 2 2 0 3 2 4 4 6
Size 14 8 7 5 0 0 4 1 9

Size 16 5 1 5 0

Key Yoga Body pump


7|2 2|3
a How many pairs of size 10 jeans were sold represents represents
on Saturday? 27 years 23 years
b Which pie chart has the larger sector for
size 14 jeans? 4 P The masses (in g, to the nearest gram) of
c Show that the same number of pairs of chicks were measured at two different farms.
size 14 jeans were sold on both days. Farm A: 23, 24, 24, 24, 25, 32, 33, 33, 34, 34,
35, 36, 37, 39, 40, 41, 41, 42, 42, 42
2 R The stem and leaf diagram shows the ages Farm B: 29, 36, 36, 37, 38, 40, 44, 44, 45, 46,
of customers in a restaurant. 46, 46, 49, 51, 52, 54, 54, 54, 55, 56
a Draw a back-to-back stem and leaf
1 2 5 7 Key diagram for this data.
2 | 4 means 24 years
2 4 9 b Use the shape of your diagram to compare
3 8 8 9 9 the distribution of the masses of chicks at
the two different farms.
4 0 2

a How many people are in the restaurant?


b What is the age of the oldest person in the
restaurant?

R = Reasoning P = Problem-solving 3.1 Statistical diagrams 1 17


5 Exam-style question 7 The frequency table shows the lengths of
time, t (in minutes), some people spend
The numbers of digital songs downloaded
watching TV each day.
by 20 customers in a month were recorded.
Time, t (min) Frequency
0 9
0 , t < 30 35
1 2 3 3 5
30 , t < 60 51
2 0 6 8 8 8 9 60 , t < 90 78
3 1 2 3 3 4 90 , t < 120 28
4 0 5 5 120 , t < 150 8

5 3 a How many people were surveyed?


b What percentage watched TV for more
Key than 1 hour?
1 | 2 represents 12 downloads c Estimate the range.
a Write down the number of customers d Copy and complete the frequency polygon.
who downloaded 45 songs. TV watching times
b Work out the range.
c What is the modal number of downloads? 80
d Work out the median. (4 marks)
60
Frequency

6 The table shows the prices, P, of 190 houses.


40
Price, P (000s) Frequency
0 , P < 100 0
20
100 , P < 200 12
200 , P < 300 19 0
300 , P < 400 27 0 30 60 90 120 150
Time (min)
400 , P < 500 32
500 , P < 600 89 8 R The heights of 30 seedlings at two
600 , P < 700 11 different nurseries were measured.
The results are displayed on the frequency
a Copy and complete the frequency
polygons.
diagram.
House prices B
Frequency

A
100

80
Frequency

60 Height (cm)

40 a Which data set has the greater range?


b Would you expect the median of data
20 setA to be greater than, less than or about
the same as the median of data set B?
0 c Which data set do you think shows
0 100 200 300 400 500 600 700 seedlings that were planted earlier?
Price (000s)
b Draw a frequency polygon on the same
diagram.

18 R = Reasoning P = Problem-solving 3.1 Statistical diagrams 1


c She also thinks that the number of viewers
3.2 Time series of programme A is rising at an increasing
rate. Is she correct?
1 R A leisure centre recorded the number of d Predict the viewing figures for both
swimmers in the pool at 2-hour intervals programmes in 2016.
during a 12-hour period.
4 P The tables show the Example
Time 0900 1100 1300 1500 1700 1900 2100 numbers of hits (in thousands)
Number of on a tourist information
35 9 12 2 35 41 28
swimmers website during each quarter
a How many swimmers were in the pool at over the last 3 years.
1300? 2012
b What is the smallest number of swimmers Q1 Q2 Q3 Q4
in the pool at any one time? 12 54 35 48
c Work out the average number of
swimmers in the pool. Give your answer to 2013
the nearest whole number. Q1 Q2 Q3 Q4
d Represent this time series on a line graph. 9 57 40 46
Comment on the variation in the number
2014
of swimmers.
Q1 Q2 Q3 Q4
2 Mr Jayshuk catches the train to work.
12 61 40 49
Herecords the number of minutes the train
is late over a 2-week period. a How many hits were there in the first
Day 1 2 3 4 5 quarter of 2014?
b In which quarter were there the most hits?
Number of minutes late 2 5 3 8 2
c Draw a time series graph for this data.
Day 6 7 8 9 10 d Describe the variation in numbers of hits
Number of minutes late 4 5 7 8 6 during the 3-year period and comment on
the overall trend.
Draw a time series graph for this data.
Comment on how the lateness of the train Q4a hint The numbers of hits are in
varies over the 2-week period. thousands.
3 R The time series graph shows the viewing
figures for two TV programmes, A and B. 5 P The tables show the population (in
Viewing figures millions) of the UK over a 10-year period.
Viewing figures (000s)

Year 2003 2004 2005 2006 2007


300
250 Programme B Population
59.4 59.7 60.1 60.4 ?
(in millions)
200
150 Year 2008 2009 2010 2011 2012
100 Programme A Population
61.2 61.6 62 63.3 63.7
50 (in millions)
0 The data for 2007 has been lost.
20 8
20 0
20 2
20 4
20 6
20 8
20 0
20 2
20 4
16

a Draw a line graph for this time series.


9
0
0
0
0
0
1
1
1
19

Year b Describe the overall trend.


a What were the viewing figures for c Estimate what the population might have
Programme A in 2004? been in 2007.
b The director of Programme A says that d Use your graph to predict the population
the viewing figures for Programme A in 2013.
have risen more than the figures for
programme B have fallen. Is she correct?

R = Reasoning P = Problem-solving 3.2 Time series 19


6 Exam-style question
70
In a grandfather clock a pendulum swings

Weight (kg)
from side to side.
60
The pendulum starts in a vertical position
and begins to swing.
50

140 150 160 170 180


Height (cm)
b Use the scatter graph to copy and
complete the sentence.
In general, patients who are taller weigh
. and patients who are
shorter weigh .
2 The height and age of a number of trees are
recorded, together with the average annual
bob rainfall in the area each tree is growing.
These scatter diagrams are plotted from
The tables show the horizontal distance of thedata.
the bob of the pendulum from the vertical a
during 1 second.
Time (s) 0 0.1 0.2 0.3 0.4 0.5
Distance (cm) 0 1 1.8 2.4 2.8 3 Height
Time (s) 0.6 0.7 0.8 0.9 1.0
Distance (cm) 2.8 2.4 1.8 1 0
a Draw a time series graph to show the data. Age
b At what time will the bob be 3cm from b
the vertical again?
c Predict the distance of the bob from the
vertical 2.5 seconds after it starts to swing.
Height
d For how long is the bob more than
1.8cm from the vertical during
1 second? (7 marks)
Annual rainfall
c
3.3 Scatter graphs
Average
1 A hospital recorded the heights and weights rainfall
of 8 patients.
The measurements are shown in the table.
Patient A B C D E F G H
Age
Height (cm) 160 145 171 165 167 162 155 149
For each graph state whether there is
Weight (kg) 60 53 66 62 65 62 59 57
positive, negative or no correlation and
a Copy and complete the scatter graph. describe in words what this means.
Patient A was 160cm tall and weighed
60kg so draw a cross at (160, 60). Q2a hint To describe what the correlation
For patient B, draw a cross at (145, 53). means in words, you could say, As age
Complete the scatter graph with crosses increases, height ....
for all the patients.

20 R = Reasoning P = Problem-solving 3.2 Time series


3 A farmer records the ages of some chickens b State the type of correlation between
in years and the number of eggs they each weight and length of spring extension for
produce in a week. weights between 1N and 6N.
Age (years) 4 6 1 2 8 5 7 3 c Describe in words what happens to the
spring when the weight exceeds 6N.
Number of
6 4 6 6 1 4 3 5
eggs 8 Exam-style question
a Draw a scatter graph for this data. Some students sit two English tests.
b Describe any relationship between these Their results are shown on the scatter graph.
two variables.
Results of English tests
4 A newsagent records how many of a
100%
particular type of chocolate bar are sold per
day as she gradually reduces the price.
The table shows her results. 80%

Price (p) 45 44 43 42 41 40 39 38
60%

Test B
Number sold 12 13 15 14 8 12 4 12
Draw a scatter graph and describe any 40%
relationship between the price of the
chocolate bars and the number of bars sold. 20%
Suggest a possible reason for this
relationship.
0
0 20% 40% 60% 80% 100%
5 R The manager of a coffee shop records the
daily temperature and the number of hot Test A
chocolates sold in one week. a What type of correlation does the
Day Mon Tues Wed Thurs Fri Sat scattergraph show?
Interpret your answer. (1 mark)
Temperature (C) 9 8 5 4 7 6
b What was the highest mark in
Number of hot Test B? (1 mark)
5 6 5 7 9 15
chocolates sold
c What was the range of marks for
a Draw a scatter graph and comment on any Test A? (2 marks)
relationship between the two variables.
b The manager says that the colder it is the
more hot chocolates are sold.
Does the scatter graph provide statistical 3.4 Line of best fit
evidence to support the managers view?
1 Which line, A, B or C, is the best line of best
6 What sort of correlation would you expect to
fit for the data points on the scatter graph?
find between
a a students mark in a maths test and in a B
science test A
b the speed of a train and the time taken to
travel 100 miles
c the number of magazines a student buys
and their weight? C

7 R In a physics experiment the weight


attached to a spring (in newtons, N) and the
length of the springs extension are recorded.
Weight (N) 1 2 3 4 5 6 7 8
Length of
1.2 2.4 3.6 4.8 6.0 7.2 7.4 7.5
spring (cm)
a Plot these points on a scatter graph.

R = Reasoning P = Problem-solving 3.4 Line of best fit 21


2 The table shows the percentage marks of Sample A
8students in two maths tests.
8
Test A
73 45 84 91 63 53 67 76 7
(%)

Growth (cm)
6
Test B
64 39 74 80 55 50 60 66 5
(%)
4
a Draw a scatter graph for Example 3
this data. 2
b Draw a line of best fit on 1
your graph. 0
c Use your line of best fit to 0 1 2 3 4 5 6 7 8 9 10
estimate the score in test B Rainfall (mm)
of a student who scores 80% in test A. Sample B
d Estimate the score in test A of a student
who scores 50% in test B. 6

Growth (cm)
5
3 A lorry driver records the distance travelled
4
and the number of litres of fuel used on each
3
of 6 journeys.
2
Distance 1
105 124 78 125 102 91
(miles)
0
Fuel 0 1 2 3 4 5 6 7 8 9 10
158 195 110 185 160 140 Rainfall (mm)
(litres)
a Draw a line of best fit on a scatter graph a Use the given lines of best fit to work out
and use the line to estimate the number two estimates for the growth of the crops
of litres of fuel used for a 110-mile journey. if the rainfall is 4.5mm.
The fuel tank on the lorry holds 1500 litres. b Which of the estimates is likely to be more
b Estimate how far the lorry can go between reliable? Give two reasons for your answer.
refuellings. 6 R The heights (to the nearest metre) and
trunk circumferences of 6 trees are recorded.
4 R The table shows the outside temperature
The table shows the height, h (in metres) and
and the daily heating costs of a house.
circumference, C (in cm) of the trees.
Temperature
10 6 7 3 0 4 h (m) 2 3 4 5 6 7
(C)
Heating cost C (cm) 30 82 93 101 112 120
12 16 14 20 24 18
() a Draw a scatter graph for this data.
a Draw a line of best fit on a scatter graph b Why is the first point classified as an
and use it to estimate outlier? Suggest a possible reason for this.
i the daily heating cost if the outside c Draw a line of best fit passing close to the
temperature is 8C remaining five points.
ii the outside temperature if the heating d Use the line to estimate the circumference
cost is 15 of a tree which is 3.5m in height.
iii the daily heating cost if the outside e Estimate the height of a tree with
temperature is 15C. circumference 125cm.
b Which of these estimates do you think is
the least reliable? 7 The table shows the age, x (in years) and
Give a reason for your answer. shoe size, y, of a sample of 9 boys.
Age, x 7 8 9 10 11 12 13 14 15
5 R Two surveys are carried out on crop
Shoe size, y 1 3 5 6 7 8 8.5 8.5 9
growth.
The results are shown on the scatter graphs. a Draw a scatter graph of this data.

22 R = Reasoning P = Problem-solving 3.4 Line of best fit


b Assuming that shoe size can be modelled 2 R The sizes of jeans sold in a shop one
using a line of best fit dayare
i estimate the shoe size of a 16-year-old 8, 8, 10, 10, 12, 12, 12, 12, 12, 12, 14, 14, 14,
ii estimate the shoe size of an 11-year-old. 16, 16, 18, 20, 20, 22, 22
c Which of the answers in part b is likely to a Work out the mean, median and mode of
be the more reliable? the sizes.
d By drawing a smooth curve close to the b The shop owner wants to order some more
data points, make new estimates of the jeans but can only order one size.
shoe sizes in part b. Which size should he order?
e Which of the two models is the more Give reasons for your answer.
accurate? Give a reason for your answer.
3 The numbers of passengers using a train
8 Exam-style question service one week are recorded in the table.
The tables show the distance, d (in km), Day Number of passengers
cars with different engine sizes, E, can travel Monday 230
on one litre of petrol.
Tuesday 180
Engine size,
1 1.4 1.6 2 Wednesday 170
E (litres)
Thursday 180
Distance,
16 14.4 13.5 12.2 Friday 210
d (km)
Engine size, a Work out the mean, median and mode of
2 3 3.5 4 the number of passengers.
E (litres)
Distance, b The train company wishes to work out the
11.8 9.4 8.4 7.1 average daily profit.
d (km)
a Plot the points on a scatter Which average should be used to calculate
graph. (2 marks) an accurate figure?
Give reasons for your answer.
b Describe the relationship between engine
size and the distance a car can travel on 4 R State whether it is best to use the mean,
one litre of petrol. (1 mark) median or mode for these data sets.
Another car has an engine size of 2.5 litres. Give reasons for your answers.
c Estimate the distance this car can travel a Colour of tablet case:
on one litre of petrol. (2 marks) red, blue, green, orange, blue
b Number of customers in a shop:
Exam hint 12, 12, 13, 17, 19
Always draw lines on your diagram for any 5 Identify the outliers of the data sets and find
readings from your graph. If you get the
the range of each.
answer wrong, you may still get marks for
using the correct method. a The heights of players in a netball team:
155cm, 145cm, 160cm, 21cm, 148cm,
150cm, 163cm
3.5 Averages and range b The monthly fuel bills of a home:
130, 125, 143, 192, 135, 33
1 The annual numbers of burglaries reported
ina town over the past 5 years are Q5 hint Think about whether you should
45, 33, 47, 47, 93 include the outlier in your calculations.
a Work out the mean, median and mode of Q5a hint Is it possible for someone in the
the number of burglaries. team to be 21cm tall?
b An insurance company bases how much
it charges on the average number of
burglaries.
Which of the averages would be the most
appropriate? Give reasons for your answer.

R = Reasoning P = Problem-solving 3.5 Averages and range 23


6 Identify the outliers of the data sets. 9 P The table shows the times taken, t (in
Calculate a sensible value of the range. seconds), for two different groups of athletes
Give a reason why the outlier has been to run a 100m race.
included or excluded in your calculation. Time, t (s) Group A Group B
a The average daily temperatures (C)
9.5 < t , 10 3 6
during a week in June in the UK:
19, 21, 14, 2, 18, 17, 16 10 < t , 10.5 5 9
b The heights of 10 sunflowers: 10.5 < t , 11 10 3
1.2m, 1.8m, 0.9m, 1.12m, 1.0m, 0.2m, 11 < t , 11.5 2 2
1.3m, 1.45m, 1.66m, 1.44m a How many athletes were in each group?
7 The grouped frequency table shows the b Explain why the median time for each
distance, D, students in a class travel to group will be between the 10th and 11th
school. values.
c In which class interval is Group As median
Distance, Frequency, Midpoint,
xf time?
D (miles) f x
d Work out which class interval contains
0<D,2 12 1 1 12 = 12 Group Bs median time.
2<D,4 11 e On average, which group has the shorter
4<D,6 7 time?
6<D,8 3 f State the modal class for each group.
8 < D , 10 2 g At an athletics competition, both groups
must compete against the current
Total
champions, Group C, who have a mean
a Copy and complete the Example time of 10.4s. Which group stands the
table to estimate the mean better chance ofbeating them?
distance the students travel Explain your answer.
to school.
10 Exam-style question
b Another student joins the
class. Bob asked each of 40 friends how many
If they travel 9 miles to school, will the minutes they took to get to work.
mean increase, decrease or stay the same? The results are shown in the frequency
polygon.
8 R The times, T (in minutes), a group of
Journey time
students spent watching TV one night are
shown in the table. 12
Time, T (min) Frequency, f 10
Frequency

0 < T , 20 2 8
20 < T , 40 19 6
40 < T , 60 14 4
2
60 < T , 80 2
0
a How many students spent less than 0 10 20 30 40 50
20minutes watching TV? Time (min)
b How many students altogether spent less
a How many people travelled for more
than 40 minutes watching TV?
than 20 minutes? (1 mark)
c How many students altogether spent less
b If the average time spent travelling
than 60 minutes watching TV?
to workis more than 30 minutes, the
d State the modal class. company will consider allowing its
e Explain why the median is the 19th value. employees to work from home.
f Use your answers to parts a to d to work out Should the company consider allowing
which class interval contains the median. employees to work from home?(4 marks)

24 R = Reasoning P = Problem-solving 3.5 Averages and range


4 The time series graph shows the increase in
3.6 Statistical diagrams 2 asupermarkets profits.
Increase in profits
1 A fixed menu at a caf offers two main course
options followed by two pudding choices. 6
One lunchtime there were 50 customers.
The table shows some information about

Profit ( millions)
their choices.
Chicken Vegetarian Total
Cheese 15 23 5
Ice cream 5
Total 50
Copy and complete the table. 4
2 R A school offers three language options 0
0 2010 2011 2012 2013 2014
and two humanities options at GCSE. Year
Students must choose one language and one
humanities option. The supermarket chain says that this shows
that the profit has increased by over 50% in
French German Mandarin Total the 4 years between 2010 and 2014.
History 57 18 126 a Explain why this graph is misleading.
Geography 12 b Draw a correct version.
Total 35 200 Comment on what this shows about the
actual increase in percentage profit.
a Copy and complete the table.
b What percentage of the students study Q4a hint Look closely at the vertical scale.
Mandarin?
c Which humanities option was more 5 A soft drinks supplier produces a bar chart
popular? showing sales of their soft drinks and uses
Give reasons for your answer. it to support the claim that sales of cola are
more than twice those of the other drinks.
3 R A survey was carried out in a town to find
Give two reasons why the diagram is
out if more parking spaces were needed.
misleading.
Of 60 men surveyed, 23 thought there were
not enough spaces. Sales of drinks
Of 60 women surveyed, 30% thought there
were enough spaces. 500
The town has equal numbers of men and
women.
Cans sold (millions)

a Display this information in a two-way


table.
b The town council will provide more parking 400
spaces if more than 60% of residents want Cola
more parking.
Will this happen?
Lemonade

Still drinks

300

0
Type of soft drink

R = Reasoning P = Problem-solving 3.6 Statistical diagrams 2 25


6 R A hospital patients temperature is taken 8 The times taken (in minutes) for employees
every hour. The time series graph is plotted of two companies to travel to work are
from the measurements. recorded.
Patients temperature Company A: 35, 42, 39, 30, 23, 43, 47, 39, 38, 42
Company B: 18, 22, 19, 12, 14, 17, 32, 39, 42, 25
42 a Explain why you would display this data
in a back-to-back stem and leaf diagram
41 instead of in a dual bar chart.
b Draw a back-to-back stem and leaf
Temperature (C)

40 diagram for this data.


c Find the median times to compare the
39
travel times for the employees of the two
companies.
38 9 Exam-style question
A company wants to compare the salaries
37 of its employees.
The table shows their current salaries.
0 Salary, S, () Frequency
01 0
02 0
03 0
04 0
05 0
06 0
07 0
08 0
09 0
0

10000 < S , 12000 3


:0
:0
:0
:0
:0
:0
:0
:0
:0
:0
00

Time 12000 < S , 14000 15

a What was the patients temperature at 14000 < S , 16000 19


03:00? 16000 < S , 18000 22
b A temperature of over 39.4C is considered 18000 < S , 20000 11
dangerous. At what time did the patients a Choose one of the following statistical
temperature drop below this? diagrams to display the data.
c Between which times was the drop in stem and leaf scatter graph
temperature the greatest? frequency polygon time series graph
7 R A librarian records the items borrowed by b Give reasons for your choice. (4 marks)
the over-50s and the under-50s, using four
categories: modern fiction (MF), classic fiction
(CF), non-fiction (NF) and other (O). Q9 hint Imagine trying to draw each
diagram. You should also think about what
During one hour the items borrowed are as
the diagram is going to be used for.
follows:
Over-50s: CF, MF, CF, NF, O, NF, NF, O, NF, NF,
CF, MF, O, MF, CF, NF, O, NF, NF, O
Under-50s: MF, MF, NF, NF, O, O, O, MF, MF, MF, 3 Problem-solving
O, O, MF, MF, MF, CF, O, O, MF, O
a Explain why it is not possible to display Solve problems using these strategies where
this data as a frequency polygon. appropriate:
b The librarian decides to display the data Use pictures or lists
in either a pie chart or a bar chart. Which
should she use if she is most interested in
Use smaller numbers.
i the proportions of items borrowed by 1 Sequences A and B are Fibonacci-like
over-50s and under-50s combined sequences. Work out the difference between
ii comparing the numbers of the different the 6thterm in sequence A and the 7th term
items borrowed by each age group? in sequence B.
c The librarian is going to buy 400 more Sequence A: 4, 7, 11, 18,
books for the library. How many modern Sequence B: 5, 8, 13, 21,
fiction titles should she buy?

26 R = Reasoning P = Problem-solving 3.6 Statistical diagrams 2


2 R Two teams are competing in a trampoline 4 Loughborough University and Birmingham
competition. Their scores are displayed in University are about 43 miles apart.
the back-to-back stem and leaf diagram. Gayleand Luke both travelled from one
Thecoach needs to decide which team to university to the other for interviews.
take to the championships. Gayle went one way and got there in
Team A Team B 57minutes. Luke went the other way and
travelled at an average speed of 50mph.
9 6 5 1 8 D
a Use the formula S=__to find how many
9 7 6 4 4 2 5 7 9 T
minutes it took Luke to travel between
6 3 1 3 1 4 5 5 7
the universities. Round your answer to the
1 4 3 9 9 nearest whole number.
b Who travelled faster?
Key Team A Team B
5|1 1|8 Marcie travelled between the universities
represents represents ona different day.
15 marks 18 marks She travelled at a speed of 72km/h.
a What are the ranges for the two teams? c Did Marcie take more or less time than Gayle?
b What is the median score for each team? 5 R A museum increased its ticket prices every
c Use the range, the median and the shape of year for 8 years.
the diagram to compare the distribution of The museum manager wants to see if this
scores of both teams. has affected the number of tickets sold.
Which team should the coach take to the The table shows the average number of tickets
championships? sold per week at the different ticket prices.
3 R Tina is working on a weather project. Ticket
1.00 1.50 2.00 2.50 2.75 3.00 3.75 4.25
Shehas found the average high temperature price ()
and the average low temperature for four Number
different cities around the world at three of tickets 250 246 262 235 228 236 182 135
different times of the year. sold
The table shows her data. In it, the average a Draw a scatter graph to show the data in
high and average low temperatures are the table.
displayed together like this: 9 / 5. b Is there a relationship between the two
Temperature (C) variables? Explain your answer.
City c The cost of running the museum increases
January April July
each year. Is raising the price of tickets the
London 9/5 15 / 7 23 / 15
best way to cover the increase in running
Chicago 1 / 11 15 / 4 29 / 17 costs? Explain your reasoning.
Sharm el-Sheikh 22 / 13 30 / 20 38 / 27
6 R The tables show the profits of a small
Sydney 26 / 19 23 / 15 17 / 8 publishing company over a 10-year period.
a What type of statistical diagram would be Year 2004 2005 2006 2007 2008 2009
a good way for Tina to display this data?
Profit () 110 575 1500 3400 3620 4340
b Draw a statistical diagram to display the
data. Year 2010 2011 2012 2013 2014
c Which city has the widest range of Profit () 6120 9435 12350 18500 23900
temperatures throughout the year?
a Use the tables to draw a time series graph of
d London is in the Northern Hemisphere. the profits made by the publishing company.
Bylooking at your diagram, how can
b What does the graph show you?
you tell which city is in the Southern
c Predict the profit for 2015.
Hemisphere?
d A large publishing company is looking to buy
smaller companies. Would this be a good
company to buy? Explain your reasoning.

R = Reasoning P = Problem-solving 3 Problem-solving 27


7 Exam-style question 9 The frequency table shows the heights of
sunflowers grown by a Brownie pack.
The frequency table gives information
about the times it took some office workers Height, h (cm) Frequency
to get tothe office one day.
150 < h , 180 2
Time, t (minutes) Frequency
180 < h , 210 4
0 , t < 10 4
210 < h , 240 7
10 , t < 20 8
240 < h , 270 8
20 , t < 30 14
30 , t < 40 16 270 < h , 300 11
40 , t < 50 6 300 < h , 330 3
50 , t < 60 2 330 < h , 360 1
a Copy the axes below and draw a a How many sunflowers were grown?
frequency polygon for this information.
b Which is the modal group?
c Estimate the range.
25
d Draw a frequency polygon for the data.
Frequency

20
e Was the number of sunflowers 240cm or
15
taller greater or less than the number that
10 were shorter than 240cm?
5 Explain how you know from looking at
0 thegraph.
0 10 20 30 40 50 60 70
Time (minutes)  (2 marks) 10 Stacey, Paula and Mark are looking at the
b Write down the modal class same scatter graph. They each draw a line of
interval. (1 mark) best fit.
One of the office workers is picked at random.
c Work out the probability that this Mark
worker took more than 40 minutes to get
to the office. (2 marks)
November 2012, Q12, 1MA0/2H
Stacey

8 R If you continue to write the arithmetic


sequence 4, 11, 18, 25 , will you eventually Paula
write 500?
Explain with reference to the nth term. a Who has drawn the line of best fit?
b Is there a correlation between the two
variables? If so, what type of correlation is
it and what does it mean?

Q10a hint What are the features of a line of


best fit?

28 R = Reasoning P = Problem-solving 3 Problem-solving


4 FRACTIONS, RATIO AND
PERCENTAGES
4.1 Fractions 10 Exam-style question
Sally has ordered a wedding cake.
She has asked that the cake be between
1 Simon and Cathy each buy an identical pizza.
8_2inches and 9 inches high.
1
Simon eats _25of his pizza and Cathy eats _38of The baker makes these three layers.
her pizza.
How much pizza is left? 1
34 3 7
28
Give your answer as a mixed number.
All measurements are in inches.
2 Find the reciprocal of each number.
When the layers are placed on top of one
a 5 b 0.125 c 2.25 d _56 another, will they make a cake that is
Use a calculator to check your answers. between 8_2and 9inches high?
1

You must explain your answer. (5 marks)


3 Find the reciprocal of
a _14 b _35 c
__
11
4_56
8 d
Exam hint
4 Giving your answer as a mixed number where Explain your answer by showing your calculations.
appropriate, work out Write a sentence, The cake will/will not be
a 2_15 _34 b
1_38 _45 between 8_21and 9inches high because

c 3_47 1_25 d
3__
7 _5
10 26 11 Work out these subtractions. Example
5 Work out a 5_34 2_16 b
6_78 1_35
a 3_14 8 b
10 _59 c 2_45 3__
3
2_16 5__
10 d
7
12

6 Giving your answer as a mixed number where


appropriate, work out
12 P Brenda earns a fixed amount each month.
a 1_18 _29 b 4_37 1_13
She spends _38of this amount on her mortgage.
c 2_56 __ 3
3_56 5__
10 d
1
12 Out of the remaining amount, she saves __ 5
12.
7 Samir says, Multiplying a fraction by its She spends the rest on supermarket shopping.
reciprocal always results in an answer of 1. Brenda spends 350 per month on
Is he correct? supermarket shopping.
Show some working to explain your answer. How much does she spend on her mortgage
each month?
8 Copy and complete the calculation.
2_78+ 4__
5 13 P Tom sorted out bags of donated clothing
16
at a homeless shelter.
7 5
= __ + ___ Out of 6 bags, 2_14were suitable for men,
8 16
5 1_13were suitable for women and the rest were
= + ___
___
16 16 for children.
= ___
a How many bags of clothes were for children?
16 Tom spent 3_12hours sorting out clothes and
= __
2_13hours working in the kitchen at the shelter.
He also took two 10-minute breaks and had
9 Work out _34hour for lunch.
a 2__
7 _1 3_23+ 4__
12+ 16 b
11
12 b How many hours did he spend in total at
_
2 _4
c 37+ 25 d _ 5 _
28+ 356 the homeless shelter?

R = Reasoning P = Problem-solving 4.1 Fractions 29


8 A scale model of a boat is 37cm long.
4.2 Ratios Thereal boat is 7.4m long.
a Work out the scale of the model.
1 Write each ratio in the form 1:n. Write it as the ratio of real length to
a 5:30 b 32:16 modellength.
c _14:3 d _
85:_35 The boat is 3.2m wide in real life.
2 Write each ratio in the form n:1. b How wide is the model?
a 16:4 b 15:60 9 The ratio of DVDs to CDs in Sams collection
c 4:_18 d
_
25:_49 is1:_58.
Sam has 135 CDs.
3 Write these ratios in the form 1:n. How many DVDs does Sam have?
a 2:80p
b 2kg:400g 10 Jana splits 350 between her two nieces in
c 3 hours:20 minutes the ratio of their ages.
d 15cm:6.6m Carlotta is 16 and Hannah is 12.
a What fraction does Carlotta get?
4 R On Saturday at 12pm, there were 45 staff
members and 375 customers in a department b What fraction does Hannah get?
store. c How much money does each niece get?
a Write the customer:staff ratio in the form Example
n:1. 11 Share 342 sweets between
Another department store had 70 staff Finn and Rosie in the ratio 5:4.
members and 637 customers at the same How many sweets does each
time. child get?
b Which store had more customers per staff
member? 12 Sahira needs to cut a length of wood into
twopieces in the ratio 5:7.
5 P Cheryl and Jon both make hair dye by
The wood is 3600mm long.
mixing hair colourant with peroxide.
How long will each piece be?
Cheryl uses 750ml of colourant and 450ml of
peroxide. 13 Share each quantity in the given ratio.
Jon uses 850ml of colourant and 650ml of a 451 in the ratio 4:5:2
peroxide.
b 52.60 in the ratio 4:1:3
Who has the greater concentration of
peroxide in their hair dye? c 930mm in the ratio 2:3:5
d 885kg in the ratio 8:2:2
6 R Jackie and Alan share the profits of their
business in the ratio of the amounts they 14 Exam-style question
invested.
Jamal makes a batch of scones.
Jackie invested 170 and Alan invested 150.
He needs to mix butter, sugar and flour in
Alan gets 18000.
the ratio 3:2:15 by weight.
How much money does Jackie get? Jamal wants to make a scone mix weighing
7 To make concrete, Ali mixes 2 parts of cement 500g in total.
with 5 parts of sand. He has
a Write down the ratio of sand to cement. 85g of butter
b To lay the foundation for an extension, 45g of sugar
Aliuses 60kg of sand. How many 400g of flour.
kilograms of cement does she use? Does Jamal have enough butter, sugar and
c On another project, Ali used 75kg of flour to make the scone mix? (4 marks)
cement. How much sand did she use?

30 R = Reasoning P = Problem-solving 4.2 Ratios


15 Write each ratio as a whole number ratio in 5 P Stuart is a plumber and is paid the same
its simplest form. hourly rate for any work he does.
a 40:22.5 b 83.25:11 For one job he is paid 712.50 for 15 hours
c 14.4:73.5 d 60.2:2.56 work.
a What fraction of this amount is Stuart paid
Q15a hint Multiply first by powers of 10 for 8 hours work?
to make both sides of the ratios whole
b Work out how much he is paid for 8 hours
numbers, then simplify.
work.
40 : 22.5 Karen also works as a plumber.
310 310
400 : 225 She gets 577.50 for doing 11 hours work.
4 4
16 : c Which plumber gets the better hourly
rate?
16 R Jude wants to make lilac paint. 6 A pattern is made from black squares, b, and
She is going to mix red paint, blue paint and white squares, w, in the ratio 2:5.
white paint in the ratio 0.8:1.5:5.7. Copy and complete.
Copy and complete the table to show how w = b = b
much of each colour Jude needs to make the
b = w = w
paint quantities shown.
Size Blue Red White 7 R Brian makes Mexican snacks by filling
tortillas with a mixture of spiced meat and
1 litre
cheese. For every 4 tablespoons of meat,
2.5 litres Brian uses 3 tablespoons of cheese.
a Write a formula for c, the number of
4.3 Ratio and proportion tablespoons of grated cheese used with
m tablespoons of spiced meat.
b Brian has 10 tablespoons of spiced meat.
1 The exchange rate between pounds and How many tablespoons of grated cheese
euros () is 1 = 1.28. does he need?
a Convert 500 to euros. Brian wants to make the mixture twice
b Convert 576 to pounds. as cheesy, so he doubles the number of
2 P Cassie buys a smartphone in the UK for tablespoons of cheese.
449. She sees the same smartphone on sale c Write a formula for the new recipe.
in NewYork for US$499.
8 Are these pairs of quantities in direct
The exchange rate is 1 = US$1.53. proportion?
By how much is the smartphone cheaper in a 12 hotdogs cost 21.60, 15 hotdogs cost
New York? 27.00
3 R Bron and Jill are both training to run a b 5 apples cost 1.60, 9 apples cost 2.97
marathon. In one week Bron runs 83 miles c Tom took 45 minutes to run 10km,
and Jill runs 122km. 5 miles = 8km. Rictook 1 hour 2 minutes to run 14km
a Write the ratio of kilometres to miles in
the form 1:n. 9 In a science experiment, the speed of a steel
ball bearing is measured at different times
b Work out who has run further in the week
after it is dropped.
and by how much.
The table shows the results.
4 R Hal makes a fruit punch for a party by
Time,
mixing orange juice with raspberry juice in 1 2 3 4 5
t (s)
the ratio 7:3. Lyn also makes a fruit punch
by mixing raspberry juice and orange juice in Speed,
9.8 19.6 29.4 39.2 49
the ratio 10:21. s (m/s)
Will the two fruit punches taste exactly the Are time and speed in direct proportion?
same? Explain your answer. Explain your answer.

R = Reasoning P = Problem-solving 4.3 Ratio and proportion 31


10 The table gives readings A and B in a science 2 Exam-style question
experiment.
Monica booked a trip to Disneyland for
A 4 9 13 herself and her family.
B 10 22.5 32.5 The total cost of the trip was 5800 plus
VAT at 20%.
a Are A and B in direct proportion?
Monica paid 1300 of the total cost when
Explain.
she booked her holiday.
b Write a formula for B in terms of A. She paid the rest of the total cost in 6 equal
c Write the ratio A:B in its simplest form. monthly payments.
11 The values of X and Y are in direct proportion. Work out the amount of each monthly
Work out the missing values of p, q, r and s. payment. (5 marks)

Value of X p 20 45 r 108
3 A train ticket normally costs 182.
Value of Y 10 q 24 54 s
Georgegets a 30% discount.
12 The cost of wood is directly proportional How much does George pay for the train ticket?
toits length.
A 3000mm piece of wood costs 58.50. 4 R Dan has 2500 invested for 2 years.
Work out the cost of 10000mm of this wood. Theinvestment increases by 5% each year.
Work out the value of the investment after
13 P The weight of a metal rod is directly a 1 year b 2 years.
proportional to its volume.
A metal rod weighs 350g and has a volume of 5 a Work out the amount of simple interest
15.71cm3. earned in one year for each of these
Work out the volume of another rod made investments.
from the same metal that weighs 475g. i 450 at 3% per year
Write your answer to 2 d.p. ii 6000 at 7.5% per year
b Gary invests 23750 for 3 years at 4.25%
14 Exam-style question simple interest.
Bob is on holiday in Spain. How much is his investment worth at the
He pays 51.98 for 45 litres of petrol. end of the 3 years?
When Bob returns to the UK, he pays
44.03 for 37 litres of petrol. 6 P Income tax is paid on any money you earn
The exchange rate is 1 = 1.28 over your personal tax allowance.
In which country is it cheaper to buy In 201415, the personal tax allowance was
petrol? (4 marks) 10000.
Above this amount, tax is paid at different
rates, depending on how much you earn.
Q14 hint You only need to convert one of The table shows the tax rates you paid in
the prices into the other currency, not both, each band in 201415.
before you look at the amount of petrol.
Tax rate Taxable income
Personal allowance 0% Up to 10000

4.4 Percentages Basic rate 20%


Higher rate 40%
10000 to 41865
41865 to 150000

1 Tariq gets an electricity bill. Work out the amount of income tax each of
The cost of the electricity used before the these people paid in the 201415 tax year.
VATis added is 493.80. a Harry earns 18900 per annum.
VAT is charged at 5% on domestic fuel bills. b Sheila earns 27950 p.a.
What is the cost of the electricity bill, c Claire earns 57560 p.a.
including VAT? d Aylish earns 85735 p.a.

32 R = Reasoning P = Problem-solving 4.3 Ratio and proportion


7 Maddy saves 4500 in an account. 14 P In 2013 Sue bought a car. In 2014 the car
At the end of one year, the balance of her depreciated by 15%. In 2015 it depreciated by
account is 4680. 6%. The value of the car in 2015 was 7990.
a What is the actual increase? a What was the total percentage decrease?
b Work out the percentage increase. Do not round your answer.
b Work out the value of the car in 2013.
8 In 2014, a local council recycled 14.28 million
tonnes of refuse. 15 R a Show that applying a 10% decrease
In 2004, they recycled 6.53 million tonnes of followed by a 10% increase is the same as
refuse. a 1% decrease overall.
What was the percentage increase over the b Will applying a decrease by a fixed amount
decade? and then an increase by the same fixed
amount always result in a decrease overall?
9 Cerys bought an electric guitar for 380.
Twoyears later she sold it for 209.
What was her percentage loss? 4.5 F
 ractions, decimals
10 Sean spent 91.35 buying ingredients to and percentages
cooka charity dinner.
He sold tickets for the dinner for a total of 1 R Georgia bought 36 boxes of chocolates
134.61. from the cash and carry.
a How much profit did Sean make? Each box of chocolates cost her 1.50.
b What was Seans percentage profit? She sold _23of the boxes of chocolates for
7.50 each and another 7 boxes for 5 each.
11 R In 2009 Mike earned an annual salary of
She gave the rest away.
11400.
a How much profit did she make?
In 2015 Mikes salary had increased by 300%.
b Express this profit as a percentage of the
Mike says his salary in 2015 is 3 times the
total cost price.
amount it was in 2009.
His boss says his salary in 2015 is 4 times the 2 Exam-style question
amount it was in 2009.
60 students took a maths test.
Who is correct?
20% of the students scored 70 or more.
_3
12 Will pays 66.78 for a portable Example 4of the students scored between 55 and 69.
hard drive. The rest scored less than 55.
This price includes VAT at 20%. Work out the number of students who
What was the cost of the hard scored less than 55. (4 marks)
drive before VAT was added?
3 Exam-style question
13 Jake invested an amount of money
2 years ago. Tom works out he spends 0.75 hours out of
every 3 hours he is awake playing computer
In the first year the amount increased
games.
by 5%.
He also works out that, out of the time he
In the second year, the amount decreased
plays computer games, he spends _5of the
3
by8%.
time playing a war game.
Copy and complete the calculation to work Tom is awake for _3of the day.
2
out the overall percentage decrease over
What percentage of each day does Tom
these 2 years.
spend playing the war game? (3 marks)

x 31.05 30.92
4 Exam-style question
Which is closer to 65%,_3or __
2 9
14?
3
You must show your working. (3 marks)

R = Reasoning P = Problem-solving 4.5 Fractions, decimals and percentages 33


5 P Sally runs a hairdressing business.
The table shows the amount of profit she 4 Problem-solving
made in 2013 and 2014 for three different
services she offers. Solve problems using these Example
strategies where appropriate:
Service 2013 2014
Cut and blow dry 4800 6400
Use pictures or lists
Colour and highlights 7500 5800 Use smaller numbers
Use bar models.
Wedding hair styles 3200 8500
Write three sentences comparing the profits 1 R Malcolm finds the times (rounded to
in the two years. the nearest minute) that 10 people take to
Use fractions, decimals, percentages, ratio or complete a half-marathon.
proportion. 97, 108, 10, 77, 104, 107, 110, 106, 113, 89
a The outlier was recorded wrongly.
6 P Work out _25of 0.45 of 96% of 200. Suggest what it might have been.
Show all your working out. b Work out the range and the median time
taking into account the corrected value.
7 R Work out
9(__
1 ___
1
+ ___
10+ 100
1
+ )
1000 2 The standard rate of value added tax (VAT) in
as an integer to 1 s.f. (where indicates that the UK is 20%. VAT is reduced to 5% on some
the sequence goes on forever). items, such as childrens car seats.
Explain your answer. If a childs car seat costs 48 before VAT is
added, how much more would it cost if the
8 R Two variables v and t are connected by standard VAT rate were charged instead of
theformula the reduced rate?
v = 10t
a Are v and t in direct proportion? Q2 hint A bar model might help.
Explain.
b Write t as a fraction of v. 3 The official width-to-length ratio of the flag
c Write the ratio t:v. of Portugal is 2:3.
The official width-to-length ratio of the flag
9 Write these recurring decimals Example of Iceland is 18:25.
as exact fractions. A gift shop sells official flags of both
Write each fraction in its Portugaland Iceland.
simplest form. Each flag has a width of 54cm.
a 0.2
0.5
b a Which flag is longer?

c 0.45

d
0.27272727... b How many cm longer?
e 0.85
2
0.171
f
4 Exam-style question
Q9 hint Multiply by a power of ten. A company surveyed 380 people.
If 1 decimal place recurs, multiply by 10. _3
5of the people surveyed were aged between
If 2 decimal places recur, multiply by 100. 18 and 65 years.
If 3 decimal places recur, multiply by 1000. 10% of the people surveyed were aged
under 18 years.
10 Which of these fractions are equivalent to How many of the people surveyed were
recurring decimals? aged over 65 years? (2 marks)
Show your working out.
a __
3
20 b _37 c __
31
40 d __
4
19

34 R = Reasoning P = Problem-solving 4.5 Fractions, decimals and percentages


5 Mirka is shopping online. She has 80. 8 R Lionel draws three scatter graphs.
Sheis deciding whether to order an item GraphA has a positive correlation, graph B
from Australia or from Germany. has a negative correlation and graph C has no
From Australia, the item she wants costs correlation.
115Australian dollars (A$115) plus A$20 One of Lionels graphs shows the relationship
delivery. From Germany, the item she wants between age (1835 years) and income.
costs 80 plus 11 delivery. Which of the three graphs is this likely to be?
The exchange rates are 1 = A$1.8 and Explain your choice.
1 = 1.3.
9 R Shola and Paul are factorising 8pq3 + 12p2q.
a How much money will Mirka have left if
she orders the item from Australia? Shola says that the highest common factor of
the terms is 4p. Paul says it is 4q.
b How much money will Mirka have left if
she orders the item from Germany? a From just looking at the expression, is
either of them correct? Explain your reason.
6 One week Eoin paid 74.92 for his weekly b What is the factorised answer?
groceries.
10 Mike spends 450 hours working on a
The next week he paid 61.32.
construction project. He spends some time
What is the percentage decrease in Eoins
on the construction site and some time in
grocery bill?
the office. When Mike calculates the fraction
7 These rectangles have the same area. of the time he spent working on site as a
Find x. decimal, he gets 0.666666666
a What fraction of the total time did Mike
x 1 10
spend working on site?
x12 x
b How many hours did Mike spend working
x14 on site?

5A
 NGLES AND TRIGONOMETRY
2 R ABCD is a parallelogram.
5.1 Angle properties B C
of triangles and
quadrilaterals
75
A D
1 ABC and CDE are straight lines.
AE is parallel to BD. a Copy the parallelogram and extend each
side as shown below.
E
B C

D
81

75
68 A D
A
C B
b Work out the other angles in the
Work out the size of parallelogram.

a AB b
D D
AE c What do you notice about the opposite

c BD d
C E
AC angles?
d Repeat with different parallelograms.
Is your observation in part c still true?

R = Reasoning P = Problem-solving 5.1 Angle properties of triangles and quadrilaterals 35


3 Triangle BCF is shown. AD is parallel to EG. b S
B C
A D

R 114
y
x z
E G
F
a What is the value of x + y + z?
Give a reason for your answer.
b Write the size of each of these angles in
terms of x, y and z.
i angle FBC ii angle FCB 63 z
Q P
Give reasons for your answers.
c Use your answer to part a to derive the V
c
sum of the angles in a triangle.
4 Exam-style question
28
B
A C
x

x
111 S
D F T U
E
d H
ABC and DEF are straight lines.
AC is parallel to DF. BE = BF.
Calculate the size of the angle marked x.
You must give reasons for your answer.
 (4 marks)
w G
5 R In this diagram a diagonal divides the F
quadrilateral into two triangles. v
f D E

d
b 7 R ABCD is a parallelogram. Example
CDE is an isosceles triangle.
ADEF is a straight line.
a
c e
Angle BAD = 64.
B C
Use the diagram to prove that the angle sum
of a quadrilateral is 360.
6 Work out the size of each angle marked with
a letter. 64
a B A D E F

Work out the size of angle CEF.


36
Give reasons for your working.

71 y
A D
C

36 R = Reasoning P = Problem-solving 5.1 Angle properties of triangles and quadrilaterals


8 R Work out the size of angle ABD. 3 A regular polygon has 16 sides.
Give reasons for your working. a Work out the sum of the interior angles of
D the polygon.
b Work out the size of each interior angle.
72
Q3a hint Substitute into (n 2) 180.
156
F C
E
4 R Work out the size of each interior angle of
a a regular hexagon
b a regular nonagon
x c a regular decagon
A B
d a regular polygon with 18 sides.
9 Work out the size of angle ADE.
5 R Work out the size of each unknown
Give reasons for your working. interior angle.
A a
106
125 116
E
x
D
a 98
x 1 10
b x
B 2x
4x
C 2x 2 20 2x x

Q9 hint Use the fact that the angles in a 6 The sum of the interior angles Example
quadrilateral add up to 360 to write an of a polygon is 1980.
equation. How many sides does the
polygon have?

5.2 Interior angles of 7 P A regular octagon is divided into


apolygon 8isosceles triangles.

1 Work out the sum of the Example


interior angles of a pentagon.
y
x
z

2 R Copy and complete the table.


Number Number of Sum of
Polygon of sides triangles interior
(n) formed angles
Work out the size of
a angle x b angle y c
angle z.
Triangle 3 1 180
Quadrilateral 4 8 R Q7 shows an octagon made from isosceles
Pentagon 5 triangles.
Hexagon 6 4 720
What polygon can you make from equilateral
triangles?
Heptagon 7

R = Reasoning P = Problem-solving 5.2 Interior angles of a polygon 37


9 Exam-style question 2 Work out the size of an exterior angle of a
regular octagon.
The diagram shows a regular pentagon and
a square. 3 Work out the sizes of the angles marked with
letters.
a
63
48

77
32
x a

82
b

Calculate the size of the angle marked x. b 97


You must show all your working. (4 marks)

5.3 Exterior angles of


apolygon 86

1 R A pentagon and a hexagon are shown.


104
a
130
e
70 b
120 Q3a hint a + 32 + 48 + 63 + 77 + 82 = 360
Q3b hint To find angle b, first work out the
exterior angle not marked with a letter.
d 130
c 4 R The sizes of five of the exterior angles of a
hexagon are 35, 79, 21, 95 and 54.
Work out the size of each interior angle.
u
p 5 R Work out the size of each unknown
110 140
t exterior angle in this polygon.
110
130 q

s 130
100 50
r x 1 20

a Work out the sizes of the angles marked


with letters.
x
b Work out the sum of the exterior angle for
each polygon. 2x 2 30
c What do you notice about the sum of the
exterior angles?

38 R = Reasoning P = Problem-solving 5.2 Interior angles of a polygon


6 How many sides does a regular Example 2 ABC is a right-angled triangle.
polygon have if its exterior A
angle is
a 36 b 15 c 9?
5.4 cm
7 How many sides does a regular polygon have
if its interior angle is
a 60 b 156 c 170? B C
7.32 cm
8 Can the exterior angle of a regular polygon Calculate the length of AC.
be 50? Explain.
Give your answer correct to 3 significant
9 P One side of a regular pentagon ABCDE forms figures.
the side of a regular polygon with n sides.
3 Calculate the length of the diagonal of a
A
square with side length 6.3cm.
Give your answer correct to 3 significant
E B figures.
4 A guy rope is attached to the top of a 10m
D C flagpole. It is anchored to the ground 4m
G F from the base of the flagpole.
What is the length of the guy rope?
5 P Ashley walks 1.8 miles east and then
Angle BCF = 90. Work out the value of n.
2.4miles north. He then walks straight back
10 P The interior angle of a regular polygon is to his starting point.
3times the size of its exterior angle. How far does Ashley walk in total?
How many sides does the polygon have?
6 P A garden is in the shape of an isosceles
triangle.
5.4 Pythagoras theorem 1 Calculate the perimeter of the garden.

1 Calculate the length of the Example


hypotenuse in each triangle. 6m
Give your answers correct
to 1 d.p.
a
11 m

5 cm 7 Exam-style question
ABCD is a trapezium.
8 cm AB = 5cm
b c 6.5 cm BC = 6cm
AD = 10cm
B 6 cm C
7 cm

7.8 cm 5 cm

3 cm
A D
10 cm

Q1 hint Do not round before taking the Calculate the perimeter of ABCD.
square root. Use all the figures on your Give your answer correct to 1 decimal
calculator display. place. (3 marks)

R = Reasoning P = Problem-solving 5.4 Pythagoras theorem 1 39


8 R Can a right-angled triangle have sides of 4 Calculate the vertical height of trapezium
length ABCD.
a 5cm, 7cm, 9cm b 6cm, 8cm, 10cm Give your answer in centimetres, to the
c 7cm, 8cm, 11cm? nearest millimetre.
Explain your answers. 13 cm
A B

5.5 Pythagoras theorem 2 9 cm

1 Exam-style question
D C
A 18 cm

Example 5 P a Calculate the length of the side of the


largest square that fits inside an 8cm
9 cm diameter circle.
b Work out the length of the side of the
smallest square that surrounds an 8cm
diameter circle.
C 4 cm B
6 Work out the length of the unknown side in
ABC is a right-angled triangle.
each right-angled triangle.
BC = 4cm
Give your answers in surd form.
AB = 9cm
a C b A B
Calculate the length of AC.
Give your answer correct to 3 significant
figures. (3 marks)
6 cm
8 cm
2 A ladder of length 6m leans against a vertical 8 cm
wall. The top of the ladder touches the wall
5.3m above the ground. C
How far is the base of the ladder from the wall? A B
4 cm

c A 13 cm
B

11 cm

6m 5.3 m
C

Q6a hint Simplify the surd__ so your answer


looks like this: AC = cm

7 P ABC is an isosceles A
3 The screen of a TV set has a diagonal length
triangle.
of 1.22m and a height of 60cm.
D is the midpoint of BC.
Work out the height
of the triangle.
1.22 m 8 cm 8 cm
60 cm Give your answer h
in surd form.

How wide is the TV screen?


B D C
Give your answer in metres, to the nearest
centimetre. 6 cm

40 R = Reasoning P = Problem-solving 5.4 Pythagoras theorem 1


8 P Work out b c
a the length of BD b the length of CD
c the perimeter of the quadrilateral. x
Give all your answers to 3 s.f. 15 cm
x
C 29
7 cm
D 68
8 cm
Q3b hint Q3c hint
5 cm opp opp
Use tanu=____ Use sinu=____
adj
 hyp
A B
9 cm
4 Calculate the length of the side marked x.
Give your answer correct to 1 decimal place.
5.6 Trigonometry 1
1 R Draw triangle ABC accurately using a ruler
and protractor. x
Angle A = 90, angle B = 40 and AB = 4cm.
a Label the hypotenuse (hyp), opposite side
(opp) and adjacent side (adj). 41
b Measure each unknown side to the nearest 12 cm
millimetre.
c Write the fraction 5 Calculate the length of the side marked x in
opposite adjacent each triangle.
i __________


ii __________

Give your answers correct to 1 d.p.
hypotenuse hypotenuse
opposite a
iii ________

40
adjacent
Convert each fraction to a decimal. x
Give your answers correct to 1 decimal place. 3.8 mm
d Repeat parts a to c for triangle ABC with
i angle A = 90, angle B = 40 and
AB = 5cm b
ii angle A = 90, angle B = 40 and 58
AB = 7cm.
2 Use your calculator to find, correct to 2 d.p.
where necessary
x
a sin25 b cos15 c tan38 6.5 mm
d cos76 e tan83 f sin63
3 Calculate the length of the side Example
marked x in each triangle.
Give your answers correct to
3 significant figures.
c 72
a x
8.1 cm
35

8 cm Q3a hint
x adj
Use cosu=____
hyp

R = Reasoning P = Problem-solving 5.6 Trigonometry 1 41


6 Exam-style question 2 Calculate the size of angle x in each triangle.
Give your answers correct to 1 d.p.
a Example
38
8 cm
5 cm
17 cm
x

b
x
11 cm
Calculate the value of x. 5 cm
Give your answer correct to 3 significant
figures. (3 marks)

7 P / R A tent makes c
an angle of 65 with
the horizontal.
The tent is 2.1m 2.5 m 2.5 m 8.3 cm
wide and the
sloping sides are
2.5m long. x
65 65
Calculate the 5.8 cm
height of the tent. 2.1 m

8 A 6m long ladder is leaning against a wall. 3 Exam-style question


The angle between the base of the ladder P Q
and the ground is 78. y
How far up the wall does the ladder reach?
8.6 m
22.4 m

PQR is a right-angled triangle.


PR = 8.6m.
6m QR = 22.4m.
Calculate the size of the angle marked y.
Give your answer correct to 1 decimal
place. (3 marks)
78
4 P Elena is flying a kite.
The kite string is 30m
long and the kite is 25m
5.7 Trigonometry 2 above Elenas hand.
Work out the angle
1 Use the inverse function on your calculator to between the kite 30 m
25 m
find the value of u correct to 0.1. string and the
a sinu = 0.436 b cosu = 0.228 horizontal.
x
c tanu = 0.763 d sinu=_34
e cosu=__
4
tanu=__
13 f
11
8

42 R = Reasoning P = Problem-solving 5.6 Trigonometry 1


5 P A cliff is 40m high. 10 ABC is an isosceles C
Work out the angle of elevation of the top of triangle.
the cliff from point A. a Use the diagram 45
to write the
value of 1 cm
tan45.
40 m
b Use Pythagoras 45
a theorem to find A 1 cm B
A the length of BC.
100 m
Leave your answer in surd form.
6 P A ship sails 7km north and then 10km c Write these ratios as exact values using
east, from A to B. surds.
Another ship sails directly from A to B. i sin45 ii cos45
N 11 ABC is a triangle with AC = 1cm, BC = 2cm
10 km and angle ABC = 30.
B
C

7 km 2 cm
1 cm
30
x A B
A a Use the diagram to write these ratios as
fractions.
a How much further does the first ship
i sin30 ii cos60
travel?
b Work out the length of AB.
b On what bearing from north (x) should the
Leave your answer in surd form.
second ship travel?
c Write these ratios as exact values using
Give your answers correct to 1 decimal place.
surds.
7 P Natasha stands at the top of a 61m tower. i sin60 ii cos30
Dominic is standing on the ground 75m from iii tan60 iv tan30
the base of the tower.
12 Find the exact value of x in these triangles.
What is the angle of depression of Dominic
a b
from Natasha?
x
8 P Work out the area of this isosceles triangle.
2
1
3

6 cm x
1
c
x
72

9 P Calculate the size of angle u in this diagram. 2 2

11 cm 9 cm

Q12a hint Which angle has this exact value


6 cm for its tangent? Look back at Q11.

R = Reasoning P = Problem-solving 5.7 Trigonometry 2 43


4 R The local election results were displayed
5 Problem-solving in the newspaper using this bar chart.

Solve problems using these Example 756


strategies where appropriate: 754
Use pictures or lists 752
Use smaller numbers 750
Use bar models
Use x for the unknown. McCline Warren

1 R Neil is building a rabbit hutch in the shape The headline read, McCline storms ahead in
of a triangular prism. The end view of the local election.
hutch makes an isosceles triangle with a base How is this bar chart misleading?
of 80cm. The angle of the roof of the hutch is
5 R Geometric tiling is used all over the world
76. Part of the side panel folds down to be
to create patterns. A geometric pattern has
flat on the ground.
been started below with a regular hexagon
Neil wants to know the exterior angle that is and a square.
made when the side panel is open, to make
a Find angles a and b.
sure he has the correct hinge.
a Use the information given to draw an end
view of the hutch with the door down.
Usea scale of 1cm:10cm.
Label all known interior angles of the
triangle. a b
c
Use the diagram below as a starting point.
b Use angles a and b to find angle c.
c Use angle c to find which regular polygon
will fit to join the two shapes.

6 Ben is putting up his tent.


The front centre pole is 1.2m high.
b Find the angle the side panel makes when Ben attaches a 2m guy line to the top of the
it is fully open. pole and pegs it out.
How far from the bottom of the pole does
Q1 hint Use x for the exterior angle. Ben put the peg for the guy line?

2 R Pia is part of a calling tree. The first person 7 Juan pours 6_16litres of water into some jugs.
calls three people, then each person in this Each jug can hold 1_14litres.
second round calls three people and so on. How many jugs can Juan fill?
a How many people have been called after
the fifth round? 8 The Elizabeth Tower in London is 96.3m tall.
b Write the expression to find the number of From a point on the ground 25m away,
people called in the nth round. what is the angle of elevation to the top of
thetower?
3 R Use the information in the diagram to find Give your answer to the nearest whole
angles a, b and c. number.
c

48

a b

44 R = Reasoning P = Problem-solving 5 Problem-solving


9 Sketch a right-angled triangle. 10 Exam-style question
Label the triangle ABC with the hypotenuse AC.
AB = 4cm R S

BC = 6.5cm
6.9 cm
a Find the length of the hypotenuse to 1 d.p. 11.5 cm
b Find the angles of the triangle to 1 d.p. x
Diagram NOT
accurately drawn
Q9 hint We know one of the angles is 90 as Q
it is a right-angled triangle. Use SOHCAHTOA QRS is a right-angled triangle.
to find one of the missing angles and then
Calculate the size of the angle marked x.
subtract from 180.
Give your answer to 1 decimal place.
 (3 marks)

6 GRAPHS
3 a Match each line to an Example
6.1 Linear graphs equation.
C y A B
1 On squared paper, draw a line with gradient 8
a 2 7 D
b _13 6
c 1 5
E
2 Copy and complete this table for the graphs 4
on the grid. 3
2
Equation of line Gradient y-intercept
1
y = 3x 2
24 23 22 21O 1 2 3 4 x
y = 3x 21
22
y = 3x + 1
23
CB A 24
y
25
4
26
3
27
2
28
1

24 23 22 21O 1 2 3 4 x y 5 3x 1 3 y 5 2x 1 1 y 5 2x 1 1
21
22
23 y 5 22x y 5 2x 2 1
24
b Which line passes through the origin?
c Which line is the steepest?
d Which lines have the same y-intercept?
e Which lines are parallel?

R = Reasoning P = Problem-solving 6.1 Linear graphs 45


4 Write the equations of these lines. 8 R Which is the steepest line?
y A y = 10 2x B 2 + 3x = 3y
C A B
5 C y = _12x D 6x y = 12
4 E x + y = 15
3
Q8 hint Rearrange to y = mx + c if necessary.
2
1
9 P Which of these lines pass through (0, 5)?
25 24 23 22 21O 1 2 3 4 5 x Show how you worked it out.
21
A y = 2x 5 B 2y 3x = 10
22
C 3y = 15 + 6x D x + y = 5
23
E 2x 10 = y
24
25
6.2 More linear graphs
F y
5
1 Draw these graphs from their Example
4
E D equations.
3
Use a coordinate grid from
2
10 to +10 on both axes.
1
a y = 2x + 1 b y = x + 2
25 24 23 22 21O 1 2 3 4 5 x c y = 5 d y=2+x
21
e y = 3 x f
y = 3x + 1
22
23 2 R Match each equation to one of the
24 sketchgraphs below.
25
y 5 3x 1 1 y5x11 y 5 2x 1 1

5 Here are the equations of some linear graphs. y 5 2x 2 1 y 5 2x


A y = x + 2 B y = x C y = 4x + 2
D y = 4x 2 E y = 4x 2 A B C
Which of these graphs y y y
a cross the y-axis at the same point 1 x
b are parallel? 1 21
x x
6 a For the equation 3y 6x = 12
i copy and complete the table of values
D E
x 0 y y
y 0
1
ii plot the graph on suitable axes. x
b Repeat part a for the lines with equation x
i x y = 5 ii 2x + y = 6
7 In Q6 you drew the graphs of 3y 6x = 12,
3 Sketch the graphs of
x y = 5 and 2x + y = 6
a y = 3x b y = 2x 4 c
x+y=6
a Rearrange each equation to make y the
subject. 4 a Find the x-intercept and y-intercept of the
b Read the gradient and y-intercept from graph with equation
each. i x + y = 7 ii x y = 4
c Look back at your graphs in Q6 to check iii 2x y = 6 iv y x = 3
the gradients and y-intercepts are correct. b Sketch the graphs.

46 R = Reasoning P = Problem-solving 6.1 Linear graphs


5 R Which of these are linear functions? 13 Find the coordinates of the point where
A y = x2 B x = y + 4 C 3x = 2y 6 these graphs intersect:
10 y = 2x + 12 and 2y x = 14
D y = ___
x E xy = 5 F 2y + 7x + 10 = 0
6 Copy and complete the definition. Q13 hint Write both as y = mx + c
A linear function has a graph that is a
. line.
7 R a Does the point (1, 2) lie on the line 6.3 Graphing rates of
y = 2x? change
b Does the point (1, 4) lie on the line
y = 2x 3?
1 Aahil cycles from his house to his friends
c Does the point (1, 5) lie on the line house.
y = 3x + 2?
The distancetime graph shows his journey.
8 P A straight line has gradient 2.
Aahils cycle ride
The point (3, 7) lies on the line.
Find the equation of the line. 6

Distance (miles)
9 P Work out the equations of these straight- 5
line graphs. 4
a The line with gradient 2 that passes 3
through the point (0, 3) 2
1
b The line with gradient 1 that passes
0
through the point (4, 0) 09:00 10:00 11:00
c The line with gradient _12that passes Time of day
through the point (2, 5)
d The line with gradient 4 that passes a How far is Aahils house from his friends
through the point (1, 1) house?
b What time does he arrive at his friends
10 Find the gradient of the line joining points
house?
A (3, 1) and B (6, 5)
c How long does he take to cycle there?
a by drawing the graph and using the formula
d How long does he stay at his friends
difference in y-coordinates
gradient=_______________________

house?
difference in x-coordinates e What was his speed on the way to his
y2y1 friends house?
b using the formula m=______
x2x1
where
f Work out the gradient for his cycle ride to
A = (x1, y1) and B = (x2, y2) hisfriends house.
(3, 1) (6, 5) What do you notice?
11 R P is the point (2, 4). Q is the point (4, 0).
Q1f hint Find the time as a fraction of an
a Find the gradient of line PQ.
hour when finding the gradient.
b Write y = mx + c using your gradient from
part a. Substitute the coordinates of Q into
2 Fadeelah goes for a run.
this equation. Solve to find c.
She runs for 1 hour and travels 5 miles.
c Write the equation of the line PQ.
Shethen stops for a half-hour break.
12 To find the coordinates of the point where Thenshe runs a further 3 miles in 40 minutes.
these graphs intersect a On graph paper draw a horizontal axis
y = 2x 1 and y = x 4 from 0 to 3 hours and a vertical axis from
a write the two equations equal to each other 0to 8 miles.
b solve to find x Draw a distancetime graph to show
c substitute x into one of the first equations Fadeelahs run.
to find y b Work out her speed for the last part of her
d write the coordinates (x, y). run.

R = Reasoning P = Problem-solving 6.3 Graphing rates of change 47


3 Dillon is practising for a swimming race. 6 Look at the graph you drew for Q3. What was
Heswims 1.5km along the coast in Dillons average speed for the whole swim?
30minutes, rests for 5 minutes, then returns
at a speed of 2km per hour. 7 Ferry A travels from Ryde to Portsmouth.
a Draw a distancetime graph to show Ferry B travels from Portsmouth to Ryde.
Dillons swim. Ferry journeys to and from Portsmouth
b Work out the fastest speed he swam.
Ferry A Ferry B
Ryde 5
4 Exam-style question

Distance (miles)
4
James drove from Petersfield to London.
3
He left home at 11:00.
The graph represents part of Jamess journey. 2
Jamess drive to London 1
Portsmouth 0
70 10:00 10:30
London
60 Time
Distance (miles)

50
a Use the graph to estimate how far the
40 ferries are from Portsmouth when they
30 pass each other.
20 b Work out the speed for each part of the
10 journey for Ferry A.
Petersfield 0 c When was Ferry B travelling fastest?
11:00 12:00 13:00 14:00 Howcan you tell this from the graph?
Time
d Which ferry travelled faster on average?
At 12:00 James stopped for a rest.
a How many minutes did he rest? (1 mark) 8 R These three vases are filled with water at a
b How far was James from home constant rate.
at 13:00? (1 mark)
The last 5 miles took James 30 minutes.
c Copy and complete the graph. (1 mark)
A B C
5 The table shows a bus journey from
Bournemouth to Salisbury. The bus stops at a In which container does the depth of water
Ringwood and Fordingbridge on the way. not increase at a constant rate?
b Which graph shows this?
Bus stop Time
Bournemouth (departing) 0705 i ii iii
Depth
Depth

Depth

Ringwood (arriving) 0740


Fordingbridge (arriving) 0755
Salisbury (arriving) 0825 Time Time Time
When the bus stops it waits for 5 minutes at c Match each graph to a different container.
the stop before leaving.
The distances between stops are 9 R Here are three baths.
Bournemouth to Ringwood 12 miles
Ringwood to Fordingbridge 7 miles
Fordingbridge to Salisbury 11 miles.
A B C
a Assuming a steady speed between stops,
draw a distancetime graph for the journey. They are filled with water at a constant rate.
b Work out the speed of the bus between The baths are all the same depth.
Fordingbridge and Salisbury. On the same axes, sketch and label three
c What was the average speed for the whole graphs showing the rate at which water fills
journey? the baths.

48 R = Reasoning P = Problem-solving 6.3 Graphing rates of change


10 Donald takes his boat out on a lake. b The company orders 150 memory sticks.
The graph shows his journey. How much does this cost altogether?
Velocitytime graph for Donalds journey c Next year the company has a budget
of500.
2 How many memory sticks can they
Velocity (m/s)

order?
1 2 R The graph shows the conversion from
miles (m) to kilometres (km).
0 Conversion graph for miles and kilometres
0
0
0
0
0
0
0
0
0
12 0
00
20
12
24
36
48
60
72
84
96
8
10
Time (s) 18
16
Work out

Kilometres
14
a Donalds maximum velocity 12
b how many minutes his velocity was greater 10
than 1m/s 8
6
c his acceleration for the first part of the
4
journey 2
d the distance Donald travelled during the 0
last 8 minutes of the journey. 0 2 4 6 8 10 12 14
e Copy and complete this description of Miles
Donalds journey. a How many km are equivalent to 10 miles?
He accelerated at m/s2 for the first b How many miles are equivalent to 14km?
minutes, then travelled at a constant c Work out the gradient of the graph.
velocity of m/s for minutes. Next, ...
3 R The graph shows the cost of renting a car
Q10c and e hint Acceleration is given by the in Spain for a number of days.
change in velocity (m/s)
gradient: ____________________

Cost of car hire
time (s)
500

6.4 Real-life graphs 400


Hire charge ()

300
1 P A company is buying branded memory
sticks to give to its customers. 200
The graph shows the price per stick
depending on how many are ordered. 100

Price per memory stick 0


0 5 10 15
10 Days

8 a Calculate the gradient of the line.


Price ()

b What is the initial charge before you add


6 on the daily hire charge?
c Write down the equation of the line.
4
d Amy is going on a two-week holiday.
0 Howmuch will her car hire cost?
0 50 100 150 200 250 300
Quantity ordered
a How much would a single memory stick
cost?

R = Reasoning P = Problem-solving 6.4 Real-life graphs 49


4 Which of these graphs show one variable in c Will the graph intercept the x-axis?
direct proportion to another? Explainyour answer.
A B C d Is the rate of decrease of temperature
constant?
How can you tell from the graph?

8 R The table shows the sales of DVDs for


different prices.
Price () 5 7 9 11
D E Sales (000s) 20 15 10 5
a On a suitable grid, plot the points and
draw a graph to illustrate this information.
b Use your graph to find the price at which
sales will be zero.
The equation of the graph is in the form
y = mx + c
5 Look at the graphs in Q1, Q2 and Q3. c Use your graph to find the values of m
Which shows direct proportion? andc.
6 R A recipe for meringues uses eggs and d Explain whether your answer to part b is
sugar in the ratio 1:60. sensible.
a Copy and complete this table. 9 R The graph shows lease packages for two
Number of eggs 1 5 10 different cars, Car A and Car B.
Sugar (grams) Car B
120
b Draw a graph showing grams of sugar (y)
against number of eggs (x). 100
Car A
c Write the equation linking x and y.
d Each egg used makes 6 meringues. 80
Cost ()

What is the maximum number of meringues


60
you can make using 400g of sugar?
7 R Angelica carries out an experiment 40
recording the temperature of a cup oftea as
20
it cools.
The graph gives information about the 0
temperature, TC, of the tea over time. 0 100 200 300 400 500 600
Temperature of cup of tea Distance (miles/month)
a How much does 400 miles per month
90
80 costwith
Temperature (C)

70 i Car A ii Car B?
60 What is the practical meaning of
50 b the y-intercept value for Car A
40 c the point where the two graphs intersect?
30
20 d Another lease package, for Car C, is
10 introduced. With Car C you will pay 90 per
0 month for unlimited miles.
0 10 20 30 40 50 60 Which package should each person
Time (minutes) choose?
a What does the y-intercept tell you? Danny: Average 250 miles per month.
b Use the graph to estimate the teas Sally: Average 500 miles per month.
temperature half an hour after it is made.

50 R = Reasoning P = Problem-solving 6.4 Real-life graphs


10 R The heights and weights of six students 3 Work out the gradient of each line segment
are recorded. in Q2.
Height (m) 1.2 1.3 1.5 1.7 1.8 1.3 4 What is the length of the line segment with
Weight (kg) 32 38 50 68 70 42 end points
a E (3, 2) and F (1, 6)
a Plot a scatter graph of the data.
b G (3, 5) and H (2, 0)
b What type of correlation does this graph c J (6, 3) and K (2, 4)?
show?
c Draw in a line of best fit. 5 R A line is parallel to the line y = 2x + 1 and
passes through the point (3, 2).
d Write the equation of your line of best fit.
a Substitute the value of m for this line into
e Rebecca is 1.35m tall.
y = mx + c
What would you expect her to weigh?
b Substitute the coordinates of the known
11 Exam-style question point to work out the equation of the line.
The tables show the age and value of 12 6 R The graph shows the prices of two
cars of the same make and type. different mobile phone packages.
Write the equation for the price with Plan B.
Age
1 1 1.4 1.6 1.8 2.0 Mobile phone packages
(years)
Value Plan B
8200 9000 8000 7200 7400 6400
()

Age
2.4 3.0 3.0 3.5 3.5 4.0
Cost ()
(years) Plan A
Value
6600 6000 5200 5000 4400 3800 y 5 2x
()
10
a From this data, work out the value of
i a 5-year old car
ii a 7-year old car. (4 marks)
b Which value is more reliable? 0
Why? (2 marks) 0
Calls (hours)

7 P Write the equation of a line parallel to


y=_12x3, which passes through the
Q11 hint You could draw a graph to show
point(1, 0).
this information and extend it.
8 P Find the equation of a line that passes
through the point (3, 3) and is parallel to
the line with equation y x = 4
6.5 Line segments
9 Here are three pairs of perpendicular lines.
B2 y A2 C2
1 Work out the midpoint of a line segment AB,
4
where B1
C1 3
a A is (6, 0) and B is (10, 4)
A1 2
b A is (1, 5) and B is (2, 8)
1
c A is (7, 2) and B is (4, 2)
d A is (0, 0) and B is (7, 5). 24 23 22 21O 1 2 3 4 x
21
2 Work out the midpoint of a line segment PQ, 22
where 23
a P is (2, 0) and Q is (6, 4) 24
b P is (5, 7) and Q is (1, 9)
a Write down the gradient of each line.
c P is (4, 8) and Q is (2, 1)
b Multiply the gradients in each pair
d P is (5, 3) and Q is (2, 6).
together. What do you notice?

R = Reasoning P = Problem-solving 6.5 Line segments 51


10 Write down the gradient of a line 6 A maths student uses a graph to model the
perpendicular to trajectory of a ball.
a y = 2x + 5 b y = x + 1 c y = _13x 4 The graph shows the height, h metres, of
the ball at a time t seconds after it is thrown
11 Exam-style question straight up.
Find the equation of a line Trajectory
a parallel to the line y = 2x + 1 that passes
through (0, 5) ( 2 marks) 7
b perpendicular to y = 2x + 1 that passes 6

Height, h (m)
through (2, 7). (2 marks) 5
4
3

6.6 Quadratic graphs 2


1
0
1 Which points lie on the graph of y = x2? 0 1 2 3 4 5
A (0, 0) B (1, 2) C (2, 4) D (3, 6) Time, t (s)
E (4, 16) F (1, 1) G (2, 4) H (3, 9) a What type of graph is this?
2 a Copy and complete this table of values for b When is the ball travelling fastest?
y = x2 + 2 c When is the balls speed zero?
d What is the maximum height that the ball
x 3 2 1 0 1 2 3
reaches?
x2 e How long is the ball in the air?
+2 +2 +2 +2 +2 +2 +2 +2 f The ball was caught when it was 1.5m above
y the ground. Use the graph to estimate the
b Plot the graph of y = x2 + 2 times this could have occurred.

7 Here is the graph of y = x2 4


Q2a hint For quadratic functions with more
than one step, you can include a row for Use the graph to solve the equation x2 4 =
each step in the table. 0
y
3 Exam-style question 4
3
Draw the graph of y = (x + 1)2
for 5 < x < 3 (4 marks) 2
1

4 a Copy and complete this table of values for 23 22 21O 1 2 3 x


y = 2x2 21
22
x 2 1 0 1 2
23
y
24
b Plot the graph of y = 2x2 25
5 R Compare your graphs from Q2, Q3 and
Q4.
a What is the same about these graphs?
b Which ones have a minimum point?
Which ones have a maximum point?
c Find the coordinates of the minimum/
maximum point for each graph.
d Describe the symmetry of each graph by
giving the equation of its mirror line.

52 R = Reasoning P = Problem-solving 6.5 Line segments


8 Here are four graphs. Use these graphs to 10 Exam-style question
solve the equations
a Copy and complete the table for
a x2 3x = 0 b x2 + 3x = 0 y = x2 + 2x + 4
c x2 + 5x 4 = 0 d (x 2)2 = 0
x 4 3 2 1 0 1
a y = x2 3x y
4 y 7
3  (2 marks)
2 b Draw the graph of
y = x2 + 2x + 4 (2 marks)
1
c By drawing a suitable line on your graph,
21O 1 2 3 4 x solve the equation x2 + x 3 = 0(2 marks)
21
22 11 An ornithologist records the height, h metres,
of a bird during flight at time, t seconds, after
b y = x2 + 3x y
it is released.
4
Thetable gives the data he records.
3
2 Time, t (seconds) 0 1 2 3 4
1 Height, h (metres) 0 3.4 6.4 9 11.2
a Use this data to draw a graph showing the
24 23 22 21O 1 x trajectory of the bird.
21
22 b Continue the graph to predict the height
of the bird after 8 seconds.
c y = x2 + 5x 4 y
3
2 6.7 Cubic and reciprocal
1 graphs
O 1 2 3 4 5 x
21 1 Which of the points are on the graph of y = x3?
22 A (0, 0) B (1, 1) C (1, 1) D (2, 6)
23 E (2, 8) F (1, 3) G (1, 1) H (2, 8)
24 2 R Here is the graph of y = x3
y
d y = (x 2)2 y
30
6
5
20
4
3 10
2
1
23 22 21O 1 2 3 x
21O 1 2 3 4 x
21 210

9 R Use the graphs in Q8 to Example


220
solve the equations
a x2 3x 2 = 0 230
b x2 + 3x 4 = 0
c x2 + 4x 4 = 0 a Use the graph___ to estimate
3
i 2.53 ii 2
d (x 2)2 _12x = 0
b Use a calculator ___to work out
e Explain why x2 3x = 3 has no solutions. 3
i 2.53 ii 2

R = Reasoning P = Problem-solving 6.7 Cubic and reciprocal graphs 53


3 a Copy and complete the table of values for 9 Exam-style question
y = x3 + 1 6
a Complete the table of values for y = __
x
x 3 2 1 0 1 2 3
y 26 x 0.5 1 2 3 4 5 6
y 6 3 1.5 1
b Draw the graph of y = x3 + 1 for 3 < x < 3.
c What is the same and what is different  (2 marks)
about this graph and the one in Q2? 6
__
b Draw the graph of y = xfor
0.5 < x < 6 (2 marks)
4 a Plot graphs of y = x3 1 and y = x3 + 2 for 6
3 < x < 3. c Hence or otherwise solve __x= 4 (2 marks)
b Compare these two graphs and the graphs
from Q2 and Q3. 10 This is the graph of y = 2x3 + 7x2 4
What similarities can you see? y
What are the differences? 30
5 R a Draw a table of values Example
2 20
for y = __
x, where x 0,
for 3 < x < 3. 10
2
b Draw the graph of y = __ x
2
c Draw a table of values for y = __ x, 23 22 21O 1 2 3 x
where x 0, for 3 < x < 3.
2
d Draw the graph of y = __ x
210

e What is the same and what is different


2 2 220
about y = __ __
xand y = x?
1 By drawing suitable lines on the graph,
6 a Draw the graph of y = __ x, where x 0,
for 4 < x < 4. a solve the equation 2x3 + 7x2 4 = 0
b Use your graph to estimate the value of b solve the equation 2x3 + 7x2 4 = 10
y when
i x = 2.5 ii x = 3.5 iii x = 1.2
6.8 More graphs
7 R Match each equation to a graph.
1
a y = x2 b y = __ y = x3
x c 1 R The distancetime graphs represent
d y = 2x e y = x f y = _12x MrMurphys cycle rides on two different days.
B Distancetime graphs for cycle rides
A C
y y y
Distance (miles)

50
40
x x x
30
Day 1
20 Day 2
D F 10
E
y y y 0
0

0
:0

:0

:0

:0

:0

:0

:0

:0
10

11

12

13

14

15

16

17

x x x Time of day
a How far does he cycle on
i Day 1 ii Day 2?
8 Use the graphs you drew in Q4 to solve the b Which day does his cycle ride take longer,
equations and by how many minutes is it longer?
a x3 1 = 0 b x3 + 2 = 0 c x3 1 = 5 c Work out his greatest speed on Day 2.

54 R = Reasoning P = Problem-solving 6.7 Cubic and reciprocal graphs


d How long did he stop for in total on Day 1? 4 R The table gives some information about
e Work out his average speed during the the value of a car over 6 years from new.
whole journey on Day 1. Age (years) 0 1 2 3 4 5 6
f What does the change in gradient Value
between 15:00 and 16:00 on Day 2 show? 22 18.5 15.5 13 11 10 9.5
(000s)
2 The graph shows the height of an object a Draw axes on graph paper using 1cm to
dropped from the top of a 100m tower. represent 1 year on the horizontal axis and
When does the object reach terminal velocity? 1cm to represent 2000 on the vertical
Height of falling object axis. Plot the values from the table and
join them with a smooth curve.
From the graph, estimate
100
b the value of the car after 3.5 years
Height (cm)

c the age at which the car is worth 75% of


its original price.
5 The graph shows Bacteria growth
the number of
bacteria in a Petri 32

Number of bacteria
dish. 28
0
0 1 2 3 a How many 24
Time (s) 20
bacteria were
there at the 16
Q2 hint Any object when dropped will reach start of the 12
terminal velocity. This is the point where the experiment? 8
velocity will not increase any more and is
b Explain how 4
constant.
you found 0
0 1 2 3 4 5
your answer
3 R Here are two sets of data. Days
to parta.
Data set A c Estimate the number of bacteria after
x 3 3 4 6 6 8 9 9 i 2 days ii 5 days.
y 7 6 10 13 14 16 19 20 d Describe the change in the number of
bacteria from
Data set B i day 1 to day 2
x 3 3.1 3.3 3.4 3.4 3.7 3.8 3.8 ii day 2 to day 3
iii day 3 to day 4.
y 10.5 10.7 12.0 12.6 12.5 14.9 15.5 15.4
6 R The graph Liquid temperature
a Plot each set of data on a scatter graph.
shows the
b Describe the correlation for each set. temperature of 100
c Draw a line of best fit for the graph for a liquid during 90
data set A. What does this show? an experiment. 80
Temperature (C)

d Draw the graph of y = x2 + 1 on the same a Estimate the 70


grid as the graph for data set B. temperature 60
Copy and complete this table of values to after 50
help you. 10minutes. 40
x 3 3.1 3.2 3.3 3.4 3.5 b After how 30
many minutes 20
y
will the 10
x 3.6 3.7 3.8 3.9 4 temperature 0
have halved? 0 10 20 30 40 50
y Time (minutes)
c Does the
e What do you think the relationship is temperature
between x and y in data set B? ever reach zero?

R = Reasoning P = Problem-solving 6.8 More graphs 55


7 P The graph shows the value of a childs 1 This distancetime graph shows part of
investment. Jasons journey.
Lee made the same journey with an average
Value of investment ()

160 speed that was 2mph faster than Jasons.


140 Yasmin made the same journey at an average
120 speed of 27mph.
100
80 Jasons journey
60
40 15
20
0

Distance (miles)
0 1 2 3 4 5 6 7
Years 10

a What was the initial value of the


investment?
b What is the value of the investment after 5
5years?
c How much did the value increase each
year? 0
0 5 10 15 20 25 30 35
d The rate of interest remained the same
Time (minutes)
forthe first five years.
Work out the percentage interest rate. a Which of Lee or Yasmin was travelling at a
e What happened after 5 years? faster average speed?
b How much faster?
Q7d hint
Percentage interest rate 2 R Look at this scatter graph.
actual change It shows the attendance at an outdoor pool
= ______________

100 against the maximum daily temperature over
original amount
two weeks in the summer.
8 On graph paper, draw the Outdoor pool attendance
Example
graphs of
35
a x2 + y2 = 4
Temperature (C)

30
b x2 + y2 = 9
25
c x2 + y2 = 100
20
d x2 + y2 = 25
15
10

6 Problem-solving 5
0
0 50 100 150 200 250 300
Solve problems using these strategies where Number of people
appropriate:
a What were the approximate highest and
Use pictures lowest maximum daily temperatures
Use smaller numbers during the two weeks?
Use bar models b What were the approximate highest and
Use x for the unknown. lowest numbers of people at the pool over
the two weeks?
c Explain how you know if there is a
correlation or not.
d Make a statement summarising the results
shown on this graph.

56 R = Reasoning P = Problem-solving 6.8 More graphs


3 R A vase holds 800cm3 of water. 7 R Ryan sketches a graph to show that
The warmth of the room makes 2.5cm3 of 1 British pound = 2.06 Singapore dollars.
water evaporate each day. Which of these graphs could be Ryans
a Find an expression, in terms of n, for the sketch?
amount of water in the vase at the end of Explain how you know.
the nthday. A B C
b How many days before the vase is
completely empty?
Explain your answer.

4 Exam-style question
The scatter graph shows information
aboutthe height and arm length of each of D E
8students in Year 11.

120

100
Arm length (cm)

90

80 8 A farm has a herd of Holstein-Friesian dairy


cows and a single Jersey cow.
70 The Holstein-Friesian cows produce an
average of 22 litres of milk each per day, and
60 the Jersey cow produces an average of 7 litres
120 130 140 150 160 170 180 of milk per day.
Height (cm)
a The total number of litres of milk
a What type of correlation does this produced every day is represented by y.
scatter graph show? (1 mark) The number of Holstein-Friesian dairy
Another student in Year 11 has a height of cows is represented by x.
148cm. Write an equation to show the information
b Estimate the arm length of this above.
student. (2 marks) b Sketch a graph to show your equation.
November 2012, Q2, 1MA0/1H
9 A road ascends from sea level at an angle of
5 Jan, Rowena and Atiq share 420 in the ratio 36 for 2.2 miles.
4:3:5. How much higher is the top of the road than
the bottom?
a How much does each person get?
Give your answer to 1 d.p.
Rowena shares her amount equally with Flo.
b Write the new ratio Jan:Rowena:Flo:Atiq 10 Work out the equation of a line perpendicular
using integers. to y = 4x 5, and passing through the point
(8, 0).
6 R Daniel and Maria are using these linear
equations to draw graphs.
y = x 2 y = 8x y = 2x + 1
y = 3x 3 y = 4x
The first line Daniel draws is parallel to
y = 2x 6. The first line Maria draws passes
through the origin.
a Which equation did Daniel use?
b Can you tell which equation Maria used?
Explain.

R = Reasoning P = Problem-solving 6 Problem-solving 57


7 AREA AND VOLUME
3 Calculate the area and perimeter of this
7.1 Perimeter and area isosceles trapezium.
16 cm
1 a Work out the area and perimeter.
5 cm 4 cm

10 cm
20.5 m
4 Exam-style question
7.5 m
Sheila has a 8m
15 m
rectangular garden 1m
b Work out the area. with two triangular
flower beds of
6 cm
equal size.
3 cm 10 m 1.5 m 1.5 m 1.5 m
3.8 cm She wants to cover
the shaded area
10 cm 2 cm
3.8 cm with gravel.
1m
The gravel is sold in
3 cm
bags.
c Work out the shaded area. There is enough gravel in one bag to cover
6m2 of garden.
75 mm
Each bag of gravel costs 12.
Work out the least cost of putting gravel on
Sheilas garden. (4 marks)
120 mm
75 mm
5 P Brian wants to paint this wall.
120 mm

2 Calculate the areas of these trapezia.


Round your answers to the nearest whole 4m
number where appropriate. 2.5 m

a 5 cm b 2.5 m
3.5 m
4 cm One tin of paint covers 8m2.
8m
11 m Brian needs to paint two coats.
8 cm
How many tins of paint does he need to buy?
6 The area of this trapezium is 66cm2.
c d 1 cm
5 cm
1.5 mm 12 cm
h
10 mm
19 cm 17 cm

a Substitute the values of a, b and A into


the formula A=_12(a+b)h
5.5 mm
b Simplify to get an equation.
Q2a hint Use the formula: area = _12(a + b)h = h
c Solve to find h.

58 R = Reasoning P = Problem-solving 7.1 Perimeter and area


7 P A trapezium has area 78mm2, and parallel 4 Calculate these areas.
sides 12.5mm and 7mm. Work out its height. a b
8 R Find the missing lengths. Example
a 7 cm 1.1 m 2.5 m
84 cm
a Area
49 cm2 10 cm 25 cm
Area = cm2 Area = m2
b b c 23 mm d 1.5 m
Area
11 mm
313.5 mm2
0.5 m
35.5 mm 65 mm
55 mm
9 P All four corners of a rectangular piece
of paper are folded in to make this shape. Area = cm2 Area = mm2
The shape is divided by a vertical line of
symmetry to give two identical trapezia. 5 A rectangular field has an area of
4.8hectares.
The length of one of the sides is 240m.
What is the length of the other dimension?

6 P A biologist counts 5 beetles in a 25cm by


25cm square of a field.
15 cm The whole field has area 30ha.
15 cm
Estimate the number of beetles in the field.
The area of this shape is 187.5cm2.
Work out the length of the piece of paper. Q6 hint Work out the area of the square in
m2. How many squares will fit in the field?
7.2 Units and accuracy
7 Bags of nuts are sold in 500g bags.
A factory packs the bags with a 5% error
1 a Explain why these two squares have the interval.
same area.
a Work out the largest and smallest possible
masses of the bags of nuts.
1m 100 cm
b Write the possible masses as an inequality.
1m 8 A factory makes pencils with a length of
100 cm
200mm.
b Work out the area of each square.
There is a 2.5% error interval on this length.
c Copy and complete.
Work out the possible lengths of the pencils
1m2 = cm2
in centimetres.
2 R a Sketch a square with side length 1m
and a square with side length 1000mm. 9 R a Each measurement has been rounded
to the accuracy given. Write its smallest
b Copy and complete.
possible value.
1m2 = mm2
i 45m (to the nearest metre)
c How do you convert from mm2 to m2? ii 209mm (to the nearest mm)
3 Convert b Each measurement has been rounded to
a 500cm2 to mm2 b 4.8m2 to cm2 1d.p. Write its smallest possible value.
c 8500mm to cm d 19.7cm2 to m2
2 2 i 8.6g
e 2.34m2 to mm2 f 4025mm2 to m2 ii 0.4cm
g 7250mm to cm h 4250000mm2 to m2
2 2

R = Reasoning P = Problem-solving 7.2 Units and accuracy 59


10 Each measurement has been rounded to the
accuracy given. 7.3 Prisms
Write an inequality to show its smallest
and largest possible values. Use x for the 1 R a Make a sketch 10 cm
measurement. of this triangular
prism. 4 cm 5 cm
a 25kg (to the nearest kg)
b 32.8cm (to 1 d.p.) b Work out the area
of the triangular 3 cm
c 8.4km (to 1 d.p.)
face. How many faces
d 0.76m (to 2 d.p.)
of the triangular prism also have this area?
11 Write c Work out the area of the three rectangular
i the upper bound faces.
ii the lower bound of each measurement. d Work out the total surface area of the
triangular prism.
a 4m (to the nearest m)
b 2.6g (to the nearest tenth of a g) 2 Calculate the surface area of
c 12.3cm (to 1 d.p.) a a cuboid 2.5cm 10cm 4.5cm
d 7.25 litres (to 2 d.p.) e 5000km (to 1 s.f.) b this triangular prism.
f 450mm (to 2 s.f.) g 2.80kg (to 3 s.f.)

12 R A rectangle measures Example 13 cm


5 cm
25cm by 34cm to the 20 cm
nearest cm. 12 cm

a Work out the upper and 3 a Explain why the shapes in Q2 are prisms.
lower bounds for the b Work out the perimeter of the triangular
length and width. face of the solid in Q2b.
b Calculate the upper and lower bounds for c Multiply the perimeter of the cross-section
the area of the rectangle. by the length of the solid and add the area
of the triangular faces. What do you notice?
13 R The dimensions of this isosceles
trapezium are given to 1 d.p. 4 Work out the volume of each prism.
5.9 cm a 8 cm b

3 cm 2 cm
3.0 cm
3 cm 9 cm
6.5 cm
2.5 cm
c 40 mm d
80 mm 10 m
Work out the upper and lower bounds for its
perimeter.
3m 2m
100 mm
14 R A triangle has area 54.4cm2to 1 d.p. 55 mm 25 m
Its base length is 9.95cm (to 2 d.p.). 8.5 m
90 mm
a Write the upper and lower bounds for the
area and the base length. 5 R This triangular
prism has volume 11.2 cm
b Work out
373.5cm3. 7.5 cm
lower bound for area 2
i _________________________

a Work out the
upper bound for base length value of b. 12 cm
b
upper bound for area 2 b Sketch its net
ii ________________________

and work out its surface area to 1 d.p.
lower bound for base length
c What is the lower bound for the height of Q5a hint Write and solve an equation:
the triangle to 2 d.p.? volume = 373.5 = _12 b

60 R = Reasoning P = Problem-solving 7.2 Units and accuracy


6 Exam-style question 12 Mike owns a factory which makes fence posts.
He needs to treat the fence posts with wood
Maggie has a petrol
preservative.
tank.
The tank is in the He will treat all the surface area of each post.
shape of a cuboid. 20 Each post is modelled as a cuboid 2m in
The depth of the 60 length with base dimensions 40mm by 40mm.
petrol in the tank 50 a Work out the surface area of each post.
is 12cm. b Mike will treat 500 posts. He needs 30ml
The tank is turned on its end, so it is of preservative for every square metre of
standing on the shaded face. wood.
Work out the depth, in cm, of the petrol in Estimate the amount of preservative he
the tank now. (3 marks) needs to the nearest litre.

13 Show that the surface area of this prism is


7 R a Sketch a cube with side length 1m and 12x(7x 1).
a cube with side length 100cm. 6x 2 1
b Copy and complete. 1m3 = cm3
c How do you convert from cm3 to m3? 5x
4x
8 R a Work out the volumes of a cube with
side length 1m and a cube with side
length 1000mm. 3x
b How do you convert from m3 to mm3?
14 R The dimensions of this triangular prism
9 Convert are measured to 1d.p.
a 8.2m3 into cm3
b 4800mm3 into cm3 15.9 cm
c 1650000cm3 into m3
d 12.55cm3 into mm3 9.2 cm
e 87500cm3 into litres 18.0 cm
f 6.05 litres into cm3 Calculate the upper and lower bounds for the
g 0.25m3 into litres volume of the triangular prism.
h 98cm3 into ml

10 P Sam wants to paint the walls and ceiling


of his loft conversion. He will paint two coats
7.4 Circles
on all the walls and the ceiling. What is the
total area Sam will paint to the nearest m2? 1 R The table gives the diameter and
circumference of some circles.
3.9 m
5m Diameter Circumference
5cm 15.7cm
2.5 m
10m 31.4m
3.3 m
42mm 131.9mm
circumference
3.8 m a Work out the ratio ____________

for each
diameter
one. What do you notice?
11 P / R A cuboid has 8 edges of the same circumference
length and 4 shorter edges which are 3.5cm b The ratio ____________


of a circle is
diameter
each. represented by the Greek letter p (pi).
Its volume is 143.5cm3. Find the p key on your calculator.
What is the length of the longer edge Write the value of p to 8 decimal places
(to 1 d.p.)? (8d.p.).

R = Reasoning P = Problem-solving 7.4 Circles 61


2 Work out the circumference of each circle. 6 P The diameter of a wheelchair wheel
Give your answers to 1 d.p. and the units of measures 63cm.
measurement. How many complete revolutions does the
a b wheel make in an 800m wheelchair race?
8 cm 5.2 m
7 R The areas and circumferences of these
circles are given in terms of p.
Match each circle to its area and circumference.
c d a b c d
47 mm 8.6 cm
4 cm 9 cm 15 cm 20 cm

3 Find the area of each circle.


a b
81p cm2 225p cm2 16p cm2 400p cm2
7 cm 9.3 m

30p cm 40p cm 18p cm 8p cm

c d
5m 8 R a Work out the area and circumference of
11.9 cm
a circle with radius 7cm
i in terms of p ii to 2 s.f.
b Which values for the area and
4 Exam-style question circumference are the most accurate?

Tariq designs a pattern for a park square in 9 The circumference of a circle is 215cm.
the city centre. a Substitute the value for C into the formula
He draws this design to be made from dark C = pd
grey and light grey gravel. b Solve the equation to find the diameter to
Each bag of gravel covers an area of 8m2. 1 d.p.
10 Find the radius of a circle with circumference
20m. Give your answer to 1 d.p.
50 m
11 a Find the radius of a circle Example
with area 484cm2.
How many bags of dark grey and light grey Give your answer to the
gravel are needed to complete the design? nearest cm.
You must show your working. (5 marks) b Find the diameter of a
circle with area 952cm2.
5 Georgie owns a factory which produces Give your answer to the nearest mm.
aluminium bars for climbing gear.
The diagram shows the cross-section of one 12 a Copy and complete to make r the subject
of the bars. of the formula for the area of a circle.
A = pr2
A
__
10 mm =r 2


___
13 mm A
__
=

b Use your formula for r from part a to work


a Work out the mean distance across the bar. out the radii of these circles to 2 d.p.
b By modelling the cross-section as a circle, i circle 1 with area 120cm2
calculate an estimate for its area to the ii circle 2 with area 750mm2
nearest square millimetre. iii circle 3 with area 6.7m2

62 R = Reasoning P = Problem-solving 7.4 Circles


13 R Paul sells cans of drink. Each can is in 5 P Two quarter circles are cut from this
the shape of a cylinder. The diameter of the rectangular piece of card.
cross-section of each can is 7cm. Work out the shaded area.
Paul packs the cans in boxes 40 cm
like this. What percentage of
the area of the base of each 20 cm
box do the cans occupy?

Q13 hint How many circles fit across and


along the rectangle of cardboard? 6 The area of this quarter circle is
33.2cm2 to 1d.p.
Find the perimeter of the
quarter circle to 1d.p.
7.5 Sectors of circles
Q6 hint Calculate the area of the whole
1 Work out the area to 1 d.p. of circle, and then find its radius.
a the semicircle b the quarter circle.
7 Exam-style question
A Example
Diagram
20 cm NOT
6 cm 10 cm accurately
45 drawn
O B
Q1 hint Find the area
of the whole circle first. OAB is a sector of a circle, centre O.
The radius of the circle is 20cm.
The angle of the sector is 45.
2 Work out the perimeter of each semicircle Work out the arc length and hence the
i in terms of p ii to 1 d.p. perimeter of sector OAB.
a b Give your answer correct to 3 significant
figures. (2 marks)

15 cm
2.5 cm
8 Work out the area of the sector in Q7.
Give your answer to 3 s.f.

9 P a Work out the arc


3 Work out the perimeter of this quarter circle
length and area of this
a in terms of p sector to 3 s.f. 155
b to 1 d.p. b The radius of the 14 cm
circle was measured
4.5 m to the nearest cm and the angle was
measured to the nearest degree.
4 P A racecourse comprises a rectangle and Work out the upper and lower bounds for
2identical semicircles. the area of the sector.

10 P The area of this sector is 39cm2.


60 m

100 m
x
Work out to the nearest metre
9 cm
a the area of the racecourse
b the perimeter of the racecourse. Find the angle of the sector, to the nearest
degree.

R = Reasoning P = Problem-solving 7.5 Sectors of circles 63


11 P Find the angle of this sector. 2 Work out the volume of each cylinder.
Giveyour answers to 3 s.f.
150 mm
a 4.5 cm
b

51 mm
x
8 cm
80 mm
94 mm
12 P This sector has area 24m2.
Find the radius.
Give your answer to a suitable degree of c 6m
accuracy.
25 cm

12 3 After an oil leak at sea, an oil slick is formed.


r The shape of the slick can be modelled as a
cylinder.
13 P Angle AOB is 135. The diameter across the slick is 150m and
5p the mean thickness of the slick is 0.1mm.
The arc length AB is ___ cm.
6 Estimate the volume of the oil slick.
Find the area of sector AB in terms of p.
Give your answer in m3 to 2 d.p.

4 A cylinder has diameter 6.2cm and volume


O 317cm3. Work out its height to 1 d.p.
135 6.2 cm
A B

h
14 P Calculate the area of the shaded region.
Give your answer in terms of p.

5 Calculate the total surface Example


area of each cylinder in Q2.
Write each answer to 3 s.f.
12 cm

7.6 C
 ylinders and 6 P A cylinder has total surface area 33cm2
and diameter 10mm.
spheres Work out its height, to the nearest millimetre.

1 R The volume of a cylinder of radius r and Q6 hint Substitute the values into the
height h is V = pr2h surface area formula and solve.

r 7 P Water is poured into a cylindrical vase with


diameter 10cm. The same amount of water
h is poured into another cylindrical vase with
different dimensions. The water rises to one
and a half times the height it reached in the
first vase.
Explain why this is a formula for the volume Work out the diameter of the second vase
of a prism. incm to 1 d.p.

64 R = Reasoning P = Problem-solving 7.5 Sectors of circles


8 Calculate the surface area and volume of
each sphere. Give your answers in terms of p. 7.7 Pyramids and cones
a b
1 Here is a square-based pyramid.
12 mm
8 cm

10 cm

9 A factory makes hollow metal hemispheres as 7 cm


garden ornaments. The factory produces the
7 cm
hemispheres with external diameter 20cm
and a thickness of 3mm. a Work out the area of each triangular face.
Work out the volume of metal used for each b Calculate the total surface area of the
hemisphere, to the nearest cubic millimetre. pyramid.
10 Exam-style question 2 This pyramid has a square base of side 5cm,
and vertical height 9cm.
A manufacturing company makes plastic
containers for childrens toys. Calculate its volume.
Each container comprises a cylinder with
identical hemispheres at each end.
The length of the cylinder is 2.5cm. 9 cm
The diameter of the cylinder is 1.75cm.
5 cm
1.75 cm 5 cm

2.5 cm 3 Exam-style question


This solid is made from
a triangular-based pyramid
and a triangular prism.
a Calculate the total volume of the container. The triangular prism has
Give your answer correct to 3 significant a right-angled triangular
figures. (3 marks) cross-section and a length
b Calculate the surface area of the container. of 10m.
5m 10 m
Give your answer correct to 3 significant The height and base of 12 m
figures. (3 marks) the triangle are 12m and
5m respectively.
11 P A cuboid-shaped block of metal has The ratio of the volume of the pyramid to
dimensions 20cm by 10cm by 4cm. the volume of the prism is 1:4
The block is melted down and made into Find
spherical ball bearings, each with a radius a the volume of the prism
of5mm. b the height of the pyramid. (6 marks)
How many of these ball bearings can be
made from the block? 4 A cone has base radius 9cm
12 P What is the radius of a sphere with volume and height 12cm.
6000cm3? Calculate its volume
a in terms of p
13 P A cylinder has a volume of 450cm3 b to 3 s.f. 12 cm
measured to the nearest 10cm3, and a radius
of 5cm measured to the nearest cm.
What are the upper and lower bounds of its
height? 9 cm

R = Reasoning P = Problem-solving 7.7 Pyramids and cones 65


5 A cone has base radius 11 This 3D solid is made from x cm
70mm and slant height a cylinder and a cone.
250mm. 250 mm Write an expression in
Calculate, in terms of p terms of x and p for
a the area of its base a the surface area of the 12 cm
b its curved surface 70 mm cylinder
area b the curved surface area x cm
c its total surface area. of the cone
c the total surface area of
6 P Work out the capacity of this disposable the solid.
cup in ml.
Give your answer to 3 s.f. 12 Calculate the volume and surface area of this
3D solid. Give your answers to 3 s.f.
4 cm

45 mm
10 cm
180 mm

7 P Work out the total surface area of a cone


with radius 3.6cm and volume 129cm3.
Give your answer to 3 s.f.

8 P A solid cone is made from 144000mm3 of


plastic. 7 Problem-solving
The diameter of the cone is 50mm.
a What is the height of the cone? Solve problems using these Example
b What is the surface area of the cone? strategies where appropriate:
Give your answers to 3 s.f.
Use pictures or lists
9 P A cone and a sphere have identical total Use smaller numbers
surface areas. The height of the cone is 24cm Use bar models
and the slant height is 26cm. Use x for the unknown
Work out the radius of the sphere to the Use a flow diagram.
nearest millimetre.
1 R Gillian sells items at a craft fair.
10 The top half of this cone is cut off to leave a The prices of the items sold at her last craft
3D solid called a frustum. fair were
3.50, 12.50, 25, 8, 5, 4.50, 5, 16, 5,
2.50, 5, 12.50, 2.50, 5, 3.50
6 cm
a Find the mean, median, mode and range
of the prices of the items sold.
6 cm b Why might Gillian want to know the mode?
10 cm c What might Gillian say about the range?

2 The first floor of an office occupies 1200m2.


Work out the volume of the frustum, in terms
of p. _38of the space is taken up by work stations
and _13of the space is taken up by the canteen.
Q10 hint Volume of frustum The rest is taken up by conference rooms.
= volume of whole cone volume of top cone
What is the floor area of the conference
rooms?

66 R = Reasoning P = Problem-solving 7.7 Pyramids and cones


3 R A local school has been given an area of a 8 Exam-style question
memorial park to create a mosaic.
The shape and measurements of the area are The cabins on a ship need repainting.
shown below. The walls overlooking the sea will be
painted blue. There are enough cans of blue
60 cm paint to cover 50m2.
100 cm 3.8 m
70 cm 60 cm 60 cm
130 cm
85 cm 2m
80 cm cabinet
150 cm
120 cm
a What is the perimeter of the shape for the The shaded area is the part of the wall to be
mosaic? painted. If all the cabins are the same, how
b What is the area of the mosaic? many complete cabin walls can be painted
c How did you work out the area of the with the paint they already have?
mosaic? Show all your working.
Round all answers to 2 d.p. (5 marks)
4 R A farmer rents two rectangular fields.
Onefield measures 350m 620m and the 9 An open-air stage is being built in a park.
other measures 430m 860m. Atrack needs to be placed along the arc of
How many hectares of field does the farmer the sector for the scenery.
rent?
back
Q4 hint 1ha = 100m 100m

5 A company makes measuring wheels where 140


4.5 m
the circumference of the wheel is used to
front
measure distance.
What would the radius of the wheel need to a What length does the track need to be?
be in cm for each turn to measure Give your answer to 1 d.p.
a 1 metre b What is the area of the stage?
b 1.5 metres Give your answer to 2 d.p.
c 5 metres? 10 R Philip and Craig are building model
Give your answers to 1 d.p. castles for a project.
Philip uses cylinders for his towers and needs
6 R The sum of the interior angles of a
to make cones for the turrets.
polygon is1080.
Craig uses prisms with square-faced ends
What is the name of the polygon?
and wants to make square-based pyramids
Explain how you know.
for the turrets.
7 R A class have been asked to design a
Craig Philip
container to hold exactly 8 litres of liquid. 8 cm 8 cm
Knowing that 1litre = 1000cm3, what possible
dimensions could the container have if it was
a a cube
24 cm 24 cm
b a cuboid
c a triangular prism?

6 cm 6 cm

Ifboth types of tower are the same height


and width/diameter, which tower has the
larger surface area? Show your working.

R = Reasoning P = Problem-solving 7 Problem-solving 67


8T
 RANSFORMATIONS AND
CONSTRUCTIONS
5 Exam-style question
8.1 3D solids
Here is a solid prism.
1 On squared paper, draw and Example
label the plan, front elevation 5 cm
and side elevation of these
solids. 3 cm
a 3 cm 8 cm
b c
4 cm 9 cm

2 cm
13 cm
2 R Sketch the solids represented by these
plans and elevations. 4 cm
a Plan Front Side On a centimetre-square grid, draw an
accurate plan view of the solid prism from
the direction of the arrow. (2 marks)

b Plan Front Side

8.2 Reflection and


rotation
c Plan Front Side

1 Draw a coordinate grid from 8 to +8 on both


axes.
3 P Here is the side elevation of a 3D solid. a Draw a rectangle Q with coordinates
A(3, 2), B(1, 2), C(1, 5) and D(3, 5).
Sketch three possible 3D
solids it could belong to. b Reflect rectangle Q in the y-axis.
Label the image R.
4 P / R Here is a cube. c Reflect rectangle R in y = 1.
a Calculate the surface 3 cm Label the image S.
area of the cube. d Reflect rectangle S in the y-axis.
b The cube is cut in 3 cm Label the image T.
half along the red 3 cm
e Describe the single reflection that maps
plane. Sketch the rectangle T onto rectangle Q.
plan, front elevation
and side elevation Q1 hint Use tracing paper to help.
of each of the new
3D solids. 2 R Draw a coordinate grid from 6 to +6 on
c Calculate the surface area of both axes.
each of the new solids. a Draw triangle A with coordinates (0, 2),
d Repeat parts b and c (2, 2) and (2, 4).
for the cube cut along b Reflect triangle A in the line y = x.
this red plane. c Reflect triangle A in the line x = 2.

68 R = Reasoning P = Problem-solving 8.1 3D solids


3 Describe the reflection that maps ii 180 about (0, 0)
a P onto Q b P onto R iii 90 clockwise about (3, 1)
c P onto S d P onto T. iv 180 about (1, 4)
v 90 anticlockwise about (1, 4)
y
vi 180 about (1, 0).
6
Label your results i, ii etc.
5
R4 P T 7 Describe the rotation that Example
3 takes each shape to its image.
2 y
1 10
8
26 25 24 23 22 21O 1 2 3 4 5 6 x
21 6
22 D 4
S 2 A
23
24 Q
28 26 24 220 2 4 6 8 x
25 22
C
B 24
26
26

4 Describe the reflection that maps


8 P Draw a coordinate grid from 5 to +5 on
a A to B b A to C.
both axes.
y a Plot the points (2, 1), (4, 2) and (1, 3).
7 Join them and label the shape A.
6 b Reflect shape A in the x-axis.
5 Label the image B.
C
4 c Reflect shape B in the y-axis.
3 A B Label the image C.
2 d Describe the transformation that takes
1 shape A to shape C.
0
0 1 2 3 4 5 6 7 8 9 10 x 9 R The rotation of a square can always be
described as a reflection.
5 Draw a coordinate grid from 5 to +5 on both Decide whether this statement is sometimes
axes. true, always true or never true.
a Draw shape A with coordinates (2, 1),
(4, 1) and (3, 3). 10 Exam-style question
b Reflect shape A in the line y = x. y
Label the image B. 5
c Reflect shape A in the line y = x. 4
Label the image C. 3
d Reflect shape C in the x-axis. 2 A
Label the image D. 1
B
e Describe the reflection that maps shape D
25 24 23 22 21O 1 2 3 4 5 x
to shape B. 21
22
6 R Draw a coordinate grid from 8 to +5 on
23
both axes.
a Draw shape A with coordinates (2, 1), (4, 1), Describe fully the single transformation
(4, 4) and (2, 4). that maps shape A onto shape B. (3 marks)
b Rotate triangle A
i 90 anticlockwise about (2, 1)

R = Reasoning P = Problem-solving 8.2 Reflection and rotation 69


d Shape A is enlarged by scale factor 4 to
8.3 Enlargement make shape D.
Work out the area of shape D.
1 Copy the diagram, drawing the x-axis from e Copy and complete this table.
2 to 8 and the y-axis from 2 to 5.
Scale Area of enlarged shape
_____________________
y Shape

factor Area of shape A
5
B 2
4
3
C 3
2 D 4
1
4 Copy these diagrams.
0
0 1 2 3 4 5 x Enlarge each shape by the scale factor given.
a scale factor _12
Enlarge the rectangle by scale factor 2, with
these centres of enlargement.
a (1, 4) b (1, 1) c (3, 2)
2 Shape A has been enlarged to give shape B.
y
6 b scale factor _13
A 4
2

26 24 22 O 2 4 6 8 10 x
22 B
24 5 a Copy and enlarge each shape by the
given scale factor about the centre of
26
enlargement shown.
a What is the scale factor of the i scale factor _13
enlargement?
b Copy the diagram.
Join corresponding vertices on the object
and the image with straight lines.
Extend the lines until they meet at the
centre of enlargement. ii scale factor _12
c Write the coordinates of the centre of
enlargement.
d Copy and complete to describe the
enlargement from A to B.
Enlargement by scale factor ____ ,
centre ( ____ , ____ ). iii scale factor _13

3 P Draw a triangle A, with base 4cm and


height 3cm.
a Work out the area of the triangle.
b Shape A is enlarged by scale factor 2 to
make shape B.
Work out the area of shape B.
c Shape A is enlarged by scale factor 3 to b R When a shape is enlarged by scale
make shape C. factor _13, is its area enlarged by scale
Work out the area of shape C.
factor (_13) ? Explain.
2

70 R = Reasoning P = Problem-solving 8.3 Enlargement


6 P Describe the enlargement that maps 8 P Describe fully the transformation that
shape A onto shape P. maps shape S onto shape T.
a y y
5 10
4 8
P
3 6
S
2 4
1 2

25 24 23 22 21O 1 2 3 4 5 x 210 28 26 24 22O 2 4 6 8 10 x


21 22
22 A 24
23 T 26
24 28
25 210
26

Q8 hint Draw lines from corresponding


b y
vertices to find the centre.
5
4
9 India said, An enlargement does not change
3 the orientation of the shape.
2 Is India correct?
1 Explain your answer.
25 24 23 22 21O 1 2 3 4 5 x 10 Exam-style question
21
22 y
23 10
A
24 8
A
25 6
P
26 4
2
7 P Draw a coordinate grid from Example
210 28 26 24 22 O 2 4 6 8 10 x
12 to +12 on both axes. 22
B
Join the points (6, 3), (8, 3) and 24
(6, 6) to make a triangle. 26
Enlarge the triangle 28
a by scale factor 2, 210
centre of enlargement (5, 2)
b by scale factor 3, Describe fully the single transformation
centre of enlargement (5, 4) that maps shape A onto shape B. (3 marks
c by scale factor 1,
centre of enlargement (6, 3).
Exam hint.
For 3 marks, give 3 pieces of information
about the transformation.

R = Reasoning P = Problem-solving 8.3 Enlargement 71


4 Draw a coordinate grid from 6 to +6 on both
8.4 Translations and axes.
combinations of a Plot a triangle with vertices at (2, 3), (0, 1)
and (1, 4). Label the triangle P.
transformations b i Translate triangle P by vector ( ).
2
7
1 Copy the diagram. Translate Example ii Translate this new triangle by vector

( 3). Label the image Q.


shape A by the vectors 4
a ( )to B b ( 3)to C
1 2
3 c Describe the translation of triangle P to
triangle Q, using a single vector.
( 0)to E
c ( )to D d
0 2
5 a R A shape is translated by vector ( )
7 2
e ( )to F
5 3
followed by a translation by vector ( ).
3 3
1
y
What is the resultant vector?
b The resultant of two vectors is ( ).
10 3
8 4
The first vector is ( ).
1
6 2
A What is the second vector?
4
2 6 Copy this diagram and shape A only on a
coordinate grid from 6 to +6.
24 22 O 2 4 6 8 10 x y
22
10
8
2 Describe these translations using column P 6
vectors.
4
a A to B b C to E
c D to C d F to B 2
e B to F
28 26 24 22 O 2 4 6 x
y 22 A
V
10 24
8 26
F B
6
A
a Translate shape A by vector ( ) .
4 7
3
2 C Label the image B.
b Reflect shape B in the line x = 2.
28 26 24 22 O 2 4 6 8 x Label the image C.
22
24 E 7 Copy the diagram from Q6 and shape P only.
a Translate shape P by vector ( ) .
D 6
26 2
Label the image Q.
b Reflect shape Q in the line y = 1.
3 R A shape is translated by vector ( ) .
a Label the image R.
c Translate shape R by vector ( ).
6
b
2
What vector would translate the shape back Label the image S.
to its original position? d Describe the reflection that maps shape P
Explain your answer. onto shape S.

72 R = Reasoning P = Problem-solving 8.4 Translations and combinations of transformations


8 Copy the diagram from Q6 and triangle V only. 11 Adam says, A shape and its transformed
a Rotate triangle V through 180 about the image always have the same area.
point (1, 0). Label the image W. Do you agree with this statement?
b Reflect triangle W in the line y = x. If not, give a counter example and explain
Label the image X. your answer.
c Translate triangle X by vector ( ).
9
0
Label the image Y.
d Describe the single transformation that 8.5 Bearings and
maps triangle V onto triangle Y.
scaledrawings
9 P / R A tessellation is made by transforming
shape A. 1 P During a search and rescue mission the
positions of a frigate, lifeboat and helicopter
D are marked in relation to the man overboard.
F C
E B Frigate
A
Man
overboard
Copy the tessellation onto a coordinate grid
with the vertices of A at (0, 0), (3, 0), (3, 1), Helicopter
(1, 1) (1, 2) and (0, 2).
Describe the transformation that would move Lifeboat
shape
a A to B b B to C c C to D The real-life distance from the frigate to the
d A to E e E to F. helicopter is 16km.
f What transformation of shape A would a What scale has been used on the map?
make a shape with 4 times the area? b From the map, estimate the distance
from
10 Exam-style question
i the lifeboat to the man overboard
y ii the helicopter to the man overboard.
4 c The helicopter flies at a speed of
3 140km/hour.
2 A
How long will it take it to fly from its
1 current position to the man overboard?
Giveyour answer in minutes.
25 24 23 22 21O 1 2 3 4 5 x
21
22
B 2 Adrian is using a map with a Example
23
scale of 1:250000.
24 He measures these distances
25
on the map.
a 5cm b 10cm
a Describe fully the single transformation c 3cm d 0.5cm
that maps triangle A onto triangle B. What are the distances in real life?
 (2 marks) Write your answers in kilometres.
b Reflect triangle B in the y-axis and
label the image C. Then translate 3 The scale on a map is 1:50000.
shape C by vector ( )and label
4 What is the distance on the map, in cm,
0
the image D. (3 marks) for a real distance of
c Describe the single transformation a 30km b 5km c 13km?
that maps triangle B onto
triangle D. (2 marks)

R = Reasoning P = Problem-solving 8.5 Bearings and scale drawings 73


4 The scale on a map is 1:10000. 9 P A man walks 7km on a bearing of 120.
a On the map, the distance between the He then turns and walks 8km on a bearing
post office and the library is 3.5cm. of320.
Work out the real distance between them. a Use a scale of 2cm to 1km to draw an
Give your answer in metres. accurate scale drawing of the walk.
b The real distance between the two b How far is the man from his starting point?
churches is 1.5km. Work out the distance c What is the bearing he should walk on to
between the churches on the map. Give return to his starting point?
your answer in cm.
10 P The bearing from Manchester Airport to
5 a The scale of the map is 1:1000000. Amsterdam Airport is 105.
Calculate the distance in km between Calculate the bearing of Manchester Airport
i St Peter Port and St Helier from Amsterdam Airport.
ii St Helier and Carteret.
N
b Which town is 57km from Cherbourg?
Alderney
St Annes

Cherbourg 105
Manchester

Vale
Herm FRANCE Amsterdam
St Peter
Port
Guernsey
Sark
Carteret 11 P a The bearing of B from A is 125.
Workout the bearing of A from B.
Jersey b The bearing of C from D is 322.
Jersey Zoo
Rozel Work out the bearing of D from C.
Captive Island
St Aubin St Helier
12 Exam-style question
The diagram shows the N
6 The diagram shows the relative positions of
positions of two boats,
towns A, B and C.
B and C.
C Boat T is on a bearing
of 190 from boat B. B 150

Boat T is on a bearing
N
35 51 of 315 from boat C.
A 6 km B Draw an accurate diagram
to show the position of boat T.
a Make an accurate scale drawing using a
Mark the position of boat T C
scale of 1:50000.
with a cross ().
b Work out the real distances AC and CB.
Label it T. (3 marks)
7 P The distance between Heathrow and
Gatwick is 35km.
The bearing of Gatwick from Heathrow is155.
Make an accurate scale map of the locations 8.6 Constructions 1
of the two airports, using a scale of 1cm to
5km. 1 Construct an accurate drawing Example
of this triangle.
8 P A ship is 120km east of a port. The ship
R
then sails on a bearing of 310 for 90km. 8 cm
a Make an accurate scale drawing. 5 cm Q
Use a scale of 1cm to 20km.
11 cm
b What is the bearing of the port from the P
ship?

74 R = Reasoning P = Problem-solving 8.5 Bearings and scale drawings


2 Construct each triangle PQR. 10 Follow these instructions to draw the
a PQ = 4cm, QR = 7cm, PR = 5cm perpendicular from point L to the line MN.
b PQ = 9cm, QR = 6cm, PR = 4cm a Open your compasses and draw an arc
c PQ = 9.5cm, QR = 6.5cm, PR = 4.5cm with centre L. Label the two points where
it intersects with the line MN as Q and R.
Q2 hint Sketch each diagram first and label
the lengths. L

3 Construct an equilateral triangle with sides


7.5cm. M Q R N
Check the angles using a protractor.
b From each point Q and R, keeping
4 R Explain why it is impossible to construct your compasses the same, draw an arc
atriangle with sides 3cm, 5.5cm and 12cm. underneath the line so they cross.
5 Construct an accurate scale c Join this point with point L.
drawing of this flower bed. 7m 11 P A walker is in the middle of a field
Use a scale of 1:100. 5m
and wants to walk the shortest possible
distanceto the marked footpath.
4m
The scale is 1:1000
6 The diagram shows the profile
of a triangular shed. Walker
15 m 15 m
Using a scale of 1cm to 2m,
construct an accurate scale
drawing of this profile.
10 m
7 P This chocolate box is in the shape of
triangular prism.

a Trace the diagram and construct the


shortest path for the walker to get to each
of the two straight parts of the footpath.
The end faces are equilateral triangles with
b Work out the difference in the distances.
sides of length 5cm.
The whole box has length 7cm. c The speed of the walker is 1.5m/s.
Construct an accurate net for the box. How long would it take to walk the shorter
distance?
8 a Draw a line segment PQ Example
10cm long. Construct the
perpendicular bisector of PQ. 8.7 Constructions 2
b Use a ruler and protractor
to check that it bisects your 1 For each angle Example
line at right angles. i trace the angle
c Mark any point R on your perpendicular ii construct the angle
bisector. bisector using a ruler
Measure its distance from P and from Q. and compasses
9 P Two schools A and B are 0.8km apart. iii check your two smaller
a Using a scale of 1:10000, draw an accurate angles using a protractor.
scale drawing of the schools. a b
b A pelican crossing is equidistant from the
two schools.
Construct a line to show where the pelican
crossing could be.

R = Reasoning P = Problem-solving 8.7 Constructions 2 75


2 P Use a ruler and compasses to construct 7 P a Use a ruler, protractor and compasses to
these angles. construct the triangle PQR.
a 90 b 135 Q

Q2a hint What angle will you get when you


5 cm
bisect a straight line?
45 R
3 P Use a ruler and compasses to construct 9 cm
P
these angles.
b Construct a line that is perpendicular to
a 22.5 b 112.5
PR and passes through Q. Label the point
where it meets PR as S.
4 P Use a ruler and compasses to construct a
210 angle. c Calculate the perimeter of triangle PQS.

8 P The net of a regular


5 P A rectangular piece of land is being split 4 cm
tetrahedron is made from
into two plots A and B.
equilateral triangles.
4m Construct the net using
a ruler and compasses.
135 9 a Draw any triangle ABC.
Plot B b Construct the bisector of each angle.
30 m c Label the point where the bisectors cross
Plot A
as point D.
d Construct the perpendicular from D to
each of the sides of the triangle.
40 m e Measure the distances from D to where
each perpendicular meets the sides.
a Make a scale drawing of the rectangular What do you notice?
plot using a scale of 1cm to 4m.
b Use a ruler and protractor to construct the 10 R a Draw a circle with centre O and radius
diagonal line. 6cm. Mark a point A on its circumference.
c Calculate the area of plot A. b Keep the compasses the same size as the
radius and draw an arc from point A.
6 P A piece of cloth in the shape of an Label the point where the arc cuts the
equilateral triangle (ABC) is cut into two circleB.
equal right-angled triangles to make two A
sails for a boat. Angle CBD is bisected and the B
lower part of the sail is dyed red.
O
C

8m 8m
c Keeping the compasses the same, repeat
from point B. Repeat until you have six
E
points on the circumference.
d Join alternate points on the circumference.
B D A
8m e Name the shape you have constructed.

a Construct a scale drawing. 11 P Draw a regular hexagon in a circle of


Use a scale of 1cm to 1m. radius 6cm.
b Measure the length of the line DE. Q11 hint What angle will you need to
c What percentage of sail BCD is dyed red? construct from the centre to two consecutive
points on the circumference?

76 R = Reasoning P = Problem-solving 8.7 Constructions 2


7 P This triangle is rotated B9
8.8 Loci anticlockwise about C.
a Copy the diagram.
1 A dog is tied to a post by a 6m length of rope. b The triangle is then
C
Sketch the locus of where the dog can roam. rotated 90 clockwise A9
about B9. Add this to
2 P Draw a line 10cm long. the diagram.
Draw the locus of all points which are 5cm c Draw the locus of
from the line. vertex A.
d Draw the locus of A B
3 P A rectangular bowling green is marked out vertex C.
to measure 20m by 3m.
8 P Two lighthouses are 100km Example
20 m apart.
3m Lighthouse A is on a bearing
of 100 from lighthouse B.
A fence is put up 1m outside the perimeter. Each lighthouse can be seen
a Draw a plan of the fenced area using a at a distance of 80km.
scale of 1cm to 2m. Draw an accurate scale drawing of the
b Construct the locus of the fence. lighthouses using a scale of 1cm to 20km.
Shade the region where both lighthouses can
Q3 hint Think carefully about what happens
be seen.
at the corners.
9 Exam-style question
4 R / P a Draw two points 8cm apart and Here is a scale drawing of a rectangular
label them A and B. garden ABCD.
b Construct the perpendicular bisector of B C
the line AB.
c Mark a point which is
i 5cm from A and 5cm from B
ii 7cm from A and 7cm from B.
d What do you notice about these points?

5 P A cinema is being built equidistant from


two towns P and Q. P and Q are 40km apart.
Using a scale of 1:1000000, construct the locus
of points where the cinema could be built.

6 A gardener wants to plant a row of bulbs in a


flowerbed equidistant from the edge AB and
the edge AD.
C B
A D
8m Scale: 1 cm represents 2 metres.
Jane wants to plant a tree in the garden
at least 5m from point C
D 10 m A
nearer to AB than to AD
a Copy the diagram using a scale of 1cm and less than 3m from DC.
to1m. On a copy of the diagram, shade the region
b Construct the locus of points where the where Jane can plant the tree. (4 marks)
bulbs will be planted. March 2013, Q15, 1MA0/1H

R = Reasoning P = Problem-solving 8.8 Loci 77


10 P A dog is tied to a rope at the corner of a 2 R Look at these designs.
barn which measures 10m by 7m. y
5
4
3
12 m
7m 2
1

10 m 25 24 23 22 21 O 1 2 3 4 5 x
21
Make an accurate scale drawing of this barn. 22
Use a scale of 1cm to 2m. 23
Accurately shade the region where the dog 24
can roam. 25
11 P ABCD is a rectangle with sides 7cm and
3cm. y
5
A 7 cm B
4
3 cm 3
2
D C
1
Copy the diagram. Shade the region that is
closer to AB than BC and more than 2cm 25 24 23 22 21 O 1 2 3 4 5 x
21
from A. 22
12 A graph x2 + y2 = 40000 shows the region 23
covered by a radio mast, where x and y are 24
inmetres. 25
a Draw a coordinate grid from 300 to +300
on both axes. Plot the graph.
Sue-Ellen is designing a patchwork quilt.
b What area does the radio mast cover? Shestarted with the shape on the first grid
Give your answer to the nearest thousand and then used three transformations to
square metres. create the pattern on the second grid.
Whatcould the three transformations be?
8 Problem-solving
3 An ancient fort has 9 sides.
Solve problems using these strategies where a What is the sum of the interior angles of
appropriate: the floor plan of the fort?
b If all 9 sides of the fort were of equal
Use pictures or lists length, what would each interior angle be?
Use smaller numbers
c The angles are actually one of three sizes.
Use bar models Three of the angles measure x, three
Use x for the unknown measure 2x, and the remaining three
Use a flow diagram. measure 3x.
What are the three sizes of angles?
1 Kelly needs to draw some
construction drawings
4 Michaela bought a car for 5700 plus
for a model she is making.
VAT at 20%. She will pay the cost of the car
Use the picture to draw
in12 equal monthly payments.
the plan, side elevation
What is the amount of each monthly
and front elevation
payment?
for Kellys model.

78 R = Reasoning P = Problem-solving 8.8 Loci


5 R Pete and Andy are hiking to the nearest 7 The manual for a photocopier needs a
village. Their map has a scale of 1:100000. diagram to show how to fix a paper jam.
The path they need is 9cm long on the map. Lever A needs to be rotated 120 clockwise
Pete says that it will only take a few hours to and lever B needs to be rotated 90
get to the village as it is only 9km away. anticlockwise.
Andy says it is too far to hike to as it is 90km
away.
a Who is correct? A B
Explain your answer.
b There is a sign for a village 12km away.
What distance would this be on the map?
c What would the scale be if 8cm represents The centre of rotation is marked on each
80km? lever (0).
Copy the drawings onto a grid and then use
6 This grid shows the position of a set of patio dotted lines to draw the images of where the
furniture. levers need to rotate to.
y
8 A farmer has invested in five wind turbines
10
to supply energy to his farm and village.
8 A Hedraws a grid to show where four of the
6 turbines are to be placed.
4
y
2 10
B
8
210 28 26 24 22O 2 4 6 8 10 x A
22 6
B
24 4
26 2
D
28
210 28 26 24 22O 2 4 6 8 10 x
210 22
C
24
The grid shows the new positions of the 26
furniture after it was blown by strong winds. 28
y 210
10
8 Use column vectors to describe the
6
translation that would move turbine
a A to B b B to C c C to D
4
A The farmer places turbine E by translating it
2
by vector ( ) from turbine D.
2
210 28 26 24 22 O 2 4 6 8 10 x 7
B 22 d Copy the grid and draw turbine E in its
24 correct position.
26
9 R There are some red counters and some
28
blue counters in a bag.
210
7 red counters are added and the number of
red and blue counters is now the same.
Fully describe the transformation of
Then 22 blue counters are added and the
a the table ratio of red:blue becomes 1:3.
b chair A How many red counters were originally in the
c chair B. bag?

R = Reasoning P = Problem-solving 8 Problem-solving 79


10 Exam-style question
In this question, use a ruler and a pair of compasses.
Leave in all your construction lines.
The scale drawing shows part of a school building and the
playground outside.
The head teacher wants to paint a mini-maze on the playground.
It needs to be
at least 4m from the fence KL
at least 3.6m from the tree T
nearer the maypole M than to the goal post G.
Construct and shade the region where the mini-maze can go.
K L

T
M

G
School

Scale: 1cm represents 2 metres. (5 marks)

9 EQUATIONS AND INEQUALITIES


3 Find the roots of these functions.
9.1 Solving quadratic a x2 3x b x2 36 c 100 z2
equations 1 4 a Use this graph to solve the equation
x2 2x 8 = 0
1 Find the solutions to these quadratic y
equations. 2
a 5x2 = 45
21028 26 24 22O 2 4 6 8 10 x
b 3x2 3 = 72 22
c 4x2 144 = 0 24
26
2 Solve y 5 x2 2 2x 2 8
Example 28
a x2 + 10x + 21 = 0 210
b a2 + 3a 18 = 0
c p2 8p + 12 = 0 b Factorise x2 2x 8 = 0 to show that you
d y2 y 20 = 0 get the same solutions.

80 R = Reasoning P = Problem-solving 8 Problem-solving


5 Solve 7 P A photo frame has a x
x2 x = 2
a x2 + 7x = 12 b space for a photo that 5 inches
c x2 = 7x 10 d x2 = 8x is 7 inches by 5 inches.
It has a wooden border x x
7 inches
Q5a hint Rearrange into the form that is the same width all
x2 + x + = 0 the way round. The area
of the wooden frame is
45 square inches. x
6 P Write any function that will give the roots
x = 2 and x = 5. a Write an equation for the area of the
wooden frame.
b Solve your equation to find x.
9.2 S
 olving quadratic 8 Exam-style question
equations 2 Solve, by factorising, the equation
6x2 19x + 15 = 0 (3 marks)
1 Write and solve an equation to find x.
x13 9 Solve, giving your solutions Example
in surd form
Area 5 28 m2 x a x2 + 8x + 5 = 0
b x2 + 5x + 2 = 0
c x2 + 4x 3 = 0
2 P A tile design is available in two sizes. d x2 + 6x 4 = 0
A small tile has dimensions l l. e 2x2 + 10x + 7 = 0
A large tile has dimensions 2l (l + 2). 10 Solve, giving your solutions to 2 decimal
The area of a large tile is 160cm2. places
What are the dimensions of the small tile? a x2 + 4x 6 = 0 b 2x2 + 3x 8 = 0
3 Copy and complete to factorise the c 3x 4x 11 = 0 d
2 5x2 + 10x 9 = 0
expression. 11 Solve 3x2 5x 8
2x2 + 5x 7 = (2x ______)(x ______) a by factorising
b by using the quadratic formula.
4 Factorise these expressions.
a 3x2 + 12x + 9 12 Exam-style question
b 2x2 + 7x + 3 Solve 3x2 + 9x 10 = 0
c 6x2 10x 4 Give your solutions correct to
d 5x2 + 13x 6 2 decimal places. (3 marks)
e 3x2 2x 8

5 Solve
9.3 Completing the square
a (3x 9)(x + 5) = 0
b (2y + 7)(3y 21) = 0 1 Write a quadratic expression for the area of
c (4t 5)(2t 5) = 0 the large square.
d (3d + 7)(3d 10) = 0 x 3

x
Q5a hint Either 3x 9 = 0 or x + 5 = 0

6 Solve 3
a 2x2 + 7x + 3 = 0
b 3x2 + 7x + 4 = 0
c 4x2 + 5x 21 = 0 Q1 hint Write an expression with two brackets.
Expand to get a quadratic expression.
d 4x2 16x + 15 = 0

R = Reasoning P = Problem-solving 9.3 Completing the square 81


2 Write these expressions in the form 10 Copy and complete to solve 3x2 + 12x 18 = 0.
(x + 3)2 + or (x + 3)2 Give your answer correct to 2 decimal places.
a x2 + 6x + 10 3x2 + 12x 18 = 0
b x2 + 6x + 7 3(x2 + x ) = 0
c x2 + 6x 1 x2 + x = 0
(x + )2 2 = 0
3 Write these as perfect squares.
(x + )2 = 0
a x2 + 4x + 4
(x + )2 = __
b x2 + 10x + 25
__ x + = __

c x2 + 16x + 64

x = + or x =
d x2 + 20x + 100
4 Write these in the form Q10 hint Begin by dividing every term by
Example
(x + p)2 + q the coefficient of x2, 3. Then complete the
square for the first two terms. Rearrange
a x2 + 4x + 7 terms and then take the square root of both
b x2 + 10x + 20 sides.
c x2 + 14x
d x2 + 8x + 20 11 Solve these quadratic equations, giving your
e x2 6x + 10 answers correct to 2 decimal places.
a 2x2 + 12x 4 = 0 b 4x2 32x 9 = 0
5 Copy and complete to solve the quadratic
c 3x2 6x 8 = 0 d 6x2 + 24x 15 = 0
equation, giving your answer in surd form.
e 4x + 20x 7 = 0
2
x2 + 6x + 6 = 0
(x + )2 + 6 = 0 12 Exam-style question
(x + )2 = __ Solve 5x2 + 30x + 8 = 0 by completing
(x__+ ) = __ thesquare.
x = or x = +
Give your answer correct to
2 decimal places. (3 marks)
6 Solve these quadratic equations, giving your
answers in surd form.
a x2 + 4x + 2 = 0
9.4 Solving simple
b x2 + 10x + 18 = 0
c x2 + 2x 4 = 0 simultaneous equations
7 Copy and complete to write the expression 1 Solve the simultaneous equations
2x2 + 8x + 35 in the form p(x + q)2 + r a y = 4 b y = 2
2x2 + 8x + 35 = 2( + ) + 35 3x + y = 13 4x + y = 10
= 2[(x + )2 ] + 35
c y = 3 d y = 3x
= 2(x + )2 8 + 35 5x + 2y = 31 2x + 3y = 22
= 2(x + )2 +
e y = x 1 f y=x+2
Q7 hint Factorise the x2 and x terms. Then x + y = 9 x + 2y = 13
complete the square for the expression g 4x y = 0 h y 2x = 0
inside the brackets. Simplify so that you 3x + 2y = 11 3x + y = 20
have p(x + q)2 + r
2 P Two shirts and one pair of trousers cost 40.
A pair of trousers costs 4 more than a shirt.
8 Write these in the form a(x + p)2 + q
a How much is a shirt?
a 3x2 + 6x 5 b 4x2 16x + 11
b How much is a pair of trousers?
c 2x + 12x 7 d
2 3x2 18x + 10
3 P Alfie buys 3 fish and 2 portions of chips
9 Solve these equations by completing the for 12. Lauren buys 4 fish and 4 portions of
square. Give your answers in surd form. chips for 18.
a 2x2 + 8x 14 = 0 b 3x2 24x + 30 = 0 How much does one portion of chips cost?

82 R = Reasoning P = Problem-solving 9.3 Completing the square


4 Solve these simultaneous equations. 8 Exam-style question
a 4x + y = 16 Example
Solve the simultaneous equations
3x y = 5
3x y = 9
b 3x + y = 11
x+y=7 2x + 3y = 5  (3 marks)
c 6x 2y = 20 9 Solve the simultaneous equations
5x 2y = 16
4x + y = 0
5 Emily solves simultaneous equations in x + 4y = 0
thisway.
10 P The sum of two numbers is 29 and their
2x y = 7
difference is 7.
x + y = 5
Let the two numbers be x and y.
y = 5 x
Write two equations and solve them to find
2x (5 x) = 7
the two numbers.
2x 5 + x = 7
3x 5 = 7 11 P A plumber charges a call-out fee of x
plus an hourly rate of y per hour. Callum
3x = 12
pays 90 to call out the plumber for 3 hours.
x = 4
Louise pays 110 for 4 hours.
4 + y = 5
a Work out the call-out charge.
So y = 1
b How much would you have to pay for
(2 4) 1 = 7 3 5hours?
Solve the simultaneous equations in
Q4 part a using Emilys method. 12 Exam-style question
The diagram shows a rectangle.
Q5 hint Write the second equation with All sides are measured in centimetres.
y as the subject. Substitute y = into the
first equation then collect like terms to y
find x. Substitute x = into the second
equation and check your solutions work by y25 x
substituting them into the first equation.
2x 2 4
6 a Copy and complete to solve the
simultaneous equations. Amy says the perimeter of the rectangle is
less than 50cm.
3x 2 y 5 13
Show that Amy is correct. (5 marks)
32 2x 1 2y 5 14 32
x 2 2y 5
2x 1 2y 5 14 9.5 M
 ore simultaneous
x 2 2y 5
equations
x10 5
x 5 1 a Write the equation of a line through (2, 7).
b Substitute your value of x into equation b Write the equation of a line through (3, 11).
to find y. c Solve your simultaneous equations from
parts a and b to find m and c.
Q6a hint Multiply every term in equation d Write the equation of the line through the
by 2. Then add equations and .
points (2, 7) and (3, 11).
7 Solve these simultaneous equations. Q1a hint Substitute x = 2, y = 7 into
a 2x y = 4 b 2x 2y = 6 y = mx + c
4x + 3y = 18 x + 3y = 7
Q1d hint Substitute your values of m and c
c x 2y = 6 d 3x y = 10
from part c into y = mx + c
2x + 3y = 2 2x + 3y = 3

R = Reasoning P = Problem-solving 9.5 More simultaneous equations 83


2 Find the equation of the line through the 10 The diagram shows a rectangle with all
points (3, 11) and (2, 1). measurements in centimetres.
2y 1 2
3 Solve these simultaneous equations.
a 4x + 3y = 24 Example 2y 2 5 x14
5x 2y = 7
b 7x + 2y = 15 3x 1 1
4x + 4y = 20 a Write down a pair of simultaneous
c 5x + 3y = 9 equations in x and y.
3x 4y = 17 b Solve the equations.
d 3x 4y = 4 c Give the dimensions of the rectangle.
2x + 2.5y = 13
e 2x + 3y = 9
3x 2y = 19 9.6 Solving linear and
4 P In a caf, two sandwiches and two bananas
quadratic simultaneous
cost 4, and two sandwiches and one banana equations
cost 3.50.
Work out the cost of a sandwich and the cost 1 Solve these simultaneous equations.
of a banana. a y = x Example
x2 + y = 20
5 P Henry buys two adult and three child
cinema tickets for 41.20. b x y = 5
Rachel buys two adult and four child tickets x2 11 = y
for 48.40. c y + 2x = 7
How much is an adult ticket and how much is 2x2 + 5x + 10 = y
a child ticket? d y = 3x + 2 e x2 + y2 = 16
y = 3x2 + 11x + 7 y = 2x 2
6 P Holly buys two pens and three pencils in a
shop and pays 3.45. 2 Solve these simultaneous equations.
At the same time Leah buys three pens and Give your answers correct to 2 decimal places.
two pencils and pays 2.70. a y 2x = 5 b 3y + 2x = 11
What is the cost of a pen? y = x2 4x + 3 y = x2 + 2x 5
What is the cost of a pencil? 3 R The diagram shows a sketch of the curve
y = 3(x2 x). The curve crosses the straight
7 Exam-style question line with equation y = 2 2x at two points.
Solve the simultaneous equations. y
5x + 4y = 4
3x + 8y = 6 (3 marks)

8 P The maximum load of a service lift is O 1 x


200kg. Two possible maximum loads are
4small crates and 2 large crates, or 1 small Find the coordinates of the points where they
crate and 3 large crates. intersect.
What is the mass of a small crate? 4 Solve these simultaneous equations.
What is the mass of a large crate?
a y = 2x2 3x + 1 b y = 3x2 9x + 3
9 P Hire charges for a taxi consist of x fixed x + y = 5 y = 2x + 7
charge + y pence for each mile travelled. c y = x 9x + 10 d
2 y = 2x2 + 3x 10
A 5-mile journey costs 6. y + 3x = 2 y 2 = 5x
A 10-mile journey costs 10. e y = x 2 f y = 2x + 3
How much would a 20-mile journey cost? y2 = 14 x y2 = 8x + 33

84 R = Reasoning P = Problem-solving 9.5 More simultaneous equations


5 R The diagram 9 Solve these simultaneous equations.
shows a patio Garden Give your answers correct to 3 significant
within a garden. figures.
2x 2 1
The length of a x2 + y2 = 24 b x2 + y2 = 30
the patio is Patio x y = x + 2 y = 2x 3
half the length c x + y = 65
2 2
y
of the garden. y = 5 2x
a Write an equation to represent the length
of the garden (y).
b Write an equation to represent the area of
the patio, which is 15m2.
9.7 S
 olving linear
c Use these two equations to find the value of inequalities
x and hence the length of the garden (y).
1 Write down four integers that satisfy each
6 A curve with equation y = x2 + 5x 12 crosses
inequality.
a straight line with equation y = 3x 4 in two
a x + 3 . 5
places.
b x 2 , 1
Find the coordinates of the points where they
intersect. c 2x 1 . 6
d 4x + 3 . 1
7 Exam-style question e 4 , x < 5
C is the curve with equation y = x2 x 1 f 4 < x , 0
L is the straight line with equation y = 2x + 3 g 4 . x > 2
L intersects C at two points, P and Q.
Calculate the exact length of PQ. (6 marks) 2 Write the inequalities that these number
lines represent for x.
Exam hint a
There are 2 marks each for finding P and Q, 0 1 2 3 4 5 6 x
and 2 marks for finding the length of PQ.
b
8 R The diagram shows a circle of diameter 23 22 21 0 1 2 3 x
6cm with centre at the origin. c
y 25 24 23 22 21 0 1 2 x
3 d
26 25 24 23 22 21 0 1 x
2
e
21 0 1 2 3 4 5 6 x
1
f
24 23 22 21 0 1 2 3 x
23 22 21 O 1 2 3 x

21 3 Exam-style question
2 < x , 5 where x is an integer.
22 a Write down all the possible values
of x. (2 marks)
23 b Write down the inequality represented
on the number line.
a Write the equation for the circle.
b Use an algebraic method to find the 25 24 23 22 21 0 1 2 x
points where the line y = 3x + 1 crosses (2 marks)
the circle.

R = Reasoning P = Problem-solving 9.7 Solving linear inequalities 85


4 Draw number lines to show these 11 a Multiply both sides of the inequality 4 , 7
inequalities. by 1. Is the inequality still true?
a x , 3 b Divide both sides of the inequality 12 . 9
b 2 < x < 4 by 3. Is the inequality still true?
c 4 , x , 0 c What happens when you multiply or divide
d 3 , x < 2 an inequality by a negative number?
12 Find the possible integer values of x in these
5 {x:x . 2} means the set of all x values such inequalities.
that x is greater than 2.
a 7 , x , 3
Write the meaning of these sets.
b 8 , 2x , 6
a {x:x . 4}
c 6 < 10 2x < 16
b {x:x < 2}
d 4 < 8 3x , 14
c {x:x , 0}
d {x:x . 0} 13 Solve these inequalities and write the
e {x:x > 3} solutions using set notation.
a 2(x + 3) > 4x + 2
6 Show the sets in Q5 on number lines. b 4x 3 . 7 x
c 7 < 3x 4 , 5
d 4 < 2(3 x) < 18
7 Solve each inequality and Example
show your answer on a
number line.
Write the solutions using
9 Problem-solving
set notation.
a 2x , 10 Solve problems using these strategies where
appropriate:
b 4x > 8
c 3x + 2 . 5 Use pictures or lists
d 3x 4 < 8 Use smaller numbers
Use bar models
8 Exam-style question Use x for the unknown
4x 7 , 12 Use a flow diagram.
Find the largest integer value of x. (3 marks)
1 Three lookout towers of a castle form the
corners of a triangle. The North Tower is 85m
Q8 hint Use the inequality symbol on each from the East Tower. The East Tower is 65m
line of your working. from the West Tower, and the West Tower is
70m from the North Tower.
9 Solve these inequalities and write the Use a scale drawing to show the shape and
solutions using set notation. dimensions of the triangle formed by the
a 4(x + 1) , 12 three towers.
b 3(x 2) > 2x 2
2 Exam-style question
c 5x 1 . 2(2x 1)
d 3(7 x) , 4(3x 6) 2 < n , 5 where n is an integer.
a Write down all possible values
10 Solve for n. (2 marks)
a 3 , 2x 1 < 9 b Write down the inequality represented
b 4 < 3x + 2 , 11 on the number line.

3x
c 3 , ___, 6 25 24 23 22 21 0 1 2 3 4 5 x
2
2x+3 (2 marks)
d 1 < ______
, 1
5

86 R = Reasoning P = Problem-solving 9.7 Solving linear inequalities


3 R Marcia is decorating a gift box she has 6 Find the possible integer values of x in these
made in the form of a triangular prism. inequalities.
Shehas enough glitter to cover 400cm2. a 10 , 3x 4 , 8
b 3 < 2x 5 , 3

7 Solve 2x2 + 4x 3 = 0.
Give your answer correct to 2 decimal places.
8 cm
12 cm 8 R The outdoor area of a pre-school is 42m2.
6 cm The storage shed in the outdoor area has
dimensions b b.
a Does Marcia have enough glitter to The dimensions of the outdoor area are
completely cover the box? 3b (b + 5).
b What extra information did you need What are the dimensions of the shed?
to find before you could answer part a?
Howdid you find it? Q8 hint Draw a picture.
c What is the volume of the box?
4 R Jessica is putting together a flat-pack 9 Solve these simultaneous equations.
bookcase. Some of the shelves are slightly 5x + y = 6
longer than others even though they are all x2 + y = 12
meant to be the same length. She works out
that the lengths have been rounded to the 10 R Three equations are written on the board.
nearest centimetre. x2 5x + 7 = 0
a If a shelf is meant to be 75cm what are x2 + y2 = 34
the possible values it could be? y = 3x + 4
b If one shelf is the upper bound value and a Clay has been asked to choose one of the
one shelf is the lower bound value, what is equations to draw a circle.
the size of the gap that would be created?
Which equation does Clay use to draw the
c How would rounding to the nearest tenth circle?
of a centimetre affect the measurements
b What is the centre point of this circle?
of the shelves?
c Clay is asked to choose one of the other
5 Jonathan has two bags of oranges and three equations to draw a straight line.
bags of apples. They weigh 17lb. Which equation does he choose?
Corinne has three bags of oranges and four d Use the equations from parts a and c and
bags of apples. They weigh 24lb. solve them simultaneously to find the
How much does one bag of apples weigh? points where the line intersects the circle.

R = Reasoning P = Problem-solving 9 Problem-solving 87


10 PROBABILITY
a Copy and complete the sample space
10.1 Combined events diagram to show all the possible
outcomes.
1 T-shirts are made in five different colours b Work out the probability of getting
(red, blue, navy blue, green and pink) and i a product of 6
with four different logos (cat, dog, fox, owl). ii a product that is a prime number
a How many possible combinations are iii a product of 20.
there?
b What is the probability that the 5 Exam-style question
combination chosen is red with an owl Bethan rolls two dice.
logo? She adds the scores on the dice together.
c What is the probability that the She records the possible scores in a table.
combination chosen is not red with a
foxlogo? + 1 2 3 4 5 6
1 2 3 4 5 6 7
2 At a school, students choose two of five
2 3 4 5 6 7 8
different activities: drama, mountain biking,
hockey, art and orchestra. 3 4 5 6 7 8 9
a How many combinations of two activities 4 5 6 7 8 9 10
are there? 5 6 7 8
b What is the probability that a student will 6 7 8 9
choose art?
a Complete the table of possible
c What is the probability that a student will
scores. (1 mark)
choose mountain biking?
b Write down all the ways in which Bethan
d What is the probability that a student can get a total score of 7
chooses art and drama?
One way has been done for you.
3 A coin is flipped and an ordinary six-sided (1, 6) (2 marks)
dice is rolled. Both dice are fair.
a Write a list of all the possible outcomes. c Find the probability that Bethans total
b How many outcomes are there altogether? score is prime. (2 marks)
c Work out
i P(head and 2) 6 Aiden spins a three-sided spinner labelled
ii P(tail and odd number). with the numbers 1, 3, 5 and a four-sided
4 Amelia rolls a six-sided dice Example spinner labelled with the numbers 2, 4, 6, 8,
and spins a four-sided then finds the sum of the two numbers from
spinner, then multiplies the his spins.
results together. a Draw a sample space diagram to show all
of the possible outcomes.
b How many possible outcomes are there
Dice altogether?
1 2 3 4 5 6 c Work out the probability of getting a
totalof
1 1 2
i 5
2 2 4 ii an odd number
Spinner
3 3 iii less than 12.
4 d Which total are you most likely to get?

88 R = Reasoning P = Problem-solving 10.1 Combined events


7 Two bags, A and B, contain coloured marbles. 4 The chance of a traffic light being red is 35%.
Bag A has 2 green, 1 red and 1 blue. The chance of it being amber is 17%.
Bag B has 3 red and 1 blue. What is the probability that next time you are
Logan takes a marble at random from each bag. at the lights it will be green?
a Draw a sample space diagram to show all
5 The lettered cards are shuffled. A card is
the possible outcomes.
picked at random.
b Work out the probability that the marbles
will be A B C D E F G H I J
i both green
ii the same colour Work out the probability of picking a vowel,
iii different colours. aletter in the word BAD or a letter in the
word CAT.
8 Shen rolls a dice twice.
Work out the probability that he will roll the 6 P The table gives the probability of getting
same number on both rolls. each of 1, 2, 3, 4, 5 and 6 on a biased dice.
Q8 hint List all possible outcomes. Number 1 2 3 4 5 6
Probability 0.5 2a 4a 2a a a
9 A group of six students, Mohammed, Susan,
Jackson, Henry, Charles and Eloise, sit Work out the probability of getting
together in a science class. Two students a 2 or 4 b an odd number.
from the group are picked at random to
7 A fair four-sided spinner Example
demonstrate an experiment.
showing the numbers 14
Work out the probability that both the
is spun.
students are male.
Work out the probability of
getting
10.2 M
 utually exclusive a 4 b not 4.
events 8 A letter is picked at random from the alphabet.
Find the probability of picking
1 A five-sided spinner lettered A, B, C, D, E is spun. a a vowel b not a vowel.
Which two of the following events are
mutually exclusive? 9 The probability that it will snow tomorrow
is0.37.
A Spinning a vowel
Work out the probability that it will not snow
B Spinning a letter in the word ADD
tomorrow.
C Spinning a consonant
2 A fair six-sided dice is rolled once. 10 Exam-style question
Work out the probability of rolling Here is a four-sided spinner. 4
a a prime number or an odd number The spinner is biased.
3
1

b a factor of 6 or an even number The table shows the 2


c a square number or a prime number. probabilities that the spinner
will land on 1 or on 3.
3 A standard pack of cards is shuffled and a
card is picked at random. Number 1 2 3 4
Find the probability of picking Probability 0.2 0.1
a a king or a 2 The probability that the spinner will land
b a red queen or a club. on 2is the same as the probability that the
spinner will land on 4.
Q3 hint A standard pack of 52 cards is equally Work out the probability that the spinner
split into four suits: hearts, diamonds, clubs willland on 4. (3 marks)
and spades. For each suit there is an ace, 2, 3,
March 2014, Q4, 1MA0/2H
4, 5, 6, 7, 8, 9, 10, jack, queen and king.

R = Reasoning P = Problem-solving 10.2 Mutually exclusive events 89


11 R William has a box of biscuits. In the box 3 Ahuva rolls a ten-sided dice Example
there are chocolate, wafer and sugar-coated numbered 110.
biscuits in the ratio 2:4:5. a What is the theoretical
William doesnt like the wafers. probability that the dice
Work out the probability that he will pick a will land on 7?
biscuit at random that is not a wafer. Ahuva rolls the dice 200 times.
12 A and B are two mutually exclusive events. b Estimate how many times the dice will
P(A) = 0.3 and P(A or B) = 0.73 land on 7.
Work out the value of P(B). 4 There are toffees, chocolates and mints in a
bag in the ratio 2:7:3.
Q12 hint For two mutually exclusive events,
P(A or B) = P(A) + P(B).
a What is the probability of picking a mint?
A sweet is picked at random from the bag
13 P C and D are two mutually exclusive events. and then replaced. This is done 240 times.
P(D) = 0.11 and P(C or D) = 0.7 b How many mints would you expect to be
Work out P(not C). picked?
5 The probability of a person failing their
10.3 E
 xperimental driving test is 0.64.
Work out an estimate for how many of the
probability next 200 people who take the test will pass.
6 There are 24 counters in a bag. There are red,
1 Eli dropped a piece of buttered toast lots of
blue, yellow and green counters.
times. Itlanded either butter up or butter down.
A counter is picked from the bag and then
He recorded his results in a frequency table.
replaced.
Position Frequency The table shows the results.
Butter down 48
Colour Frequency Relative frequency
Butter up 52
Red 35
a Work out the total frequency. Blue 14
b Work out the experimental probability of Yellow 33
the toast landing
Green 18
i butter up ii butter down.
c He drops the toast 50 more times. a Copy and complete the table, calculating
How many times do you expect it to land the relative frequency for each outcome.
butter down? b What is the experimental probability of
picking a red?
2 Exam-style question
c The experiment is repeated 400 times.
A car manufacturer wants to work out an How many times would you expect to get
estimate for the number of cars of each ared?
colour that will be bought next year. d Are there the same number of counters of
The Managing Director says to record the each colour? Explain your answer.
colours of the next 100 cars sold.
The Assistant Director says to record the 7 The table shows the results of rolling a dice.
colours of the next 1000 cars sold. Number 1 2 3 4 5 6
Who is more likely to get the better estimate?
Frequency 18 13 14 16 17 22
Give a reason for your answer. (1 mark)
Is the dice fair? Give a reason for your answer.
Exam hint 8 Isaac rolls two dice and records the result.
You only get the mark for the job title and Hedoes this 180 times.
a reason. Just writing the correct name One possible outcome is (2, 2). Estimate the
scores 0 marks. number of times he will get two 2s.

90 R = Reasoning P = Problem-solving 10.2 Mutually exclusive events


9 The probability of winning a game is __ 1
10. 3 60 customers bought a holiday at a travel
Ghayth says that if he plays 10 times he will agents one day.
win. Of the 60 customers, 45 bought a holiday
Is he right? Give a reason for your answer. abroad.
10 Ben flips two coins 100 times. A total of 35 of the customers paid in full.
Of these, 13 were going on holiday in the UK.
How many times would you expect him to get
a two heads b one head and one tail? Paid in full

11 A vet estimates that the probability of an Abroad


animal brought to the vets being a dog is0.47. Part paid
Of the next 150 animals brought in, 37 are 60
Paid in full
dogs. UK
Is the vets estimate a good one?
Explain your answer. Part paid

a Copy and complete the frequency tree.


10.4 I ndependent events b Work out the probability that a person
and tree diagrams who goes abroad pays in full.

4 Two teams, A and B, play each other at


1 There were 30 customers in a restaurant one hockey and at football.
lunchtime. Of these, 17 chose the set menu. The probability that Team A wins the hockey
13 of the customers who chose the set menu is 0.5. The probability that Team B wins the
had coffee. football is 0.7.
25 customers ordered coffee in total. Assuming that the two events are
Copy and complete the frequency tree for the independent, work out the probability that
30 customers. Team A wins both games.
Coffee
5 R The probability that it rains is 0.3.
Set menu 17
Theprobability that Amy remembers
No coffee her coat is 0.7.
30 Assuming that the two events are
Coffee
independent, work out the probability that
Other
a it rains and Amy has her coat
No coffee b it doesnt rain and Amy doesnt have
hercoat
2 A driving test centre examines 50 students c it rains and Amy doesnt have her coat.
one day.
Of the 50 students, 27 are female. 6 Two letters are picked one after the other
A total of 32 students passed, 18 of whom from a set of alphabet cards.
were male. (The same letter can be picked twice.)
Work out the probability that
Pass
a both letters are vowels
Male b both letters are consonants
Fail c the first letter is a vowel and the second
50
letter is a consonant
Pass
Female d the first letter is a vowel and the second is
in the word CONSONANT
Fail e the first letter is a consonant and the
second is a vowel.
a Copy and complete the frequency tree.
b Work out the probability that a student 7 A fair coin is flipped. What is the probability
picked at random is a male who fails. of flipping three heads one after the other?

R = Reasoning P = Problem-solving 10.4 Independent events and tree diagrams 91


8 In a game, a set of cards is Example b Work out the probability that
numbered 110. i he wins both games
If you pick a prime number ii he wins the first game but not the
you win a prize. second.

1 2 3 4 5 11 Exam-style question
A fair dice is rolled.
6 7 8 9 10
Hanifa notes whether it is a square number.
James plays the game twice. She then repeats this process.
a Complete the probability tree diagram.
a Copy and complete the tree diagram to 1
show the probabilities 3 Square
1st game 2nd game 1 Square
3
2 Not square
5 Win
4 52 Square
Win
10 5
Lose Not
square Not
Win square
Lose
(2 marks)
Lose
b Work out the probability that Hanifa
b What is the probability of winning rolls at least one square number.
i two prizes ii nothing  (3 marks)
iii one prize iv at least one prize?
9 Marganita has two bags of sweets. 12 R Daniel spins two spinners.
Bag A contains 7 toffees and 3 mints. On spinner 1, P(blue) = 0.1
Bag B contains 4 toffees and 6 mints. On spinner 2, P(blue) = 0.72
She picks a sweet at random from each bag. a Draw a tree diagram to show all the
a Copy and complete the tree diagram to possible outcomes.
show the probabilities. b What is the probability of only one spinner
Bag A Bag B landing on blue?
c What is the probability of both spinners
Toffee
Toffee landing on blue?
Mint d If each spinner was spun 1000 times, how
many times would you expect them both
Toffee to land on blue?
Mint
Mint
b Work out the probability of picking 10.5 C
 onditional
i two sweets the same
ii one toffee and one mint
probability
iii no mints
iv at least one toffee. 1 For each of the events, state if the events are
independent or dependent.
10 Terry plays two games on his phone. He has a Picking a card from a pack, replacing it and
a 1 in 10 chance of winning the first game then picking another one.
and a 1 in 5 chance of winning the second.
b Flipping a coin and rolling a dice.
a Copy and Game A Game B
complete c Picking two marbles from a bag, one after
Win the other.
the tree Win
diagram to Lose d Picking a counter from a bag, then rolling
show the a dice.
probabilities. Win e Picking a student from a class, then
Lose picking another student.
Lose

92 R = Reasoning P = Problem-solving 10.4 Independent events and tree diagrams


2 GCSE students in a school have to study one 6 There are 3 red, 4 blue and 5 green marbles
subject from each of two lists. in a bag. Husni picks a marble then John picks
The table shows their choices. a marble.
a Copy and complete the diagram to show
List A
all the probabilities.
History Geography French Total
1st marble 2nd marble
DT 12 7 8 27
ICT 8 17 12 37 2
List B

11 Red
Art 3 6 7 16
Red Blue
Total 23 30 27 3 51
12 4 Green
Work out the probability that a student
Red
chosen at random
a studies history Blue Blue
b studies DT Green
c studies ICT, given that he studies history Red
d studies French, given that she studies art. Green Blue
3 Marion has a bag containing Green
Example
6 chocolates and 6 mints. b What is the probability that they both pick
She chooses a sweet at the same colour?
random and eats it.
She then chooses another 7 P There are 3 tins of red paint and 11 tins
sweet at random. ofgreen paint.
a Copy and complete the tree diagram to James and Melek both take a tin at random.
show all the probabilities. Work out the probability that they do not
1st sweet 2nd sweet
pick the same colour.

5 8 P A box contains 3 new batteries, 5 partly


11 Chocolate used batteries and 4 dead batteries.
6 51 Chocolate Melissa takes two batteries at random.
12 2
Mint Work out the probability that she picks new
batteries.
Chocolate
Mint 9 Exam-style question
Mint
Here are seven tiles.
b Work out the probability that the sweets
will be 1 1 2 2 2 3 3
i both chocolate ii one of each
iii both mint. Jim takes at random a tile.
4 P In a survey, 45% of the people asked were He does not replace the tile.
male. 35% of the men and 40% of the women Jim then takes at random a second tile.
supported a football team. a Calculate the probability that both the
One person is chosen at random. tilesJim takes have the number 1
Find the probability that this is a woman who on them. (2 marks)
does not support a football team. b Calculate the probability that the
numberon the second tile Jim takes is
5 P A train is either late or on time. greater than the number on the
Theprobability it is late is 0.85. If the train is first tile he takes. (3 marks)
late, the probability Mr Murphy is late is 0.7. November 2012, Q21, 1MA0/2H
Ifthe train is on time, the probability he is
late is 0.1.
Work out the probability that Mr Murphy Q9 hint Draw a tree diagram to help you.
arrives at work on time.

R = Reasoning P = Problem-solving 10.5 Conditional probability 93


5 R Dan asked the 30 students in his class if
10.6 Venn diagrams they were studying French (F) or Spanish (S).
and set notation 15 were studying both and a total of 21 were
studying French.
They were all studying at least one language.
1 A = {square numbers , 10}
a Draw a Venn diagram to show Dans data.
B = {positive odd numbers , 10}
b Work out
a List the numbers in each set.
i P(S)
A = {1, } B = {} ii P(F > S)
b Write true or false for each statement. iii P(F < S)
i 5A iv P(F9 > S).
ii 9B
iii 11B 6 100 customers were surveyed to find out
about their shopping habits for groceries (G)
2 The Venn diagram shows two sets, P and Q, and and clothing (C). Of those surveyed:
the set of all numbers being considered, j. 64 bought their groceries online
P Q 13 bought both groceries and clothing online
8 bought neither online.
3 15 4
a Draw a Venn diagram to show the data.
12
6 8 b Work out the probability that a customer
9 from the sample buys clothing online.
14
j 1 2 5 7 10 11 13 c Work out the probability that a customer
from the sample buys groceries online,
Write all the elements of each set inside curly given that they buy clothing online.
brackets { }.
a P b Q c j 7 At a school, the students can play football (F),
hockey (H) or tennis (T).
d Which set is {multiples of 3 < 15}?
Nahal carried out a survey to find out which
e Write descriptions of the other two sets.
sport students in his year played.
3 For the Venn diagram in Q2, write these sets. He recorded his results in a Venn diagram.
a P < Q b P>Q
F H
c P9 d Q9
e P9 > Q f Q9 > P 20 7 4
4 The Venn diagram shows two Example 3
8 2
events when a 12-sided dice
is rolled: square numbers and 4
factors of 6.
j T
X = {number is square}
Y = {number is a factor of 6} a How many students took part in the
X Y survey?
One of the students is chosen at random.
4 2
b Work out
1 3
i P(H)
9 6 ii P(F > T | F)
12 iii P(T > H | H).
j 5 7 8 10 11

Work out
a P(X) b P(Y)
c P(X > Y) d P(X < Y)
e P(X9) f P(Y9)
g P(X > Y9) h
P(X9 < Y9).

94 R = Reasoning P = Problem-solving 10.6 Venn diagrams and set notation


8 The Venn diagram shows the instruments 1 R Maya is playing a board game with Ellen.
played by members of an orchestra: violin (V), She has two dice and needs to roll two 6s.
flute (F) and piano (P). Ellen says that two 6s is the hardest roll
toget.
V F
a Draw the sample space diagram and find
0 the probability of rolling two 6s.
8 6
b What is the probability of rolling
4
2 3 i two 4s
ii two 5s
6 iii a 3 and a 2?
j P c Was Ellen correct?
Explain your answer.
a How many people play all three
2 Frankie is trying to find a matching pair of
instruments?
socks. A blue, a red, a black and two green
b How many people are in the orchestra? socks have been washed and are in a clean
c Work out pile. A blue, two white, a red and a black sock
i P(V > P > F) are still in the dryer. Frankie takes one sock
ii P(V > F) from the dryer and one from the pile, both of
iii P(V < P | P). them at random.
What is the probability that she picks out a
9 Exam-style question matching pair of socks?
There are 100 students studying maths. Draw a sample space diagram to show all the
All 100 study at least one of three courses, possible outcomes.
Pure, Mechanics and Statistics. 3 R Kai wants to make a set of nesting gift
18 study all three. boxes. He uses a grid to draw bases for the
24 study Pure and Mechanics. the different sizes of box.
31 study Pure and Statistics. Kai draws a 3cm 4cm rectangle using the
22 study Mechanics and Statistics. points (0,0), (4,0), (4,3) and (0,3).
57 study Pure. Using (0,0) as his centre of enlargement,
37 study Mechanics. hedraws the bases of the next two boxes.
a Draw a Venn diagram to show this Kai uses scale factors of 1.5 and 2.
information. (3 marks) a Use a grid of 0 to 12 on both axes. Draw the
One of the students is chosen at random. first box and the two enlargements.
b Work out the probability that this student b Kai wants to draw the bases for two more
studies Pure but not Mechanics.(2 marks) sizes. Thefourth box has a width of 7.5cm
Given that the student studies Statistics, and the fifth has a width of 9cm.
c work out the probability that this What scale factors (from the first box) does
student also studies Mechanics.(2 marks) Kai use for the fourth and fifth boxes?
c Draw the bases of the fourth and fifth
boxes on the grid.
4 R A biased dice is twice as likely to land on a
10 Problem-solving 6as on a 5 or a 2.
It is three times as likely to land on a 1 as on
Solve problems using these strategies where a 2, but a 4 is only half as likely as a 1.
appropriate: Landing on a 3 is equally as likely as landing
Use pictures or lists on a 4.
Use smaller numbers Work out the probability for landing on each
Use bar models number. Write your answers as percentages.
Use x for the unknown
Q4 hint Use a flow diagram to work out the
Use a flow diagram. probability of each event in turn.

R = Reasoning P = Problem-solving 10 Problem-solving 95


5 R Blake is making a five-sided spinner 9 R A nursery school teacher has a box of
labelled 15 for a board game he has made. stickers. There are 10 smiley face stickers,
He wants to check that the spinner is fair. 5bear stickers and 5 rainbow stickers.
a How can Blake test the spinner? Twochildren take a sticker at random.
Blake spins the spinner 200 times. a Draw a diagram to show all the
The table shows his results. probabilities.
b What is the probability that the two
Number 1 2 3 4 5
children will take different stickers?
Frequency 42 36 45 38 39
10 Mike and Syeona are rolling two ordinary
b Is the spinner fair?
dice. They make a Venn diagram to show
Explain your answer.
all the possible outcomes of the totals, and
6 R Jackson buys four apple and berry whether they are multiples of 2, multiples of
smoothies and six cartons of orange juice. 3 or a prime number.
He pays 29.60. j
Multiples of 2 (M2)
April buys five apple and berry smoothies
and three cartons of orange juice. 4 8
Shepays 24.40.
How much does one apple and berry 6 2
smoothie cost? How much does one carton of 5
3
orange juice cost? 7
Multiples of 3 (M3) Primes (Pr)
7 A glass hemisphere dome and base are being
made for a garden fountain. The diameter of
a Copy and complete the Venn diagram.
the dome is 1.2m. What are the surface area
and volume? Give your answers in terms of p. b Mike uses the diagram to find
i P(M2 > M3)
8 Exam-style question ii P(M39)
There are 30 dogs at the animal shelter. iii P(Pr > M2 | Pr)
Each dog is at least one colour from black, c Syeona writes the following in set notation
brown and white. for Mike to find.
3 of the dogs are black, brown and white. Totals that are either multiples of 2 or 3,
15 of the dogs are black and white. given that the total is not a multiple of
5 of the dogs are brown and white. both 2 and 3.
8 of the dogs are black and brown. Write the set notation and find the
22 of the dogs are black.
probability.
15 of the dogs are brown.
a Draw a Venn diagram to show this
information. (3 marks)
One of the dogs is chosen at random.
b Work out the probability that this dog is
brown but not white. (2 marks)
Given that the dog is black,
c work out the probability that this dog is
also brown. (2 marks)

96 R = Reasoning P = Problem-solving 10 Problem-solving


11 MULTIPLICATIVE REASONING
7 Jade buys a car for 16000.
11.1 Growth and decay In the first year the car depreciates by 22%.
In the second year it depreciates by 15%.
1 Caz bought a caravan for 8000. It lost 25% What is the value of the car at the end of the
of its value in the first year. It lost 18% of its second year?
value in the second year. Work out
8 5000 is invested for 2 years at 2.7% per
a the multiplier to find the value of the
annum compound interest. Work out the
caravan at the end of the first year
total amount in the account after 2 years.
b the value of the caravan at the end of the
Give your answer to the nearest penny.
first year
c the multiplier to find the value of the 9 R 10500 is invested for 2 years at 5.5% per
caravan at the end of the second year annum compound interest.
d the value of the caravan at the end of the a Work out the total amount in the account
second year after 2 years, to the nearest penny.
e the decimal multiplier that the original b How much interest is paid in total for the
value of the caravan can be multiplied by 2years?
to find its value at the end of 2 years.
10 R a Are these formulae the same or different?
Q1e hint
Formula 1: amount after n years
3 single decimal multiplier
= initial amount ( ________________ )
n
100 + interest rate


original
value at value at 100
3 end of 3 end of
value Formula 2: amount after n years
one year two years

= initial amount (1+___________ )


n
interest rate




2 Work out the decimal multiplier that represents 100
a an increase of 10% each year for 2 years b Apply each formula to Q9 and then
b a decrease of 20% for 3 years. explain your answer.
3 Su earns a salary of 27000 a year. At the 11 4500 is invested for 3 years at 2.4% per
end of the first year she is given an increase annum compound interest.
of 3%. At the end of the second year she
Work out the total interest earned over the
is given an increase of 2.5%. Work out Sus
3years.
salary at the end of 2 years.
4 R Billy says a decrease of 10% followed by 12 Exam-style question
an increase of 15% is the same as an increase Kyle puts 1600 in a savings Example
of 5%. Is Billy correct? Explain. account for 2years.
5 Work out the multiplier as a decimal number The account pays compound
for interest at an annual rate of
a an increase of 15% followed by an increase 3.7% for the first year
of 5% 2.2% for the second year.
b a decrease of 2% followed by a decrease a Work out the total amount of money
of3% inKyles account at the end of
c an increase of 4% followed by a decrease 2 years. (3 marks)
of 4%. Kyles monthly mobile phone bill increases
by 7.5% to 64.50. The cost of his broadband
6 Robina buys a house for 154000. In the first connection increases by 20% to 26.40.
year, the value of the house decreases by 4%.
b Compare the increase in Kyles mobile
In the second year, the value of the house phone bill with the increase in his
increases by 1.5%. Work out the value of the broadband connection bill. (3 marks)
house at the end of the second year.

R = Reasoning P = Problem-solving 11.1 Growth and decay 97


13 P Lee invests 75000 in a savings account 2 Water flows from a hosepipe at a rate of
for 2 years. The account pays 4.47% 1200litres per hour.
compound interest per annum. a Work out how much water flows from the
Lee has to pay 20% tax on the interest hosepipe in
earned each year. The tax is taken from the i 10 minutes ii 35 minutes.
account at the end of each year. Two identical hosepipes are used at the same
How much is in the account at the end of time to fill a garden pool. They each have the
2years? same flow rate as in part a. The pool has a
14 R Theo invested 5700 in a savings account. capacity of 216000 litres. Initially the pool is
He is paid 2.75% per annum compound empty.
interest. b Work out how many days it takes to fill the
How many years before he has 6183.30 in pool.
the savings account? 3 R Dave drives his motorbike for 180 miles
15 The number of cells in a biological sample and uses 25 litres of petrol.
increase by 7% each hour. a Work out the average rate of petrol usage.
A scientist estimated there were 1.2 million State the units with your answer.
cells present at time t = 0. b Estimate the amount of petrol Dave
a How many complete hours later will the would use if he travels 275 miles on
number of cells reach 1.68 million? the motorbike. Give your answer to an
b Estimate the number of cells 24 hours appropriate degree of accuracy.
after time t = 0, to 3 significant figures.
4 Convert these speeds from Example
16 In 2014 the rate of increase in the population m/h to km/h.
of the UK was 0.3%. The population of the UK a 7500m/h
in 2014 was 63.7 million. b 970m/h
If the population keeps growing at the same
c 36500m/h
rate, how big is the population likely to be in
2025? d 243600m/h

17 A new virus has spread among a population 5 R Convert these speeds from metres per
at a rate of 17.5% per day. In a city there are second (m/s) to metres per hour (m/h).
now 371 cases. a 3m/s b 15m/s
How many people had the virus 7 days c 100m/s d 0.5m/s
earlier? e Would more or fewer metres be travelled
in 1 hour than in 1 second?
11.2 Compound measures 6 Copy and complete the table.

1 P Abdul works for 40 hours from Monday to Metres per second Kilometres per hour
Friday. His rate of pay for working Monday to 10
Friday is 7.90 an hour. 25
Abdul is paid time and a half for each hour 126
he works on a Saturday and double time for
180
each hour he works on a Sunday.
a How much is Abdul paid for a week when 7 The average speed of a small plane is 360km/h.
he works 40 hours from Monday to Friday, What is this speed in metres per second?
plus 5 hours on Saturday and 4 hours on
Sunday? 8 P / R The new fast train in the UK has an
b In one week Abdul works 40 hours from expected top speed of 272km/h. Usain Bolts
Monday to Friday and some hours on fastest recorded running speed is 10.44m/s
Sunday. He is paid 371.30 for the week. (run on 16 August 2009).
How many hours did Abdul work on How much faster than Bolt is the train
Sunday? expected to be?

98 R = Reasoning P = Problem-solving 11.1 Growth and decay


9 a A bike travels at bm/s. 3 The cross-sectional area of Example
Write an expression for this speed in km/h. this plastic cylinder is 300cm2.
b A man runs at ckm/h. Its length is 100cm.
Write an expression for this speed in m/s.

10 Exam-style question
Paul travels 45 miles in 1 hour and 20
minutes and then 120km in 2_2hours.
1 Area 5
300 cm2
5 miles = 8 kilometres 100 cm
What is his average speed for the total
journeyin km/h to the nearest km? (3 marks) The plastic has a density of 2.34g/cm3.
What is the mass of the cylinder?
11 Kelly flies for 45 minutes at an average speed 4 Air has density 0.0012g/cm3.
of 15m/s in a hang glider. The mass of air in a room is 54kg.
How far has Kelly flown in kilometres? What is the volume of the room?
12 Archie skis cross-country for a distance of 5 The density of iron is 8000kg/m3.
7.5km. It takes him 40 minutes. What is the density of iron in g/m3?
What is his average skiing speed in m/s?
6 The density of gold is 19.32g/cm3.
13 A fast sports car starts from rest and What is the density of gold in kg/m3?
accelerates at 4.5m/s2 for 81m.
Use the formula v2 = u2 + 2as to work out 7 A plastic has density yg/cm3.
the final velocity in m/s, where v is the final Write an expression for its density in kg/m3.
velocity, u is the initial velocity, a is the 8 R 1m3 of water from the North Sea has mass
acceleration and s is the distance. 1025kg. 1m3 of water from the Dead Sea has
mass 1240kg.
14 A car has a velocity of 540m/minute when it
drives past a service station on a motorway. In which sea is the water more dense?
It then accelerates and travels a distance of 9 Exam-style question
1.2km in 50 seconds.
Use the formula s=ut+_12at 2to work out the An alloy is made from a mix of copper and
acceleration of the car in m/s2. tin. The density of copper is 8.94 grams per
cm3. The density of tin is 7.3 grams per cm3.
15 A motorbike travels with an acceleration of 500cm3 of the alloy is made from 360cm3
7m/s2 and reaches a speed of 63km/h in of copper with 140cm3 of tin.
2.5seconds. Work out the density of the alloy to3s.f.
Use the formula v = u + at to work out the  (3 marks)
initial velocity of the motorbike in m/s.
10 A force of 54N is applied to an area of
21600cm2.
11.3 M
 ore compound Work out the pressure in N/m2.

measures 11 A force applied to an area of 0.36m2 produces


a pressure of 50N/m2.
Work out the force in N.
1 A steel pipe has a mass of 9.765kg and a
volume of 1260cm3. 12 Copy and complete the table.
What is its density in g/cm3? Give your answers to 3 significant figures.
2 A cubic block of wood has side length 1m Force Area Pressure
and mass 690kg. 40N 3.2m2 N/m2
What is the density of the wood in g/cm3? 6.4m2 16.5N/m2
N
2000N m2 250N/m2

R = Reasoning P = Problem-solving 11.3 More compound measures 99


13 A cylindrical plant pot has a circular base with 2 The table shows some masses in both
diameter 0.3m. pounds and kilograms.
The plant pot exerts a force of 60N on the Kilograms 10 20 30 40
ground.
Pounds 22 44 66 88
Work out the pressure in N/cm2.
Give your answer to 3 significant figures. a What is the ratio of kilograms:pounds in
the form 1:n?
14 A cylinder with a movable piston in an engine b Plot a line graph for these values.
contains hot gas. c Are pounds and kilograms in direct
The pressure of the gas is 350N/m2. proportion? Explain your answer.
The area of the piston is 0.05m2. d What is the gradient of the line?
What is the force exerted by the piston? e Write a formula that shows the relationship
between kilograms and pounds.
15 R Claire sits on a table with four identical
3 The table shows the distance (s) in
legs. Each table leg has a flat rectangular
kilometres travelled by a car over a period of
base measuring 2cm by 3cm.
time (t) in hours.
Claire has a mass of 68kg and the table has a
mass of 4kg. Distance, s (km) 15 30 45 60
a Use F = mg to work out the combined Time, t (hours) 2 4 6 8
weight of Claire and the table, where g is
a Is s in direct proportion to t? Explain.
the acceleration due to gravity.
Use g = 9.8m/s2. b What is the relationship between distance
(s) and time (t)?
b Work out the pressure on the floor exerted
by the table only in N/m2, when only the c Work out the distance travelled after
four table legs are in contact with the 1.5hours.
floor. 4 The cost of buying Example
c The area of Claires feet is 330cm2. 150 US dollars is 100
Workout the pressure on the floor when 300 US dollars is 200.
Claire is standing up in bare feet. a Show that the amount in
Give your answer in N/cm2. US dollars, $D, is directly
Use g = 9.8m/s2 and give your answer proportional to the amount
to3 s.f. in sterling, P.
d Is the pressure on the floor greater when b What is the relationship between D and P?
Claire is sitting on the table or when she is c How much is 570 in US dollars?
standing up in bare feet?
5 It takes 4 cleaners 3 hours to clean a house.
Q15a hint Weight is a force on an object due How long would it take 9 cleaners?
to gravity and is measured in newtons (N). Give your answer in hours and minutes.
6 It takes 8 women 3 hours to dig a ditch.
16 a Convert 2500N/m2 to N/cm2. Howlong will it take
b Convert xN/cm2 to N/m2. a 2 women b 5 women?
c For both parts a and b, multiply the exact
answer in hours (H) by the number of
11.4 Ratio and proportion women (N). What do you notice?
7 P and Q are in inverse proportion.
1 Copy and complete these. Work out the values of A, B, C and D.
a A:B = 5:2 so A=__B P 25 10 B C 3.6
Q 7.2 A 45 62.5 D
b X:Y = 3:7, so X=__Y
c 10P = Q so Q:P = : k
Q7 hint P Q = a constant k, so Q=__
P

100 R = Reasoning P = Problem-solving 11.3 More compound measures


8 Do these equations represent direct
proportion, inverse proportion or neither? 11 Problem-solving
2
a y=__x b y = 5x c y = 10x 5
y Solve problems using these Example
d __
x=3 e xy = 6 strategies where appropriate:
Use pictures or lists
9 P The frequency, fHz, of a sound wave is
inversely proportional to the wavelength, Use smaller numbers
lm, of the sound wave. Use bar models
When the frequency is 200Hz, the Use x for the unknown
wavelength is 1.7m. Use a flow diagram
Find the wavelength when the frequency is Use arrow diagrams.
680Hz.
1 R Water flows from a hose at 0.42 litres
10 P Boyles law states that the pressure of a per second. If the hose is turned off after
gas and its volume are inversely proportional, 10minutes, will it have filled more than half
providing the temperature and the amount or less than half of a 500-litre pond?
of gas stay constant. Explain how you found your answer.
A sample of gas occupies 0.0015m3 when the 2 R Four child tickets to a show cost 14.
pressure is 100000N/m2. Five adult tickets to the show cost 31.25.
What is the volume of the sample when the a What is the cost of tickets for 9 children
pressure increases to 200000N/m2? and 13 adults?
b Two adults take some children to the show.
11 v is inversely proportional to u.
The total price of the tickets is 26.50.
v = 35 when u = 0.2.
How many children did they take to the
a Find a formula for v in terms of u. show?
b Calculate the value of v when u = 140.
3 R Therese wrote down these values:
12.5
12 a Copy and complete the table for q=____

p
5, 4, 3, 2, 1, 0, 1, 2
Write down two inequalities that these
p 1 2 5 10 values would satisfy. Use the letter n to
q represent an integer in your inequality.

b Use the table to sketch a graph of q 4 Malcolm ran for 4_34miles.


against p. Pritpal ran for 5_25miles.
c Work out the value of q when p = 50. a What is the total distance run by Malcolm
Doesit fit the shape of your graph? and Pritpal?
d Work out the value of q when p = 0.05. b How much further did Pritpal run than
Isyour answer consistent with your sketch? Malcolm?
Give your answers as mixed numbers where
13 Exam-style question appropriate.
The time, t minutes, it takes to pull an object 5 The amount, M, Sandy earns is directly
up a ramp with an electric motor is directly proportional to the number of hours, H,
proportional to the mass, mkg, of the object. she works. When H = 21, M = 472.50.
When m = 1kg, t = 2.5 minutes. Find the value of M when H = 35.
a Find m when t = 1.25 minutes. (3 marks)
The time, t minutes, it takes to pull an object 6 A survey was taken of people coming out of
up a ramp with an electric motor is inversely a grocery shop. 35% of the people were men.
proportional to the power, P watts, of the 70% of these men bought milk. 15% of the
electric motor. women did not buy milk.
When P = 1800W, t = 3 minutes. One of these customers was chosen at
b Find the value of t when P = 1600W. random to take part in a longer survey.
 (3 marks) Find the probability that this customer was a
woman who did buy milk.

R = Reasoning P = Problem-solving 11 Problem-solving 101


7 A plastic block in the shape of a cuboid 9 R Yogesh has 9866.37 in his savings
has a density of 3.9g/cm3. account. He has received 3% per annum
Ithas a length of 24cm. compound interest since opening the
The width of the block is _13of its height. account 2 years ago. He has not deposited
Its height is _14of its length. any extra money since opening the account.
What is the mass of the block? a How much did Yogesh originally deposit
when he opened the account?
8 R A company needs to complete a b Tara says that if Yogesh had originally
construction project in 8 days. deposited 500 more, he would now have
It will take 18 workers 12days to complete 500 more in his savings account.
the project. Is Tara correct? Explain how you know.
What is the minimum number of workers
they will need, to be able to complete the 10 Exam-style question
project on time? h is inversely proportional to the square of r.
When r = 5, h = 3.4.
Q8 hint More than 18 workers will be needed
Find the value of h when r = 8. (3 marks)
to complete the project in less than 12 days.
June 2013, Q22, 1MA0/2H

12 SIMILARITY AND
CONGRUENCE
c 3 cm 3 cm
12.1 Congruence
2.1 cm
4.2 cm 4.2 cm
2.1 cm
1 Here is a pair of congruent triangles.
14 cm x
35 y
d
4.5 cm 4.5 cm
87 87 35 35
6.5 cm 6.5 cm
Write down
a the size of angle y b
the length of side x. 3 State whether each pair of triangles
2 Each pair of triangles is congruent. described below is congruent or not.
Explain why. If the triangles are congruent, give the reason
a and write the corresponding vertices in pairs.
64 7 cm 72 a ABC where AB = 5cm, BC = 9cm,
7 cm angle B = 37
64 PQR where PQ = 50mm, QR = 95mm,
72
angle Q = 37
b 9 cm b ABC where AB = 6cm, angle B = 51,
11 cm angle C = 97
PQR where PQ = 6cm, angle Q = 97,
11 cm
angle R = 51
9 cm

102 R = Reasoning P = Problem-solving 11 Problem-solving


4 Which of these triangles is congruent to
triangle ABC? Give reasons for your answer. 12.2 G
 eometric proof
B D and congruence
43 1 ABC is an equilateral triangle. Example
32 10.9 cm 8.5 cm
7.7 cm C

105
E
C 6 cm A F
I 6 cm L
H
105
7.7 cm A D B
7.7 cm
10.9 cm 110 a Prove that triangle ACD is congruent to
32
J triangle BDC.
6 cm
K b Which angle is the same as angle CAB?
G
2 The diagram shows A
5 Are these triangles congruent? a kite ABCD.
Justify your answer. a Prove that triangles 48
ABC and ADC are D 95 B
13 cm 13 cm congruent.
5 cm
M N
b Find the size of
i angle ABC
12 cm
ii angle BCD.
C
6 Are all right-angled triangles with one side
3cm and one side 5cm congruent? Explain. 3 In the diagram, P S
X is the
7 ABCD is a parallelogram. midpoint of PQ. 102 2.2 cm
A B Xis also the X
3.7 cm
midpoint of ST.
Prove that the
triangles PSX and T Q
112 QTX are congruent.
D C
a Work out the size of angle ABC. 4 Exam-style question
b Show that triangle ABC and triangle ACD In the diagram, R is the midpoint of PT
are congruent. and ofQS. PQis parallel to ST.
P Q
8 Exam-style question
A B R
105
62 C
S T
D E Prove that triangle RST is congruent to
AB is parallel to DE. Which angle is equal to triangle PQR. (2 marks)
a angle CDE (1 mark)
b angle DEC (1 mark) Exam hint
c angle ECD? (1 mark) You need to write a series of logical
Triangles ABC and CDE are congruent. statements that show the statement is
d If AE = 12cm what is the true. You must give a mathematical reason
length of CE? (1 mark) for each statement.

R = Reasoning P = Problem-solving 12.2 Geometric proof and congruence 103


5 P FGH is an isosceles triangle with FG = GH. 10 P KLMN is a square. The diagonals KM and
Point E lies on FH. LN are drawn and meet at P.
EG is perpendicular to FH. Prove that the diagonals KM and LN are
a Prove that triangle FGE is congruent to perpendicular bisectors.
triangle GHE.
11 Exam-style question
b Given that angle GFE = 70 work out the
size of angle FGE. ABC is a right-angled isosceles triangle
with AB = BC.
Q5 hint Sketch the triangle. D is the midpoint of AC.
A
6 ABCD is a rectangle. A B
Use congruent
D
triangles to show
that the area of ADC
= the area of ABC. D C
B C
7 P PQRS is a parallelogram. Prove that triangle ABD is congruent to
P Q triangle CBD. (2 marks)

12.3 Similarity
S R

a Prove that triangles PQT and SRT are 1 Write the pairs of corresponding sides in
congruent. triangles X and Y.
b Hence, prove that the diagonals of a E
C
parallelogram bisect one another.
42
42
8 P ABCD is an isosceles trapezium.
Two lines are drawn from A and B that meet X Y
35
DC at right angles. 35
A B
A B D F

2 In Q1 triangle X has side AB = 5cm.


Triangle Y has side DF = 7.5cm.
Write the ratio of corresponding sides as a
fraction.
D K J C
3 Here are two right-angled triangles.
Prove that triangle ADK is congruent to
triangle BCJ. C
F 3 cm D
1 cm
9 ABCD is a rectangle. B C 4 cm
E
AC and BD are the
diagonals of the E A 12 cm B
rectangle, which a Which side in triangle ABC corresponds to
cross at point E. i DF
A D
a Draw the rectangle ii FE?
showing both diagonals. DF
b Mark on all equal angles. b Work out the ratio ___

AB
c Which triangles are congruent in your DE
c Work out the ratio ___

diagram? AC
d Using your answers to parts ac, show that d Use your answers to parts a and b to show
lines AC and BD bisect at point E. that the triangles are similar.

104 R = Reasoning P = Problem-solving 12.2 Geometric proof and congruence


4 The diagram shows parallelograms ABCD and 7 These two rectangles are Example
EFGH. similar.
Angles ABC and EFG are the same size. Find the missing side length x
F in the larger rectangle.
E

A 7 cm B x cm
8 cm
3 cm 10 cm
6 cm 8.4 cm
D C
G
6 cm
H
AD 8 Exam-style question
a Write the ratio ____

HG
b Write the ratios of the other corresponding A scale diagram of a room has length 12cm
sides. and width 8cm.
lm Diagram NOT
c Are the parallelograms similar? drawn accurately
12 cm
Explain your answer. 4m
8 cm
5 State which of the pairs of shapes are similar.
a scale diagram actual dimensions
E 6.2 cm F of room
A 3 cm B
Work out the actual area of the room.
 (3 marks)

D C 9 P A scale drawing of a shed is 5cm wide by


H G 8cm long.
C
b F The real shed is 7.5m wide.
What is the length of the shed?
12 cm 13 cm 10.4 cm 10 a Show that triangles A and B are similar.
10 cm

A 5 cm B D 3 cm E

c 4 cm hyp opp
3 cm
1.5 cm A 2 cm C
B

3 cm 60 60 60
7 cm 2 cm x adj

b Work out length x in triangle B.


c Show that triangle C is similar to A and B.
6 Show that quadrilateral ABCD is similar to Explain.
quadrilateral EFGH. adjacent
d Write down the value of __________

H hypotenuse
for these triangles.
D e What is another name for the ratio
adjacent
__________
47 G
?
C 133 hypotenuse

2 cm 3 cm 11 P A chocolate bar has length 12cm and


width 4.5cm. A miniature of the chocolate
A 2 cm B bar is similar to the large one but has length
E 3 cm F 4cm. Find the width of the miniature bar.

R = Reasoning P = Problem-solving 12.3 Similarity 105


12 P The diagrams show the octagonal base of
a summer house. 12.4 More similarity
ym
7.5 cm 4.5 m 1 Are triangles ABC and A B
x cm 108 40
CDE similar?
A 4.5 cm B 6.75 m Explain.
32

The sizes of the base are shown on diagram B. C


The sizes of the plan are shown in diagram A.
Work out the value of
E D
a x b y.
Q1 hint Find the angles in triangle CDE.
13 Are the triangles in each pair similar?
Explain.
2 a Show that triangles PQR and RST are
a similar.
12 cm
4.5 cm 22.4 cm 31.5 cm P Q
A B

3.2 cm 9 cm
b R
3 cm
4.2 cm
1.2 cm S T
x
8.4 cm S 2.8 cm T
b Find the missing length x.
3 a Explain why K
triangles FGH
and FJK are H
similar.
14 Here are two circles.
b Calculate the 9 cm
length HK. 12 cm
c Calculate the
4 cm 1 cm length JK. F J
6 cm G 2 cm
4 a Find the sizes of angles PQN and LMN.
Q

a Are they similar? Explain your answer.


b Are all circles similar? N
15 cm
15 Triangle CDE is similar to triangle FGH. 65
/CDE = /FGH 10 cm 14 cm
H 72
E P
M
4.2 cm 13.3 cm
L
C 3.1 cm 7.75 cm F
b Explain why triangle LMN is similar to
2.7 cm D triangle LPQ.
G
c Find the length of LQ.
Calculate the length of d Find the length of NQ.
a FG b FH e Find the length of MP.

106 R = Reasoning P = Problem-solving 12.3 Similarity


5 Calculate the height of the Statue of Liberty 10 Two similar triangles have areas of 45cm2
using similar triangles. and 405cm2 respectively.
The base of the smaller triangle is 3.5cm.
Find the length of the base of the larger
triangle.

31 m
11 The area of the front face of a coin is 12cm2.
What is the area of a similar coin with
57 m 114 m
a twice the diameter of the original coin
6 The diagram shows 3 cm b half the diameter of the original coin?
a logo made of 12 P A rectangular photograph has width 6inches
two similar and area 24 square inches. An enlargement of
parallelograms. it has area 486 square inches.
The perimeter of the a Work out the area scale factor.
small parallelogram is
13.5 cm b Work out the length scale factor.
9cm. The area of the small
parallelogram is 6cm .
2 c Use the length scale factor to work out the
dimensions of the enlarged photograph.
Work out the perimeter and area of the large
parallelogram. 13 P The circular tops of two tins of beans are
similar. The area of the larger top is 75cm2.
7 Exam-style question
The area of the smaller top is 8_13cm2.
A company makes two different-sized The circumference of the larger top is 31cm.
chocolate boxes. What is the circumference of the smaller top?
The smaller box has length 20cm, and Give your answer to 1 decimal place.
surface area 350cm2.
The larger box is mathematically similar to
the smaller box and has length 30cm. 12.5 Similarity in 3D solids
Work out the surface area of
a large box. (3 marks) 1 In each pair of diagrams, solid A is enlarged to
make solid B. Copy and complete the table.
8 Shape K is similar to shape L.
a

21 cm 1 A 3 B
4.2 cm 6
2
K L 1 3
b

2 2k B
The perimeter of shape K is 42cm and its A
3 3k
area is 112cm2. 1 k
Find the perimeter and area of shape L.
Linear Volume Volume Volume
9 Shape A is similar to shape B. Example scale factor of A of B scale factor
a
x b
4.2 cm
A 2 Cuboids A and B are similar.
B The volume of cuboid A is 6cm3.
5.3 cm y

A
The area of shape A is 64cm2. B
3 cm
The area of shape B is 10.24cm2.
Calculate 6 cm
a length x b length y. Calculate the volume of cuboid B.

R = Reasoning P = Problem-solving 12.5 Similarity in 3D solids 107


3 Cylinders C and D are similar. 8 P Cuboids B and C are mathematically similar.
The volume of C is 200cm3. The volume of B is ___
1
125 of the volume of C.
The surface area of B is 13.5cm2.
Work out the surface area of cuboid C.
C 7 cm D 10.5 cm
9 P A bucket has a surface area of 2200cm2
and capacity 9.4 litres.
Calculate the volume of D. Find the surface area of a similar bucket
which has a capacity of 75.2 litres.
4 Frustums E and F are similar.
The volume of frustum E is 648cm3. 10 Exam-style question
12 cm The bases of two similar cylinders, A and
8 cm B, are 846cm2 and 94cm2 respectively.
E F

A
B
Calculate the volume of frustum F.
5 Sphere J is similar to sphere K. Example
The volume of J is 8 times the The volume of cylinder A is 1296cm3.
volume of K. Show that the volume of cylinder B
is 48cm3. (5 marks)

J K 11 Exam-style question
The diagram shows two similar solids, A and B.
24 cm
Work out the diameter of sphere K. B
A
6 Prisms L and M are similar.
The volume of M is 512 times the volume of L. 3 cm
x 9 cm
2 cm
Solid A has a volume of 29cm3.
y 56 cm a Work out the volume of solid B.(2 marks)
L M
Solid B has a total surface area of 819cm2.
b Work out the total surface area
of solid A. (2 marks)
Calculate the value of
a length x b length y.
12 R Are two spheres always similar?
7 R Pyramid A and pyramid Z are similar. Explain your answer.

12 Problem-solving
A
Z Solve problems using these Example
strategies where appropriate:
The volume of pyramid A is 216 times the
Use pictures or lists
volume of pyramid Z.
Use smaller numbers
Thesurface area of pyramid Z is 11cm2.
Use bar models
a Write down the volume scale factor, k3.
Use x for the unknown
b Work out the linear scale factor, k. Use a flow diagram
c Work out the area scale factor, k2. Use arrow diagrams
d Calculate the surface area of pyramid A. Use geometric sketches.

108 R = Reasoning P = Problem-solving 12.5 Similarity in 3D solids


1 R Jason draws a right-angled triangle with 5 Martha has 60m of fencing and two plots of
hypotenuse 45cm and one side 27cm long. land. The two plots are similar. The perimeter
Yasmin draws a triangle where one side is of Plot B is 38m. Martha puts fencing around
36cm long, the hypotenuse is 45cm long and both plots of land. What length of fencing
one angle is 90. will she have left?
Are the two shapes congruent?
Explain how you know.
A B
2 At a bottle stall at a summer fair, tickets from
4.5 m
1 to 100 are placed in a bag.
Customers can pull out 5 tickets for 1. 11.25 m
To win a bottle from the stall, the number on 6 What are the roots of these quadratic
a ticket must end in either 5 or 0. functions?
The first customer picks out 5 tickets.
a y2 + 11y + 24 = 0 b t2 t 12 = 0
a What is the probability that the first ticket
c b + 6b + 8 = 0
2
will be a winner?
b What is the probability that the first two 7 R Jermain has two similar boxes, A and B.
tickets will not be winners? Box A is shown below. Box B has a height of
The first customer finds one winning ticket 9cm. What is the volume of box B?
out of the five they picked out.
c What is the probability that the next 3 cm A
customer will find a winning ticket on their 2 cm
12 cm
first turn?
8 A delivery company charges 80 per cubic
3 A rectangular park has a pathway which goes metre for express delivery. Mason pays
diagonally across it. to send two containers. On Monday, he
The park is 21m wide and 28m long. sends a container with a volume of 15m3.
There is enough tarmac to make a path On Thursday, he sends a container that is
32mlong. mathematically similar and twice as wide.
Will there be enough tarmac for the path? a How much did Mason pay to send the
container on Monday?
4 R Rectangles A and B are similar.
b How much did Mason pay to send the
Rectangle C is an enlargement of rectangle B
similar container on Thursday?
by the same scale factor as rectangle B is an
enlargement of rectangle A. 9 R XYZ is a triangle.
XY = YZ = XZ
B y cm Point O is the midpoint of XY.
30 mm A
92 mm 46 cm a What type of triangle is XYZ?
Explain your answer.
b Jamie draws the line OZ to make triangles
C zm OXZ and OYZ.
Prove these two triangles are congruent.
xm 10 Exam-style question
a Work out the value of x. WXYZ is a square. Prove that triangles
b Work out the value of y. WXY and WYZ are congruent.
c Work out the value of z. X Y

Q4 hint Find the scale factor of the


enlargement.
W Z (2 marks)

R = Reasoning P = Problem-solving 12 Problem-solving 109


13 MORE TRIGONOMETRY
6 In this diagram, the measurements are
13.1 Accuracy correct to 2 s.f.
Find the upper and lower bounds for the
1 In this diagram, the lengths of AC and BC are value of x, to 3 d.p.
given correct to 1 d.p.
77
A 14 m

x xm
3.7 cm

7 Exam-style question
C 4.5 cm B
Will wants to calculate his running speed
a Find the upper bound for the length of over 100m.
i AC ii BC. He measures the length of the track, D, as
b Use your answers to part a to work out the 100m to the nearest metre.
upper bound of x. He measures his time, T, as 12.8 seconds to
c Find the lower bound for the length of 3significant figures.
D
i AC ii BC. Will uses the formula S=__to calculate his
T
d Use your answers to part c to work out the speed.
lower bound of x. Calculate the upper bound and the lower
bound for Wills speed. (4 marks)
2 The upper bound for cosx is Example
0.3213899726 and the lower
bound is 0.3015986744. D
Q7 hint The upper bound of __ is not
a Find the upper and lower T
upper bound of D
_______________
bounds for x.
and the lower bound of
upper bound of T
b What do you notice?
D lower bound of D
__ is not _______________


3 In this diagram, the T lower bound of T
measurements are
correct to 3 s.f. 7.52 cm
5.17 cm
Find the upper and
lower bounds for x
13.2 Graph of the sine
the value of x. function
4 In this diagram, the 1 Find the value of sinu in each diagram.
measurements are a y
correct to 2 s.f. 7.5 cm
5.2 cm
1
Find the upper and
lower bounds for x
the value of x.

21 O 1 x
5 In this diagram, the
measurements are
correct to 2 s.f. 7.5 cm x cm 21
Find the upper and
lower bounds for the y
b
value of x, to 3 d.p. 62
1

110 R = Reasoning P = Problem-solving 13.1 Accuracy


21

b y 4 As u increases from 0 to 90, sinu increases


1 from 0 to 1.
Copy and complete these statements in the
same way.
a As u increases from 90 to 180,
21 O 1 x sinu ...
b As u increases from 180 to 270,
sinu ...
21 c As u increases from 270 to 360,
sinu ...
c y
1 5 Here is the graph of y = sinx for
0 < x < 180.
y
1
21 O 1 x

21

d y 0.5
1


21 O 1 x
0
0 30 60 90 120 150 180 x

a Use the graph to find


21
i sin30
ii sin15.
2 a What is the largest value that sinu b Describe the symmetry of the curve.
cantake? c Use the graph to check your answer to Q3.
b What is the smallest value that sinu d Copy and complete, inserting numbers
cantake? greater than 90.
c Find two values of u so that sinu = 0. i sin30 = sin
__

3 ii sin75 = sin
3 sin60 = ___ iii sin0 = sin
2
Use the diagram to__ find an obtuse angle u e Use the graph to give an estimate for the
solutions to sinx = 0.75.
3
such that sinu = ___
.
2 6 Here is a sketch of the graph of y = sinx for
y 0 < x < 360.
3
1 2 y
1
1 1
60
0
21 O 1 x 90 180 270 360 x

21

21 Describe the symmetry of the curve.

R = Reasoning P = Problem-solving 13.2 Graph of the sine function 111


7 The graph of y = sinx repeats every 360 in
both directions. 13.3 Graph of the cosine
a Sketch the graph of y = sinx for function
0 < x < 540.
b Use your sketch to find 1 Find the value of cosu in each diagram.
i sin390 y
a
ii sin480.
1 1
c The exact value of sin45 is ___
__ .

2
Writedown the exact value of
i sin405
ii sin495. 21 O 1 x
d Explain how you worked out your answers
to part c.
21
Q7a hint Include x-values 0, 90, 180, y
270,360, 450, 540 and y-values 1, 0.5, 0,
b
0.5, 1. 1

8 a Write down four values of x such that


1__
sinx = ___ . 21 O 1 x

2
b Write down __ four values of x such that
3
___
sinx = . 21
2
c Check each of your answers using your c y
calculator.
1
9 Exam-style question
Here is a sketch of y = sinx.
y 21 O 1 x
A
1

21
D O B x
d y
21
C 1

Write down the coordinates of each of the


labelled points. (4 marks)
21 O 1 x
10 Solve the equation 10sinx = 3 Example
for all values of x in the
21
interval 0 to 720.
__ y
3
2 cos30 = ___ 1
2
11 Solve the equation 8sinu = 6.5 for all values Use the diagram
of u in the interval 0 to 720. to find a reflex 1
angle u such __ 30
3 21 O 1 x
that cosu = ___ .
2 1

21

112 R = Reasoning P = Problem-solving 13.2 Graph of the sine function


3 cos60 = 0.5 8 Exam-style question
a Find a reflex angle u such that cosu = 0.5.
The diagram shows a sketch of the graph
b Find an obtuse angle such that cosu = 0.5. y = cosx
y
4 As u increases from 0 to 90, cosu decreases A
from 1 to 0.
Copy and complete these statements in the C
O x
same way.
a As u increases from 90 to 180,
cosu ... D B
b As u increases from 180 to 270, Write down the coordinates of points
cosu ... A, B, C and D. (4 marks)
c As u increases from 270 to 360,
cosu ...
9 8cosx = 4.64
5 Here is the graph of y = cosx for a Use cos1 on your calculator to find one
0 < x < 360. value of x.
y b Use your answer to part a and your sketch
1 from Q6 to solve 8cosx = 4.64 for values
of x in the interval 0 to 720.
0 10 Solve the equation 7cosu = 4.3 for values of
180 360 x
u in the interval 0 to 720.
21

a Use the graph to find 13.4 The tangent function


i cos90 ii cos240.
b Describe the symmetry of the curve.
1 Find the value of tanu in each diagram.
c Copy and complete y
a
i cos0 = cos 1.0
ii cos30 = cos 0.8
iii cos150 = cos P 0.6
0.4
6 The graph of y = cosx repeats every 360 in 0.2

both directions. 1 0 x
a Sketch the graph of y = cosx for 20.2
0 < x < 720. 20.4
b Use your graph to find 20.6
20.8
i cos420 ii cos600.
1__ 21.0
c The exact value of cos45 is ___ .

2
b y
Write down the exact value of 1.0
i cos405 ii cos675. 0.8
P 0.6
Q6a hint Include x-values 0, 90, 180,
0.4
270, 360, ... , 720 and y values 1, 0.5, 0,
0.2
0.5, 1.
1 0 x
20.2
7 Use your sketch from Q6 to find four values 20.4
of x such that __ 20.6
1__
___ 3
___ 20.8
a cosx =
b
cosx = .

2 2 21.0
Check your answers using a calculator.

R = Reasoning P = Problem-solving 13.4 The tangent function 113


c y 5 Here is the graph of y = tanx for
1.0 0 < x < 360.
0.8
y
0.6
P 10
0.4
0.2 8

6
1 0 x
20.2 4
20.4 2
20.6 0
20.8 90 180 270 360 x
22
21.0
21.2 24
26
y 28
d
1.4
210
1.2
1.0
0.8 a How often does the graph repeat?
0.6 b Use the graph to estimate the value of
0.4 i tan70
0.2 ii tan150.
1 0 x c Describe the symmetry of the curve.
20.2 d Copy and complete, inserting numbers
20.4 greater than 180.
20.6
i tan45 = tan
P 20.8
ii tan80 = tan
21.0
iii tan135 = tan

1__
2 tan30 = ___ 6 a Sketch the graph of y = tanx for

3 0 < u < 540.
1
Use the diagram 3 b Use your sketch to find
to find a reflex i tan495 ii tan420.
angle u such 30 1
0 c The exact value of tan30 is ___
__ .
1
3
that tanu = ___
__ .

3 Write down the exact value of
i tan390 ii tan150.
d Explain how you worked out your answers
to part c.
3 tan45 = 1
a Find a reflex angle u such that tanu = 1. 7 a Write down four values of x such that
1
b Find an obtuse angle such that tanu = 1. tanx = ___
__ .

3
b Write down four values of x such that
4 As u increases from 0 to 90, tanu increases 1
from 0 to infinity. tanx = ___
__ .
3
Copy and complete these statements in the c Check your answers using a calculator.
same way.
a As u decreases from 180 to 90, 8 4tanx = 9
tanu ... a Use tan1 on your calculator to find one
b As u increases from 180 to 270, value of x.
tanu ... b Use your answer to part a and your sketch
c As u decreases from 360 to 270, from Q6 to solve 4 tanx = 9 for values of x
tanu ... in the interval 0 to 540.

114 R = Reasoning P = Problem-solving 13.4 The tangent function


9 Solve the equation 8 tanu = 7 for values of u 5 Exam-style question
in the interval 0 to 720.
Q

10 Exam-style question
P R
a Sketch the graph of y = tanx in the
100 12 cm
interval 0 to 720. __
b Given that__tan60 = 3solve the O
equation 3tanx = 3 in the
PQR is an arc of a circle centre O with
interval 0 to 720. (4 marks)
radius 12cm.
PR is a chord of the circle.
Exam hint Angle POR = 100.
You are expected to use your sketch from Calculate the area of the shaded region.
part a to solve the equation in part b. Give your answer correct to
3 significant figures. (5 marks)

13.5 C
 alculating areas 6 P a Calculate angle AOB.
and the sine rule Give your answer correct to
1 decimal place.
1 a Write h, the perpendicular height of the A
triangle, in terms of a and u.
15 cm
24 cm
a O
h

b
B
b Write a formula in terms of a and u to
calculate the area of the triangle. b Work out the area of the shaded segment.
Give your answer correct to 3 significant
2 Find the area of each triangle.
figures.
a b
3.5 cm 122 4.8 cm
6 cm Q6a hint Split the triangle into two right-
62 angled triangles.
9 cm O

3 The area of triangle ABC is 52.96cm2.


Work out the length of AB.
A B
A

7 P In the diagram, O is the centre of


thecircle.
37 Work out the area of the shaded segment.
C 16 cm B Give your answer correct to 3significant
4 a Find the area of figures.
A B
triangle AOB in
70 m
this circle.
55 8 cm
b Find the area of 45 m
the sector AOB. O
c Find the area of O
the shaded
segment of the
circle.

R = Reasoning P = Problem-solving 13.5 Calculating areas and the sine rule 115
8 Find the length of the side Example 12 In triangle XYZ, XY = 15cm, YZ = 13.5cm and
labelled x in each diagram. angle YXZ = 57.
Give your answers correct to Work out the size of angle XZY.
3significant figures. There are two possible answers.
Give each of them correct to 1 decimal place.
a b x cm
28
x cm 7 cm
76 11.5 cm
13.6 The cosine rule and
49 65 2D trigonometric
c d problems
21
24 mm 32.8 m 33
1 Find the length of the side Example
41
marked with a letter in each
x mm 82
diagram.
xm
Give your answers correct to
3 significant figures.
9 Find the size of angle u in each diagram.
Give your answers correct to 1 decimal place. a b 8.4 cm b cm
4 cm 21
a b 109 a cm
12 cm 6.4 cm 56 12.7 cm
9 cm
14.7 cm
65 5 cm
c c mm d 112 7.87 m
c d
13.1 mm 4.29 m
15.1 m 8.45 m 6.86 m 15.3 mm 142 dm
13.2 m

59 81 2 Calculate the angles marked with letters in


these triangles.
10 a Work out the length of BD. Give your Give your answers correct to 1 decimal place.
answer correct to 3 significant figures.
a b
B 7 cm 8 cm 18.2 cm
16.3 cm 8.6 cm
C a b
48 13.3 cm
10 cm
c 8.3 cm d 5.91 m
101 84 c
A 11.5 cm d
D 4.5 cm 9.6 cm 7.86 m
b Work out the size of angle BCD. Give your 7.23 m
answer correct to 1 decimal place.

11 In triangle ABC, AB = 12cm,


3 In the diagram, O is the centre of the circle of
BC = 7cm and angle
B radius 8cm.
BAC = 35.
AB is a chord of length 11cm.
Work out the size
of angle ACB. Work out the size of angle AOB.
12 cm
The diagram 7 cm 7 cm A
shows that 8 cm
there are two 35 11 cm
possible A C C O
triangles.
Hence there are two possible answers. B
Give both, correct to 1 decimal place.

116 R = Reasoning P = Problem-solving 13.5 Calculating areas and the sine rule
4 R a Work out the B
15 cm C 8 Exam-style question
length of BD. A
Give your 13 cm 16 cm ABC is a triangle.
answer correct 38 AB = 16.4cm
to 3 significant A 11 cm D BC = 10.9cm 36 16.4 cm
figures. Angle CAB = 36.
b Work out the size of angle CBD. Give your Work out the area of B
answer correct to 1 decimal place. triangle ABC. 10.9 cm
c Work out the area of quadrilateral ABCD. C
Give your answer correct to 3 significant Give your answer correct to
figures. 3 significant figures. (6 marks)
5 R The diagram shows B
the positions of three
railway stations, A, B
and C.
N
33 km
14 km
13.7 S
 olving problems
Calculate the bearing C in3D
of C from A. 32
25 km 1 R ABCDEFGH is a cuboid. Example
H G
A
E F
6 R A ship leaves port P and sails for 40km on 9 cm
a bearing of 041. It then sails a further 31km
on a bearing of 126 to reach port R. D C
8 cm
N A 15 cm B
a Calculate the length of diagonals
i EG ii AG
Q 126
N iii CF iv CE.
31 km b Find the angle between the
40 km
diagonal BH and the plane ABCD.
41 R c Find the angle between the
diagonal CE and the plane EFGH.
P d Find the angle between the
diagonal AG and the plane ADEH.
a What is the direct distance between P
andR? Q1 hint Sketch separate triangles using
b What is the bearing of R from P? information from the cuboid.

Q6 hint The north lines are parallel. Use this 2 P In the diagram, ABCDEF is a prism.
to find an angle inside the triangle.
The cross-section is a right-angled triangle.
All of the other faces are rectangles.
7 Find the size of each lettered angle or length.
B
a b 11.7 cm
a 3.2 cm
b
A
14 cm 10.3 cm
53 6 cm F
11 cm C
c 17 cm E 16 cm
103
9 cm
D
c
d 142 9 cm Calculate the angle that the diagonal AC
15 cm makes with the plane CDEF.

R = Reasoning P = Problem-solving 13.7 Solving problems in 3D 117


3 R In the diagram, ABCD is a tetrahedron. 6 P ABCDE is a pyramid with a rectangular base.
A AB = AC = AD = AE = 35cm
A
29
16 cm
62 D 35 cm

E
70
B C B
13 cm D
14 cm 22 cm
a Work out the length of AC. C
b Work out the length of CD. Calculate the size of angle BAD correct to the
c Given that BD = 17cm, calculate angle BCD. nearest degree.

4 R ABCDE is a square-based pyramid.


The base BCDE lies in a horizontal plane. 13.8 Transforming
AB = AC = AD = AE = 18cm
trigonometric graphs 1
AM is perpendicular to the base.
A 1 Here is the graph of y = cosx for
180 < x < 180.
y
18 cm
E 1

D E
D 0.5
M
B
15 cm C F
2180 290 0 90 180 x
C
a Calculate the length of 20.5
B G
i BD ii BM iii AM.
b Calculate the angle that AD makes with 21
A H
the base, correct to the nearest degree.
c Calculate the angle between AM and the a Copy the table.
face ABC, correct to the nearest degree. i Write in the values of x and cosx at
points A to H on the graph.
5 Exam-style question ii For each x-value, write the value of cosx.
The diagram shows a A x cosx cosx
square-based
A 180 1 1
pyramid ABCDE.
Each triangular face B 120 0.5 0.5
is an isosceles 16 cm C
triangle. E b Sketch the graph of y = cosx for
B 180 < x < 180.
D c Describe how the graph of y = cosx is
7 cm
7 cm
C transformed to give the graph of y = cosx.
a Calculate the length of the diagonal BD. 2 a Use your table from Q1. Add a column for
Give your answer correct to 3 significant cos(x). Find the cosine values from the
figures. graph to fill in the cos(x) column.
b Calculate the area of triangle ABD. b Sketch the graph of y = cos(x) for
Give your answer correct to 180 < x < 180.
3 significant figures. (6 marks)
c What do you notice?

118 R = Reasoning P = Problem-solving 13.7 Solving problems in 3D


3 Here is the graph of y = tanx for 10 Exam-style question
180 < x < 180.
Here is a sketch of the graph of y = cosx.
y y
B
6
5
A
4
x
3
2
C
1
Write down the coordinates of each of the
2180 290 0 90 180 x
21 labelled points.  (3 marks)
22
23
24 13.9 Transforming
25 trigonometric graphs 2
26
1 a Copy the graph of y = cosx for
Sketch the graph of y = tan(x) for 180 < x < 180 from Q1 in lesson 13.8.
180 < x < 180. b Add 1 to the y-coordinate at each of the
labelled points.
4 a Look at your graph of y = cos(x) for c Draw the cosine graph that passes through
180 < x < 180 in Q2. the new points. Label it y = cosx + 1.
What transformations will turn the graph of d Describe the transformation from the
y = cos(x) into the graph of y = cos(x)? graph of y = cosx to this graph.
b Sketch the graph of y = cos(x). e Now subtract 1 from the y-coordinate at each
c Can the transformation in part a be of the labelled points on the original graph.
described in any other way? f Draw the cosine graph that passes through
the new points.
5 a Sketch the graph of y = sinx for g Describe the transformation from the
180 < x < 180. graph of y = cosx to this graph.
b Sketch the graph of y = sin(x). h Write down the equation of the graph.
2 Write down the equation of each graph.
6 Explain why the graph of y = sin(x) is the
same as the graph of y = sinx. Q2 hint First decide whether it is a sin, cos
or tan graph.
7 a Describe the transformation that maps
a y
thegraph of y = tanx to the graph of
1.5
y = tan(x).
1
b Sketch the graph of y = tanx and the graph
0.5
of y = tan(x) for the interval 0 to 360.
0 90 180 270 360 x
8 a Describe the transformation that maps the 20.5
graph of y = sinx to the graph of y = sinx.
y
b Sketch the graphs of y = sinx and b
3
y = sinx for the interval 180 to 180.
2
9 a Describe the transformation that maps 1
thegraph of y = tanx to the graph of 0 90 180 270 360 x
y = tan(x). 21
b Sketch the graphs of y = tanx and 22
y = tan(x) for the interval 180 to 180. 23

R = Reasoning P = Problem-solving 13.9 Transforming trigonometric graphs 2 119


c y b y
1
0 90 180 270 360 x

21

0 90 180 270 360 x


22

21
3 Here is the graph of y = sinx for 0 < x < 360.
y c
1 y 5 0.866 y
4
3
2
0 90 180 270 360 x 1

290 0 90 180 270 360 x


21
21
22
23
a Copy and complete this table of values for 24
sin(x + 60).
x 0 30 60 90 7 a Sketch the graph of y = cosx for
sin(x + 60) sin60 = 0 < x < 360.
b Sketch the graph of y = sin(x + 60) b Copy and complete the table of values for
y = 2cosx.
c Describe the transformation that takes
the graph of y = sinx to the graph of x 0 30 60 90 120
__
y = sin(x + 60). 3
cosx 1 ___ 0.5
2
4 Describe the transformation of the graph of __
y = sinx to make the graph with equation 2cosx 2 3
a y = sin(x + 30)
c On the same axes, sketch the graph of
b y = sin(x + 50)
y = 2cosx.
c y = sin(x 60)
8 Sketch the graphs of these functions for
5 Describe the transformation of the graph of 0 < x < 360.
y = tanx to make the graph with equation a y = 2cosx
a y = tan(x + 60) b y = 0.5tanx
b y = tan(x + 20) c y = 3sinx
c y = tan(x 30).
9 a Copy your sketch graph of y = cosx for
6 Match each graph with one of these equations. 0 < x < 360 from Q7.
A y = tan(x + 30) B y = sin(x 45) b Copy and complete the table of values for
y = cos(2x).
C y = cos(x 45)
a y x 0 30 60 90 120
1 cos(2x)
c Sketch the graph of y = cos(2x) on the
same axes.
0 90 180 270 360 x 10 Sketch the graphs of these functions for
0 < x < 360.
21 a y = sin2x b y = cos3x
c y = tan4x

120 R = Reasoning P = Problem-solving 13.9 Transforming trigonometric graphs 2


11 Exam-style question 2 R Triangle XYZ has angles 96, 39 and 45.
Can you tell if triangle XYZ is congruent to
The diagram shows part of a sketch of the
triangle ABC in the diagram below?
curve y = cosx
Explain your answer.
y
B
1
96
B
O 90 180 270 360 x
45 39
21 A C
A

a Write down the coordinates of the 3 Fiona invests 1250 at 4.4% per annum
points A and B. (2 marks) compound interest. Ian invests 900 at 5%
Here is a sketch of the curve y = asinbx + per annum compound interest.
c, for 0 < x < 360. After 2 years, what is the difference between
the amounts in the two accounts?
y
2 4 R A town roundabout is being replanted.
1 Theroundabout has a diameter of 8m and
has been divided into 6 equal sectors.
O 90 180 270 360 x
21 a What is the area of each sector?
Give your answer to 2 d.p.
22
23
One sector is split into three sections
A, B and C, as shown in the diagram.
24

b Find the values of a, b and c. (3 marks)

A B C

13 Problem-solving
Solve problems using these strategies where
appropriate:
Section A extends from the centre to the
Use pictures or lists midpoint of the radii and will be planted with
Use smaller numbers tall grasses.
Use bar models Section B will be planted with flowers.
Use x for the unknown Section C is a segment and will be covered in
Use a flow diagram slate.
Use arrow diagrams b What is the area of section A + section B?
Give your answer to 2 d.p.
Use geometric sketches.
c One bag of slate covers 1m2. If only one
1 A box of pens contains 12 black, 9 red, bag is available, will there be enough slate
5 green and 20 blue. to cover the whole of section C?
A pen is picked at random. 5 Omar needs to make a triangular cover to fit
What is the probability of over his deck. He knows two lengths of the
a picking red or green triangle and one of the angles.
b picking blue or green Use the sine
c picking red or black or green rule to help 4.34 m 123
d not picking green? Omar find y x
the other two 8.46 m
angles to 1 d.p.

R = Reasoning P = Problem-solving 13 Problem-solving 121


6 R In a woodcraft shop a crate is used to 8 A scientist stands on the roof of a building
store offcut pieces of dowel. The offcuts are 100ft above the ground, to the nearest foot.
all different lengths but cannot stick out The scientist fires a zip line down from the
above the top of the crate as another crate roof at an angle of 62, to the nearest degree.
will be stored on top. a How long does the zip line need to be to
What is the longest piece of dowel that can exactly reach the ground, for the upper
be placed in the crate withoutit sticking out and for the lower bound of the angle?
over the top? Give your answers to 2 d.p.
Look at the diagram to help you. b What is the difference between the
Work out your answer to 1d.p. upper bound length and the lower bound
A B length?
E 9 A cruise ship visits two islands (Q and R)
F 48 cm before returning to port (P).
D N
C
H 42 cm
66 cm G

Q6 hint Use Pythagoras theorem to find DG. N


153 Q
50.4
7 Exam-style question miles
57.8 miles
Match each of the sketch graphs below to
one of these equations.
P
y = x2
55.3 miles R
y = sinx
y = x3 + 1 What is the bearing from island Q to island R?
y = x3 + x Give your answer to the nearest whole degree.
y = x2 2 10 A circular stage has a special floor covering.
y = cosx One segment has been damaged and needs
y to be replaced. To compare quotes for
a
replacing the shaded segment, the stage
owners need to find its area.
A
O x

y 11.8 m
b

O
B 7.4 m
O x

c y O is the centre point of the circle.


Chord AB = 11.8m
The radius of the circle = 7.4m
O x a Find the angle AOB.
b Find the area of sector AOB.
 (3 marks) c Find the area of the damaged segment.
Give all your answers to 1 d.p.

122 R = Reasoning P = Problem-solving 13 Problem-solving


14 FURTHER STATISTICS
5 R Dr Hyden wants to collect some data on
14.1 Sampling the lengths of appointments at the local
health centre.
1 R Muhammad wants to find out about the One day there are 500 patients, of whom 210
films people watch. are female.
He wonders whether to ask 50 people coming a Dr Hyden wants a sample of 10% of
out of a cinema. thepatients.
a Is this sample likely to be representative of How many patients should she ask?
the population? Explain. b Describe how Dr Hyden could select a
b Hamish suggests that he pick 50 people at simple random sample.
random from the telephone book. c She decides to ask 25 males picked at
Is this sample likely to be representative of random and 25 females picked at random.
the population? Explain. i What percentage of the males are
selected?
2 R Explain whether each of these samples is ii What percentage of the females are
biased. selected?
a A supermarket wants to find out what
customers think of their product range. 6 The table shows the numbers of students
They ask the first 30 customers through studying different languages.
the tills on a Monday morning. Language Number of students
b A sample of voters is asked what they French 120
think of the current government.
They are chosen randomly from the German 140
electoral roll. Spanish 90
c A science teacher asks the students in his Mandarin 110
lunchtime science club what they think of Latin 40
science.
a Habiba chooses a sample of 50 students.
d A company wants to know whether their
Show that this is 10% of the total number
product is being well advertised. They
of students.
carry out a street survey in 30 towns
across the UK. b Work out 10% of each subject group.
c Show that taking 10% of each subject
3 Elizabeth wants to generate 20 random group gives a sample of 50 students in
numbers. total.
She rolls an ordinary dice twice to give her a
2-digit random number. 7 A library wants to find out what users think
a Use this method to generate 5 random of their new lending service.
numbers. There are 870 women and 630 men
b Will all possible 2-digit numbers be registered with the library.
included using this method? a Explain why a stratified sample should
beused.
4 A head teacher wants to select Example b The librarian wants to survey 10% of
20 students randomly from a theusers.
list of 100 students. How many women and how many men
She uses a calculator to should be asked?
generate random numbers c The librarian decides to ask 100 users.
between 1 and 100. How many of each gender should he ask?
Describe in words how she could use these
numbers to select the students.

R = Reasoning P = Problem-solving 14.1 Sampling 123


8 Exam-style question 2 This frequency table gives the heights of
50seedlings.
A gym has 2000 members.
Draw a cumulative frequency table for this data.
Each member has one of three types of
membership: Bronze, Silver or Gold. Height, h (cm) Frequency
The table shows the number of members in 0.5 , h < 0.7 2
each group. 0.7 , h < 0.9 5
Type of Number of 0.9 , h < 1.1 18
membership members 1.1 , h < 1.3 12
Bronze 450 1.3 , h < 1.5 9
Silver 980 1.5 , h < 1.7 4
Gold 570
3 Exam-style question
The manager wants to carry out a survey on
a stratified sample of 200 of these members. This table gives the weights of Example
30 new-born babies.
Find the number of members from each
group that should be in the sample.(3 marks) Weight, w (kg) Frequency
2.7 , w < 3.0 1
Q8 hint Show all your calculations clearly. 3.0 , w < 3.3 0
You could add columns to the table. 3.3 , w < 3.6 5
3.6 , w < 3.9 7
9 P In a colony of birds, 50 are caught and
tagged. A month later another 50 birds are 3.9 , w < 4.2 12
caught. Two of them are tagged birds. 4.2 , w < 4.5 5
a What fraction of the birds in the second a Draw a cumulative frequency
sample are tagged birds? diagram. (2 marks)
b Assume the fraction of tagged birds in the b Use the diagram to find an estimate
second sample is the same as the fraction for the median weight. (1 mark)
of tagged birds in the colony. c Estimate the range. (2 marks)
Estimate how many birds are in the colony.
10 P A scientists captures and tags 4 a Draw a cumulative frequency diagram for
100penguins from a colony of penguins. the seedlings data in Q2.
There are approximately 1200 penguins in b Find an estimate for the median height of
the colony. The scientist returns a month the seedlings.
later and captures 60 penguins. 5 The times taken by 50 students to complete
How many would he expect to find tagged? their maths homework are shown in the table.
Time, m (minutes) Frequency
14.2 Cumulative frequency 10 , m < 15 1
15 , m < 20 7
1 The frequency table shows the ages of 20 , m < 25 11
90customers in a spa. Copy and complete 25 , m < 30 12
the cumulative frequency table. 30 , m < 35 12
Cumulative 35 , m < 40 7
Age, y (years) Frequency
frequency a Draw a cumulative frequency diagram.
20 , y < 30 6 6 b Estimate the median time taken.
30 , y < 40 18 6 + 18 = c Estimate the lower quartile of the time
40 , y < 50 22 taken.
d Estimate the upper quartile.
50 , y < 60 34
e Use your answers to parts c and d to work
60 , y < 70 10 out an estimate for the interquartile range.

124 R = Reasoning P = Problem-solving 14.1 Sampling


6 R The table shows the masses of 100 cakes
in grams. 14.3 Box plots
Mass, m (grams) Frequency
1 Draw a box plot for this data Example
900 , m < 950 2
on the ages of customers.
950 , m < 1000 37
1000 , m < 1050 32
1050 , m < 1100 22
1100 , m < 1150 5
Minimum LQ Median UQ Maximum
1150 , m < 1200 2
19 25 33 45 61
a Draw a cumulative frequency diagram.
b Estimate the median, quartiles and 2 R This data shows the lengths of time, in
interquartile range. minutes, it took 11 people to walk 1 mile.
c Estimate how many cakes weigh less than 9, 11, 12, 12, 15, 15, 16, 16, 17, 18, 18
1075g. a Write down the median time taken.
d Copy and complete. b Find the upper and lower quartiles.
90 cakes are estimated to weigh less than c Draw a box plot for the data.
______ grams. 3 R This stem-and-leaf diagram shows the
amounts of time 25 people spent in a coffee
7 Exam-style question
shop.
The table below shows information about 1 8 8 9
the heights of 60 students.
2 0 3 3 4 5 7 8
Height, x (cm) Number of students 3 5 5 7 9 9 9 9 9
140 , x < 150 4 4 0 2 2 5 7 8
150 , x < 160 5 5 2
160 , x < 170 16
Key: 1 | 8 represents 18 minutes
170 , x < 180 27
a What was the shortest length of time a
180 , x < 190 5 customer spent in the coffee shop?
190 , x < 200 3 b What is the median length of time?
a Draw a cumulative frequency table. c Find the lower and upper quartiles of the
 (1 mark) length of time.
b Draw a cumulative frequency graph. d Work out the interquartile range.
 (2 marks) e Draw a box plot.
c Use your graph to find an estimate of
the median height. (1 mark) 4 The results in two different maths tests are
shown in the comparative box plots.
d Use your graph to find an estimate for
the number of students who are less than Results
185cm tall. (2 marks) Test A

Exam hint
For parts c and d draw lines on the graph Test B
with a ruler to show how you got your
answers.
50 60 70 80 90
Marks
a Which test has the higher median?
b Work out the interquartile range for each
test.
c Work out the range for each test.

R = Reasoning P = Problem-solving 14.3 Box plots 125


5 R Summary statistics on the ages of people Harry weighed 60 tomatoes from group A.
living in two different blocks of flats are given The cumulative frequency graph shows
in this table. some information about these weights.
Block A Block B
60
Minimum 3 17
LQ 18 35
50
Median 29 37

Cumulative frequency
UQ 42 42 40
Maximum 73 91
a Draw comparative box plots for the two 30
blocks of flats.
b Compare the blocks of flats. 20

Q5b hint Compare the medians, 10


interquartile ranges and ranges.

0
6 R The cumulative frequency graph gives 140 150 160 170 180 190
information about how long the batteries last Weight (g)
in two different types of mobile phones.
a Use the graph to find an estimate for the
Battery life median weight. (1 mark)
Type B The 60 tomatoes from group A had
100 minimum weight of 153 grams
90 maximum weight of 186 grams.
Cumulative frequency

Type A
80 b Use this information and the cumulative
70 frequency graph to draw a box plot for
60 the 60 tomatoes from group A.
50 Use a scale like this:
40
Group
30
A
20
10 140 150 160 170 180 190
0 Weight (g)
0 1 2 3 4 5 6 7 8 9 10 11 12  (3 marks)
Time (hours) Harry weighed 60 tomatoes from group B.
a Use the graph to find the median and He drew this box plot for his results.
quartiles for each type of phone.
b Draw comparative box plots for the two Group
B
phones.
c Compare the two phones. 140 150 160 170 180 190
Weight (g)
7 Exam-style question
c Compare the distribution of the weights
Harry grows tomatoes.
of the tomatoes from group A with
This year he put his tomato plants into two the distribution of the weights of the
groups, group A and group B. tomatoes from group B. (2 marks)
Harry gave fertiliser to the tomato plants in June 2012, Q15, 1MA0/1H
group A.
He did not give fertiliser to the tomato
plants in group B. Q7c hint Be sure to put your comparisons in
the context of the question.

126 R = Reasoning P = Problem-solving 14.3 Box plots


a Work out an estimate for the mean
14.4 Drawing histograms mark of Year 11. (4 marks)
b Draw a histogram for the information
1 The table shows the ages of 60 patients. given in the table. (3 marks)

Class Frequency
Age, a (years) Frequency
width density Q4 hint Use the midpoint of each class
__
3 interval.
0 , a < 10 3 10 10 =0.3
10 , a < 20 14
20 , a < 40 17 14.5 I nterpreting
40 , a < 60 19
histograms
60 , a < 80 7
a Work out each class width. 1 R The histogram shows the heights of a
b Work out the frequency density for each sample of students.
class. Student heights
2 This table shows the lengths Example
3

Frequency density
of time that 100 people spent
watching TV one evening.
2

1
Time, t (hours) Frequency
0 < t < 0.5 5 0
150 160 170 180
0.5 , t < 1 35
Height (cm)
1,t<2 56
a How many students were between
2,t<3 4
150and 155cm tall?
Draw a histogram for this data. b How many students were between
3 This table contains data on the weights of 170and 180cm tall?
65women. c How many students were measured intotal?
Weight, w (kg) Frequency 2 R The histogram shows the distances an
40 , w < 45 2 audience at a theatre had travelled to the
production.
45 , w < 55 17
Distance travelled
55 , w < 65 31
65 , w < 70 11 6
Frequency density

70 , w < 90 4
4
Draw a histogram for this data.
4 Exam-style question 2
Mrs Morris records the results in the Year
11 maths exam. 0
0 10 20 30 40 50
Mark, m (%) Frequency Distance (miles)
0 , m < 40 2 a How many of the audience travelled less
40 , m < 50 4 than 10 miles?
50 , m < 60 12 b Estimate how many of the audience
60 , m < 80 63 travelled less than 15 miles.
c Estimate how many people travelled
80 , m <100 9
between 15 and 40 miles.

R = Reasoning P = Problem-solving 14.5 Interpreting histograms 127


3 Exam-style question a Draw a grouped frequency table for the data.
b Work out an estimate for the mean time
The incomplete table and histogram give
taken.
some information about the distances
people travel to work. c How many athletes took longer than
11.5seconds to run 100m?

5 Work out an estimate for the Example


median of the data in Q2.
Frequency density

6 R The histogram shows the weights of all


the babies born in one day in a maternity
hospital.
0 10 20 30 40 50
Weights of babies
Distance in miles

a Use the information in the histogram to


complete the frequency table below.
Distance (d) in miles Frequency
0,d<5 140
5 , d < 10
10 , d < 20
20 , d < 35 120
35 , d < 50 30
(2 marks) 0
b Complete the histogram. (2 marks) 2.8 3 3.2 3.4 3.6 3.8 4 4.2
Weight (kg)

Exam hint a Five babies weighed between 2.8 and 3kg.


Draw the bars on the histogram neatly Work out the frequency density for that
with a ruler. Show your working to class.
calculate the frequencies. b How many babies were born in total?
c Work out an estimate for the median weight.
4 P The histogram shows the times taken by a d Draw a frequency diagram for the data in
number of athletes to run 100m. the histogram.
100 m times
e Work out an estimate for the mean weight
from your frequency table.
40 f How many of the babies were heavier than
the mean weight?
Frequency density

30
Q6b hint Label the frequency density scale.

20

10

0
10 11 12 13 14
Time (seconds)

128 R = Reasoning P = Problem-solving 14.5 Interpreting histograms


7 P The histogram shows the heights of some 4 R Ten male and ten female swimmers
sunflowers. compete in a race.
Heights of sunflowers The times, in minutes, to complete the swim
are recorded.
Frequency density

0.4 Males: 32, 34, 35, 37, 39, 41, 42, 44, 49, 63
Females: 30, 41, 43, 44, 47, 49, 50, 52, 52, 53
0.2 a Explain which of the median and
interquartile range or mean and range
0 should be used to compare the data.
50 100 150 200 250 300 b Compare the times for males and females.
Height (cm)
5 P The table shows how far employees at
a How many sunflowers are there in total? Gatwick Airport travel to work.
b Work out an estimate of the median
height. Length of journey,
Frequency
x (miles)
c Estimate how many sunflowers are taller
than 110cm. 0 , x < 10 1
10 , x < 20 16
20 , x < 30 25
14.6 Comparing and 30 , x < 40 15
describing populations 40 , x < 50 3
a Draw a cumulative frequency diagram.
1 Every day one week, a passenger group b Find the median and interquartile range.
recorded how late two trains were, in minutes. The box plot shows far employees at
Train A 3 7 5 6 8 Heathrow Airport travel to work.
Train B 1 9 2 8 3 Length of journey

Compare the lengths of time the trains were


late by.

2 The ages, in years, of the first five clients at 0 10 20 30 40 50


two different physiotherapists are recorded. Distance (miles)
Back Health 54 49 39 44 63 71 65 c Compare the lengths of the employees
Joints Ease 33 39 45 57 62 21 25 journeys to work at the two airports.
a Write down the median age of the clients Q5c hint Compare the medians and
for each physiotherapist. interquartile ranges.
b Work out the interquartile range of
theage of the clients for each 6 P This back-to-back stem-and-leaf diagram
physiotherapist. shows the weights of 20 members at two
c Compare the ages of the clients at the two different slimming clubs.
physiotherapists. Weight Loss World Slimmers Hut

3 The lengths of time, in minutes, students 7 5 7 0 1 1 3 5


spent playing computer games one day were 9 7 6 6 3 8 0 5 5 7 9 9
recorded. 9 8 6 6 5 5 9 1 1 6 8 9 9 9
17, 19, 23, 25, 28, 31, 34, 38, 40, 41, 120 9 9 6 6 2 0 0 10 2 4
a Work out the mean length of time. Key: 5 | 7 Key: 7 | 0
b Work out the median length of time. represents 75 kg represents 70 kg
c Work out the range and interquartile
range. Compare the weights of the members at the
two different clubs.

R = Reasoning P = Problem-solving 14.6 Comparing and describing populations 129


7 P The cumulative frequency graph shows 1 Gary conducted a survey asking people of
the ages of children at two different different ages if they shopped more often at
nurseries. Shop A or Shop B. This back-to-back
Ages of children
stem-and-leaf diagram shows the results.
Shop A Shop B
40
Cumulative frequency

9 1 8 8 9 9
Nursery A 8 7 5 2 2 4 4 6 7
30 7 6 5 3 3 2 3 1 2 2 5
9 6 6 5 1 0 0 4 0
20 Nursery B 8 8 8 6 3 2 5 1
Key: Key:
10 9 | 1 represents 19 years 1 | 8 represents 18 years
Compare the data by finding the median
0 ages and interquartile ranges.
0 1 2 3 4 5
Age (years) 2 Plane A took 11 hours to travel 6022 miles.
Plane B travelled at the same average speed on
Compare the two sets of data. a journey of 3904 miles. Round your answers to
the questions to the nearest whole number.
8 Exam-style question
a What was the average speed of the planes?
A survey was carried out on the speeds at b About how long did Plane Bs journey take?
which cars travelled past a primary school.
3 R Judy is conducting a survey of her
Speed, m Cumulative companys employees. There are 250
(miles per hour) frequency employees. Judy will survey a random sample
0 , m < 10 15 of 10 employees. She uses a random number
10 , m < 20 74 table to work out whom to ask.
20 , m < 30 45 9845679256938259793198984293232456
30 , m < 40 18 a Explain one way Judy can generate
40 , m < 50 3 10random numbers.
b Would you consider Judys survey a
A similar survey outside another school gave a
good-sized sample? Explain.
median of 27 and an interquartile range of 15.
Compare the speeds at which cars travelled 4 R Ian buys two similar pictures.
outside the two schools. (2 marks) The larger picture has a height of 23cm.
Thesmaller picture has a length of 7.5cm
and a height of 5.75cm.
Q8 hint Draw a cumulative frequency What is the length of the larger picture?
diagram for the results given in the table.
5 The frequency table shows the test scores for
Year 9 students in a recent science test.
Test scores, t (%) Frequency
14 Problem-solving 0 , t < 25 13
25 , t < 50 18
Solve problems using these strategies where
appropriate: 50 , t < 75 34
75 , t < 100 20
Use pictures or lists
Use smaller numbers Topass the test, students need to score
60% or higher. Draw a cumulative frequency
Use bar models
diagram and estimate how many students
Use x for the unknown passed the test.
Use a flow diagram
Use arrow diagrams Q5 hint Use the data to draw a cumulative
Use geometric sketches. frequency table.

130 R = Reasoning P = Problem-solving 14.6 Comparing and describing populations


6 Exam-style question 8 R Mark says the interquartile range for
the data in this stem-and-leaf diagram is 8
Chilli plants are watered with water or with because 12 4 = 8.
a water and fertiliser mixture.
Is he correct? Explain how you know.
The box plot shows data about their heights.
2 9 9
3 1 3 3 5 7 9
Water
4 0 2 4 6
5 0 5 6
Water and
fertiliser Key: 2 | 9 represents 29

0 10 20 30 40 50 60 9 A market researcher wants to find out what


Height (cm) customers of different ages think about a
new product. The table shows the numbers
a Write down the median height of chilli
of customers who bought the product in the
plants watered with the water and
last month. She wants to look at a stratified
fertiliser mixture. (1 mark)
sample of 20% of the customers.
b Give two comparisons between the
heights of chilli plants watered only with Age (years), a Number of customers
water and chilli plants watered with the 18 , a < 25 160
water and fertiliser mixture. (3 marks) 26 , a < 39 125
40 , a < 60 85
7 This table shows summary statistics from a 61 , a 70
data set of the heights of sunflowers. a How many customers will be surveyed in
Minimum LQ Median UQ Maximum total?
b How many more customers in the
6ft 7ft 8ft 10ft 12ft
18 , a < 25 age range will be surveyed
a Draw a box plot for the data. than in the 40 , a < 60 age range?
b Suppose some more sunflowers were c Explain why a company might want to use
measured and the lower quartile a stratified sample like this.
changedto 5ft.
10 Solve the equation 4sinx = 3 for
Describe how your box plot would change. 0 , x , 720.
Describe what would stay the same. Give your answers correct to 1 d.p.

15 EQUATIONS AND GRAPHS


y
15.1 Solving simultaneous C 5
B

equations graphically 4
A 3
1 R a Match the equations to the three lines 2
A, B and C shown opposite. 1
i x + y = 4 ii y = x + 2 iii 3y + x = 6
22 21O 1 2 3 4 5 6 x
b Hence write down the solutions to these 21
pairs of simultaneous equations.
i x + y = 4 ii x + y = 4
y = x + 2 3y + x = 6
iii 3y + x = 6
y=x+2

R = Reasoning P = Problem-solving 15.1 Solving simultaneous equations graphically 131


2 Solve the pairs of simultaneous equations by 7 a On a suitable grid draw the graph of
drawing the graphs. x2 + y2 = 36
a y = 4x 1 b On the same grid draw the graph of
y = 4x + 1 y=x+6
b 5y = x 8 c What are the coordinates of the points at
x + 5y = 12 which the graphs intersect?
c 0 = 2y x + 4
4y = 13 x 8 Use a graphical method to find an estimate
for the solution to the simultaneous
d 3x 2y = 11 equations
x + y = 4
x2 + y2 = 16
e Show that solving the equations xy=3
algebraically gives the same solutions to
the equations in part d.

3 Tim buys 3 rollerball pens and 5 biros for 9.75. 15.2 Representing
In the same shop, Jenny buys 2 rollerball inequalities graphically
pens and 2 biros for 5.50.
Write a pair of simultaneous equations and 1 a Write down the inequalities represented
solve them graphically to find the cost of by the shaded regions.
a one rollerball pen y
i
b one biro. 5
4
4 R Two mobile phone companies offer the 3
following prices.
2
Fones4U MobilesRUs 1
No monthly contract Monthly line rental 24
Calls cost 0.30 per minute Calls cost 0.06 per minute 23 22 21O 1 2 3 x
21
a For each company form an equation to
ii y
calculate the monthly cost, with y = total
3
monthly cost and x = minutes of calls.
2
b Use a graphical method to work out how
1
many minutes of calls are used if the two
companies charge the same. 24 23 22 21O 1 2 x
21
5 Use a graphical method to Example 22
find an approximate solution 23
to the pair of simultaneous
equations iii y
y + x2 = 1 5
y+ 2 = x 4
3
Q5 hint Start by rearranging the equations, 2
then plot the graphs. 1

6 R a Solve this pair of simultaneous equations 23 22 21O 1 2 3 x


21
i graphically 22
ii algebraically to 2 decimal places.
4x 3y = 7
2x2 y = 3
b Which method gives the more accurate
solution? Explain.

132 R = Reasoning P = Problem-solving 15.1 Solving simultaneous equations graphically


b On a suitable coordinate grid, shade the 6 R The diagrams show a shaded region
region of points whose coordinates satisfy bounded by three lines.
i x > 1 ii y , 4 For each diagram
iii 0 . y iv y + 1 , x < 2 i write down the equations of the lines
v 1 , y and y . 1 vi 1.5 < x < 3.5 ii write down the three inequalities satisfied
vii 4.5 , y < 2 by the coordinates of the points in this
region.
2 a Draw a coordinate grid with 4 to 4 on
a y
both axes.
5
b Draw the graph of y = 3x 1
4
c Does the point (4, 2) satisfy the inequality
y < 3x 1? 3
d Shade the region of points that satisfy 2
y < 3x 1 1

Q2c hint At the point (4, 2), y = and 22 21 1 2 3 4 5 6 7 x


21
3x 1=
22
23
3 Draw a coordinate grid with 5 to 5 on both
axes.
b y
Shade the regions that satisfy
5
a y , x b y < 2x 3
4
c y . _12x + 2 d
y > 4 3x
3
4 R x and y are integers. Example 2
On a coordinate grid with 1
5 to 5 on both axes, mark
on all the points with 26 25 24 23 22 21O 1 2 3 4 x
21
integer coordinates which
22
satisfy all three inequalities:
y + x < 3 x . 2 y > 1
c y
5 Exam-style question 5
4
The lines y = x + 3 and x + y = 7 are
drawn on the grid. 3
y 2
10 1
9
y5x13
8 25 24 23 22 21O 1 2 3 4 5 x
7 21
6 22
5 23
4
3
24
2
1
7 P How many points with integer coordinates
2524232221O 1 2 3 4 5 6 7 8 9 10 x satisfy these inequalities?
21
22
x1y57
y + 2x , 4 y . 2x 1x . 1
23
8 a Draw the graph of y = 4 x2 for values of
On the grid, mark with a cross (X) each of x from 4 to +4.
the points with integer coordinates that are b Draw the line y = x + 1 on the same axes.
in the region defined by y , x + 3, x + y < 7
c Shade the region that satisfies y , 4 x2
and y . 2 (3 marks)
and y . x + 1

R = Reasoning P = Problem-solving 15.2 Representing inequalities graphically 133


9 This is the graph of y = 5 + 3x 2x2 y
y 10
8 9
7 8
6 7
5 6
4
y 5 x2 2 6x 1 8
5
3 4
2 3
1 2
24 23 22 21O 1 2 3 4 x 1
21
22 21O 1 2 3 4 5 6 7 8 9 10 x
21
22
a For what integer values of x is the graph
below the x-axis?
d What are the coordinates of the turning
b For what integer values of x point?
is 5 + 3x 2x2 , 0?
2 a Plot the graph of y = x2 + 4x 21
Q9 hint Write your answer as two b Use your graph to work out the roots of
inequalities: x , and x .
the equation y = x2 + 4x 21
10 R a Sketch the graph of y = 12 2x 2x2, c Where does the graph intersect the y-axis?
marking clearly the points where the d Does the graph have a maximum or
graph intersects the x-axis. minimum point?
b From the graph identify the values of x for e What are the coordinates of the turning
which 0 > 12 2x 2x2 point?
Give your answer using set notation. 3 a Solve the equations
11 P By sketching the graph of y = + 6x 9,
3x2 i 0 = x2 + 2x 8
find the values of x which satisfy ii 0 = x2 7x + 6
0 > 3x2 + 6x 9 b Find the value of y when x = 0 for the
Give your answer using set notation. equations
i y = x2 + 2x 8
12 R a Sketch the graph of y = x2 + x 12 ii y = x2 7x + 6
b Hence find the values of x which satisfy
the inequality 12 > x2 + x 4 R Match the graphs to their equations,
Give your answer using set notation. explaining your reasoning.
a y = x2 + x 20 b y = x2 x 20
13 R Sketch graphs to find the values of x
c y = x2 + 8x 15 d y = 2x2 + 13x 15
which satisfy these inequalities.
i y
Give your answers using set notation. 8
a x2 + 3x . 4 b x + 14 < 3x2 c x2 , 25 6
4
15.3 Graphs of quadratic 2

functions 21O
22
1 2 3 4 5 6 7 8 9 10 x

24
1 Look at the graph of y = x2 6x + 8
26
a Use the graph to find the roots.
28
b Where does the graph intersect the yaxis?
210
c Is the turning point a maximum or a
212
minimum?

134 R = Reasoning P = Problem-solving 15.2 Representing inequalities graphically


ii y 7 Use the method in Q6 to sketch these graphs.
2 a y = x2 + x 56
b y = x2 5x + 36
21O 1 2 3 4 5 6 7 8 9 10 x
22 c y = 3x2 6x + 9
24 d y = 4x2 9
26
8 Exam-style question
28
a Solve the equation x2 + 6x 5 = 0
iii y  (2 marks)
5 b On a suitable coordinate grid,
sketch the graph of y = x2 + 6x 5,
21028 26 24 22O 2 4 6 8 10 x marking clearly the coordinates of the
25
points of intersection with the axes
210 and the coordinates of the turning
215 point. (4 marks)
220
9 a Write down the coordinates of the turning
iv y
point of the graph of y = (x 2)2 7
5
b Substitute y = 0 into the equation
21028 26 24 22O 2 4 6 8 10 x y = (x 2)2 7 and hence find the
25 coordinates of the roots, giving your
210 answers in surd form.
215
10 Find the roots of these equations given in
220 completed square form, giving your answers
in surd form where appropriate.
5 R For each quadratic function, a y = (x 1)2 4
Example
work out the coordinates of b y = 2(x + 3)2 32
the turning point and state c y = 5(x 2)2 15
whether it is a maximum or
11 By writing the equations in completed square
a minimum.
form, calculate the roots of the equations.
a y = x2 4x + 7
Give your answers in surd form.
b y = x2 2x 5
a y = x2 + 6x 7
c y = x2 12x + 19 d y = 2x2 + 16x + 40
b y = 2x2 12x + 4
e y = 3x2 18x + 33 f y = 4x2 2x + 5
c y = 3x2 8x + 1
6 a Factorise the expression x2 + 2x 35 12 R Give three reasons why the graph shown is
b Hence write down the coordinates of the not y = 3x2 + 12x + 36
roots of y = x2 + 2x 35 y
c Where does the graph of y = x2 + 2x 35 5
cross the y-axis?
d Write x2 + 2x 35 in completed square 21028 26 24 22O 2 4 6 8 10 x
25
form. 210
e Hence write down the coordinates of the 215
turning point of y = x2 + 2x 35 220
f Is the turning point a maximum or a 225
minimum? 230
Explain your answer. 235
g Using your answers to parts a to f, sketch 240
the graph of y = x2 + 2x 35
245
Q6b hint Solve x2 + 2x 35 = 0 250

R = Reasoning P = Problem-solving 15.3 Graphs of quadratic functions 135


13 P Find the equation of this graph. iv y
y (4, 6) 4
3
2
O x 1

(0, 210) 24 23 22 21O 1 2 3 4 5 6 x


21
22
23
15.4 Solving quadratic 24
equations graphically 25

1 R Match each graph to its equation. 2 a Copy and complete the table of values for
Hence estimate the roots of the equation. y = 2x2 + 5x 4
a y = x2 5x + 2 b y = 2x2 2x 3 x 2 1 0 1 2 3
c y = x 3x + 1 d
2 y = 3x2 + 5x + 3 y 7
i y
4 b Plot the graph of y = 2x2 + 5x 4 on a
3
suitable grid.
2 c From the graph estimate the roots of the
1
equation y = 2x2 + 5x 4
3 a Plot the graphs of the following functions.
24 23 22 21O 1 2 3 4 5 6 x Hence estimate the roots.
21
22 i y = x2 + 3x 6 ii y = x2 + 4x + 3
23 iii y = 3x2 + 2x 4 iv y = 2x2 5x + 5
24 b Use the quadratic formula to find the
25 roots of the equations in part a to
26 3significant figures.
Check your answers to part a.
ii y
4
4 Exam-style question
3 a Complete the table of values for
2 y = 2x2 + 9x 3
1 x 1 0 1 2 3 4 5
25 24 23 22 21O 1 2 3 4 x y
21
 (2 marks)
22
b Plot the graph of y = 2x + 9x 3
2
23
 (2 marks)
24
c Use your graph to estimate the roots of
the equation y = 2x2 + 9x 3(3 marks)
iii y
6 d Write the expression 2x2 + 9x 3 in
5 the form a(x + b)2 + c. (2 marks)
4
3 5 R For each function
2
i find the coordinates of the turning point
ii find the y-intercept
1
iii sketch the graph.
24 23 22 21O 1 2 3 4 x a y = x2 + 3x + 4
21
b y = x2 5x 5
22
c y = 2x2 + 5x 6

136 R = Reasoning P = Problem-solving 15.3 Graphs of quadratic functions


6 R Dan is sketching the graph of c Write the set of values of x that satisfy
y = 2(x + 4)2 1 x2 + 2x 3 . 0 (where the curve is above
He is finding it difficult to identify the roots the x-axis).
of the equation. Explain why. d Write the set of values of x that satisfy
x2 + 2x 3 , 0 (where the curve is below
7 R By completing the square, decide whether the x-axis).
these quadratic equations have
no roots 12 Find the set of values that satisfy each
two roots inequality.
one repeated root.
a x2 + 3x 4 . 0
a y = x2 + 7x + 12 b y = x2 + 5x 4
b x2 + 2x 15 , 0
c y = x2 8x 13 d y = 2x2 + 8x + 8
c x2 + 6x + 8 , 0
e y = x + 3x 3 f
2 y = 4x2 11x + 7
d x2 + 12x + 35 . 0
g y = 3x2 10x 9 h y = 9x2 6x 1
e x2 5x + 6 . 0
8 Exam-style question f x2 7x + 6 , 0

a By completing the square, find the roots


of the equation y = x2 9x 5, giving
your answers in surd form. (3 marks) 15.5 Graphs of cubic
b Show algebraically that y = x2 + 3x + 8 functions
has no real roots. (3 marks)
1 Expand the expression (x + 1)(x2 + 2x + 3)
9 Write an equation for each graph.
a y b y 2 Copy and complete to expand the expression
(x + 1)(x + 3)(x 2) = (x2 + x + )(x 2)
= x3 + x2 x
23 O 3 x
O 2 6 x
3 Expand the expressions
a (x2 + 2x + 1)(x + 1) b (x + 1)(x + 2)(x + 3)
c y c (x + 2)(x + 4)(x + 2) d (x 2)(x + 5)(x 4)
e (x + 4)(x 1)(x 5) f x(x 6)(x 3)
g (x + 2)2(x 2) h (x 3)3

4 Here is the graph of y = x3 + 3x2 13x 15


O 3 x
y
Q9 hint Write y = (x a)(x b) and expand. 25
20
15
10 Use the iterative formula Example
and the starting point given 10
to find one root for each 5
quadratic equation.
Give your answers correct to 25 24 23 22 21O 1 2 3 4 5 x
25
5 decimal places. ______ 210
a y = x2 7x 5 x = ______
5 + 7x x0 = 7 215
b y = x 2x 6 x = 2x + 6
2 x0 = 3.5 220
4
c y = x2 x 4 x=__
x

+1 x0 = 2.5 225

11 a Solve the quadratic equation


x2 + 2x 3 = 0 a What are the roots of the equation?
b Sketch the graph of y = x2 + 2x 3 b Where does the graph cross the y-axis?

R = Reasoning P = Problem-solving 15.5 Graphs of cubic functions 137


5 a What are the roots of the Example iv y
equation 5
y = (x + 2)(x + 4)(x 6)? 4
b Where does the graph of 3
y = (x + 2)(x + 4)(x 6) 2
cross the y-axis? 1
c Sketch the graph of
y = (x + 2)(x + 4)(x 6) 25 24 23 22 21O 1 2 3 4 5 x
21
22
6 R Match each graph to its equation. 23
a y = (x 1)(x + 2)(x 3) 24
b y = (x + 2)(x 3)2 25
c y = (x 3)(x + 3)(x 4)
v y
d y = x(x 2)2
25
e y = (x + 1)(2 x)(x + 5) 20
f y = x2(x 2) 15
i y 10
20 5
15
10 28 26 24 22O 2 4 6 8 x
25
5
vi y
26 25 24 23 22 21O 1 2 3 4 x 45
25
40
210
35
215
30
220
25
225
20
15
ii y
5 10
4 5
3
28 26 24 22O 2 4 6 8 x
2 25
1
7 How many solutions does each of these cubic
23 22 21O 1 2 3 4 x equations have?
21
22 a y = (x 2)(x + 7)(x 8)
23 b y = (x 1)3
24 c y = x(x 2)(x + 5)
25 d y = x(x 4)2
e y = (x2 3x + 4)(x + 5)
iii y f y = (x + 6)(x 2)(3 x)
15
10 8 Sketch the graphs, marking clearly the points
5 of intersection with the x- and y-axes.
a y = (x 4)(x 2)(x + 1)
28 26 24 22O 2 4 6 8 x
25 b y = x(x 1)2
210 c y = (2 x)(x + 2)(x 5)
215 d y = (x 3)2(x + 4)
e y = (x 4)3

138 R = Reasoning P = Problem-solving 15.5 Graphs of cubic functions


9 Exam-style question
15 Problem-solving
Sketch the graph of y = x(x + 4)2
on a suitable coordinate grid marking
Solve problems using these Example
clearly the points of intersection
strategies where appropriate:
with the axes. (3 marks)
Use pictures
10 P The graph has equation Use smaller numbers
y = x3 + ax2 + bx + c Use bar models
y Use x for the unknown
4
Use a flow diagram
2
Use arrow diagrams
24 23 22 21O 1 2 3 4 x Use geometric sketches
22
Use graphs.
24
26 1 A group of Explorer Scouts are on an
28 orienteering expedition.
210 They start at point A and are told to travel
one mile on a bearing of 050 to point B
Work out the values of a, b and c. where they find their next instruction.
11 P A graph has equation y = x3 + ax2 + bx + c Point C is 3 miles from point B on a bearing
of 135.
It crosses the x-axis at x = 4, x = 1 and x = 3.
a Using 3cm for every mile, draw a diagram
Without drawing the graph, work out the
to show where the group needs to go to
values of a, b and c.
visit both points B and C.
12 Use an iterative formula y b What is the bearing of C from A?
to find the one root of
x3 3x2 2 = 0 to 4 d.p. 2 R Look at the graph and the shaded region
The first steps have been bounded by the three lines.
done for you: Write down the three inequalities satisfied by
O 2 4 x
x3 = 3x 2+2
______ the coordinates of the points in this region.
22
x = _______
3
3x2 + 2
y
xn+1 = 3xn + 2
3
2 6
x0 = _______ 5
x1 = 3x02 + 2 4
3
=
_______
3
x2 = 3x1 + 2
3 2 =
2
13 Use an iterative formula to find the negative 1
root of the equation x3 2x2 7x + 5 = 0
to 5 d.p. 24 23 22 210 1 2 3 4 x
y 21
22
23
24
25
26
24 22 O 2 4 x

R = Reasoning P = Problem-solving 15 Problem-solving 139


3 R This frequency table shows different 7 R Joshua is working on a woodwork project.
lengths of material in a craft shop. Sean has measured the height and the
Length, l (cm) Frequency opposite angle of the piece of wood Joshua
needs to cut and has rounded them to the
10 , l < 15 9 nearest cm and the nearest degree.
15 , l < 20 6
20 , l < 25 12
25 , l < 30 16
21 cm a
Which of the following statements are true
about this data?
58
A The higher class boundary of each class
is30. Joshua wants more precision and so cuts the
B The range is 16 9 = 7 wood to the upper bounds so that he can
C The lower quartile is in the 15 , l < 20 sand it down if he needs to.
class. a What is the upper bound of length a, to
D The median is in the 15 , l < 20 class. the nearest mm?
E The upper quartile is in the 25 , l < 30 b What is the difference between the upper
class. bound and the lower bound of the length?

4 Show the area formed by these inequalities 8 Work out the minimum value of n2 6n + 14.
on a grid.
x , 4 y . 2 y , 2x + 1 9 R Naomi is making a maths display showing
different graphs.
5 R Kevin has just fitted a new granite area The labels for the graphs have got mixed up.
around his sink. Match the labels to graphs X, Y and Z.
(2x 1 3) in linear graph cubic graph
(x 1 2) in quadratic graph
(x 1 2) in (x 1 1) in x1y55 y 5 2x 1 2

y 5 x3 1 3x2 2 6x 2 8

a Find an expression in x for the area of the 8 5 4y 1 x y 5 x2 1 x 2 5


granite.
b The actual area of the granite is Graph X
484 inches square. y
Find the dimensions of the sink. 12
10
6 Exam-style question 8
a Complete the table of values for 6
y = x2 2x. 4
2
x 2 1 0 1 2 3 4
y 25 24 23 22 21O 1 2 3 x
22
(2 marks)
24
b Draw the graph of y = x2 2x for
values of x from 2 to 4. (2 marks) 26
c Solve x 2x 2 = 1.
2 (2 marks) 28
June 2013, Q15, 1MA0/2H 210
212

140 R = Reasoning P = Problem-solving 15 Problem-solving


Graph Y Graph Z
y B y A
8 5
7 4
6 C 3
5 2
4 1
3
21O 1 2 3 4 5 6 7 8 9 10 x
2 21
1

25 24 23 22 21O 1 2 3 4 5 x 10 Solve these equations simultaneously.


21
y 3 = 4x
22
x2 + 2y = 6
23
24
25
26

16 CIRCLE THEOREMS
2 R Kate says that Q cannot be the centre of
16.1 Radii and chords the circle. Explain why she is wrong.

1 R Each diagram shows a circle with centre O.


Work out the size of each angle marked with
a letter.
Q
a a b
100
b 35 140
O 132 O C B A

3 R O is centre of the circle. AB and CD are


straight lines through the centre. Prove that
triangles ACO and BDO are congruent.
c c A
C
110
O O

D
B

R = Reasoning P = Problem-solving 16.1 Radii and chords 141


4 R O is the centre of a circle. Example 2 R The diagrams all show Example
The length of chord AB is 10cm. circles, centre O.
OM is perpendicular to AB. Work out the size of each
OM is 12cm. angle marked with a letter.
a Work out the length of AM. Give reasons for your
State any circle theorems answers.
that you use. a b
b What is the length of the radius of the 25
circle? 120 b
O a O
Q4b hint Use Pythagoras theorem.

5 R O is the centre of a circle.


OA = 20cm and AB = 24cm. c d
M is the midpoint of AB. 54
c
O d
O
O
20 cm 120

e f
A M B g
f
Work out the length of OM. O
O
e
6 R O is the centre of a circle. The radius of
the circle is 15cm. The distance from O to the
midpoint of chord AB is 12cm.
Work out the length of chord AB.
30
7 R O is the centre of a
circle. M is the midpoint
of chord AB.
O
Angle OAB = 45. 3 Exam-style question
a What is angle AMO? 45 Diagram NOT
b Work out angle AOM. D A accurately drawn
A M B
c Work out angle AOB.
d Which of the triangles AMO, BOM and ABO
are similar? O
34 C

16.2 Tangents B
A, B and D are points on the circumference
1 P AB is a tangent to the circle with centre O. ofa circle, centre O.
AOB = 40. Work out the size of angle ABO.
BOD is a diameter of the circle.
A BC and AC are tangents to the circle.
B Angle OCB = 34.
40 Work out the size of angle DOA. (3 marks)
O March 2013, Q19, 1MA0/1H

142 R = Reasoning P = Problem-solving 16.1 Radii and chords


4 P A and B are points on the circumference of 3 R The diagrams show circles, centre O.
a circle, centre O. Workout the size of each angle marked with
TA and TB are tangents to the circle. a letter. Give reasons for your answers.
A a b
a
40
55
O T b b
O O

Show that triangles OAT and OBT are


c d
congruent. 4c
220
5 R OA is the radius of a circle with a diameter
O O d
of 20cm. AT is a tangent to the circle joining
point A to point T. AT = 24cm. c
Calculate the distance from T to the centre of
the circle.
State any circle theorems that you use.
e f
Q5 hint Draw a diagram and mark the values 300 100
on it.
e O O f

16.3 Angles in circles 1


1 R The diagrams show circles, Example 4 Jarod says the size of a
centre O. angle a is 80.
a Henrysays the size of O
angle a is 100.
35 Show that Henry is 200
O correct.
a b
5 Find the size of each angle marked witha
112 letter.
The centres of the circles are marked O.
c O
d a b
b
100 36
b
O c O 54 O
120
a
a
Work out the size of each angle marked with
a letter.
Give reasons for your answers. 6 Shumi says the size of
angle a is 10.
Charlie says the size O
2 R AB is the diameter of a circle. 20
C is a point on the circumference of the circle. of angle a is 40. a
Use the circle theorem from Q1 to prove that Show that Charlie
ABC is a right-angled triangle. is correct.

R = Reasoning P = Problem-solving 16.3 Angles in circles 1 143


7 Exam-style question Angle AOB = x. Write down the size of angles
iAEB iiADB iiiACB
D
Diagram NOT in terms of x.
accurately drawn b What can you say about angles in the
same segment?
O
3 R In each diagram, O is the centre of the circle.
y C
a b
35
a 64
B
A
O d O c
A, B, C and D are points on the
circumference of a circle, centre O. b
Angle AOC = y.
Find the size of angle ABC in terms of y. c d
Give a reason for each stage of your e f
working. (4 marks)
November 2013, Q22, 1MA0/1H O O
112 175

Exam hint g h
Each reason given must be a statement
of a mathematical rule and not just the Work out the size of each angle marked
calculations you have done. witha letter.
Give reasons for each step in your working.
4 R In each diagram, O is the centre of the circle.
16.4 Angles in circles 2 What is the size of angles a, b and c in each
diagram?
1 O is the centre of a circle.
b a b
a Work out the size of a
each angle marked O b 210 a
with a letter. a c O
O b
120
80
c c

b Work out the size of 5 R ABCD is a cyclic quadrilateral.


a
angles a, b and c in O is the centre of the circle.
terms of x. O B
x c b

O
2 P a Look at the diagram. x A
D
C
D
C
a In terms of x
E O i what is the reflex angle AOC
ii what is the obtuse angle ADC
iii what is the acute angle ABC?
B b Find the sum of the acute angle ABC and
A the obtuse angle ADC.

144 R = Reasoning P = Problem-solving 16.3 Angles in circles 1


6 R In each diagram, O is the centre of the 9 R O is the centre of a C
circle. circle. AT is a tangent
a to the circle. O B
a Copy the diagram. 36
80 T
a b Copy the working
O 110 and complete the A
reasons.
b
c Angle OAT = 90 because the angle
between the radius and the is 90
b
94 Angle OAB = 90 36 = 54
132
a OA = OB because
b
O Angle OAB = angle OBA because the
c
angles of an isosceles triangle are
d Angle AOB = 180 (2 54) = 72 because
angles in a triangle to
Work out the size of each angle marked
Angle ACB = 72 2 = 36 because the
witha letter.
angle at the is half the angle at
Give reasons for each step in your working.
the
7 Prove that the exterior angle of a cyclic
10 RRepeat Q9 but this time with angle
quadrilateral is equal to the opposite interior
BAT = 50.
angle.
What is the size of angle ACB?
a
11 R Prove that angle ACB = angle BAT.
C
b c
x
O B

a Write down the size of angle c in terms T


ofb.
A
b Write down the size of angle a in terms
ofb.
c Are a and c equal? 12 Exam-style question
M, N and P are three points on the
8 R Work out the size of each angle marked circumference of a circle, centre O.
with a letter. A M B
Give reasons for each step in your working. y
a b
a
37 a O N
b 105 72 x
c
P
42
d c
b
The straight line AMB is the tangent to the
c d circle at M.
g Angle MPN = x and angle BMN = y.
35 b a
c 37 i Prove that x = y(5 marks)
O e
c d
a e
d f h
b Exam hint
48
Draw on the diagram the angle MON.
28

R = Reasoning P = Problem-solving 16.4 Angles in circles 2 145


3 R Work out the size of each angle marked
16.5 Applying circle with a letter.
theorems Give reasons for each step in your working.
a A
1 R In each diagram, O is the centre of the C
a
circle.
64 T
a
g b
j B B A
i O
b
h 32
b c C T
55 A B D O 35
40 k d c
O C
c C
E e
F 66
O 49 D B A T
I i
H G 4 R O is the centre of the circle.
DAT and BT are tangents to the circle.
Angle CAD = 62 and angle ATB = 20.
Work out the size of each angle marked with
a letter. D A
Give reasons for each step in your working. 62

2 R In each diagram, AT is a tangent to the O 20 T


circle. C
a T B
38
Work out the size of
A a angle CAO b angle AOB
c angle AOC d angle COB
a e angle CBO.
Give reasons for each step in your working.
b
5 Exam-style question
T c
35
B, C and D are points on the circumference
b 52 of a circle, centre O.
ABE and ADF are tangents to the circle.
A d
F

C
c A D
T
62 g
f O

66 70
79
e A B E

Angle DAB = 66
Work out the size of each angle marked with Angle CBE = 70
a letter.
Work out the size of angle ODC. (3 marks)
Give reasons for each step in your working.

146 R = Reasoning P = Problem-solving 16.5 Applying circle theorems


6 P Find the equation of Example 4 Exam-style question
the tangent to the circle
A and B are points on the circumference
x2 + y2 = 100 at the
of a circle, centre O.
point B (6, 8).
BC is a tangent to the circle.
AOC is a straight line.
7 P Find the equation of the tangent to the Angle ABO = 35
circle x2 + y2 = 25 at the point C (3, 4). Diagram NOT
8 P Find the equation of the tangent to the accurately drawn
A
circle x2 + y2 = 225 at the point D (9, 12). O
35
9 P Find the equation of the tangent to the
circle x2 + y2 = 676 at the point E (10, 24). x
B C

16 Problem-solving Work out the size of the angle marked x.


Give reasons for your answer. (5 marks)
Solve problems using these Example
strategies where appropriate: 5 R The diagram shows a circle, centre O.
Use pictures or lists CB and CD are tangents to the circle.
Use smaller numbers Find angle g. Explain your reasoning.
Use bar models B
Use x for the unknown
Use a flow diagram
Use arrow diagrams A 56 O g C
Use geometric sketches
Use graphs
D
Use logical reasoning.
1 R 3 pairs of football boots cost 54. 6 R Pria says that just from knowing the angle
8 football shirts cost 152. GHO, she can work out all the angles inside
There are no special offers or discounts if you triangles GFO and HGO.
buy more shirts or pairs of boots. F
a A football coach has 200. How much
change will she get if she buys 11 pairs of O
football boots? G
b Another coach has 250. Does he have
enough money to buy 7 shirts and 7 pairs
H
of boots?
a Prove that Pria is correct.
2 A park fountain is in the centre (F) of a
circular pool with a radius of 2.5m. b If angle GHO = 41, work out the angles in
triangles GFO and HGO.
A row of lights form a chord across the pool
(GH). The shortest distance from the lights to 7 R Work out the sizes of angles a, b and c.
the fountain is 1.8m. Give reasons for your answers.
a Draw a diagram to show this information.
b
b What is the length of the row of lights?
Give your answer to 2 d.p. 87 c
A B
3 R O is the centre of a
circle. CB and CD are a
tangents to the circle. 98 O x C
108
Find angle x. Explain
your reasoning. E D

R = Reasoning P = Problem-solving 16 Problem-solving 147


8 R 3 avocados and 2 red peppers were 10 R O is the centre of the circle. A, C and E are
bought for 2.95. all points on the circumference.
2 avocados and 4 red peppers were bought BCD is a tangent touching the circle at point C.
for 3.70. DEF is a tangent touching the circle at point E.
How much is C
B
a 1 avocado b 1 red pepper? D
z x
9 This table shows the lengths of 40 caterpillars. 132
O
Length, l (mm) Frequency
A y
8 , l < 10 7
58 E
10 , l < 12 10
12 , l < 14 14 F
14 , l < 16 9 Work out the sizes of angles x, y and z.
a Find the frequency density for each class. Give reasons for any statements you make.
Explain how you found the answers.
b Draw a histogram for this data.
Whydoyou need to work out the
frequency densities to do this?

17 MORE ALGEBRA
6 Make q the subject of the formula
17.1 Rearranging formulae T = pq + pr qs
7 R5ab 7 = c + 2ab
1 Make c the subject of the formula
a Make a the subject.
E = mc2
b Make b the subject.
a the subject of the formula
2 Make_____
8 R L = 4x + xy 5
P=a+b
Jordan rearranges the formula to make x the
3 Make x the subject of each Example subject.
L4x+5
formula.
__ __ His answer is x=_________
y

x
__
a A=4
y b
1
__
K=t
x a Explain why this cannot be the correct
___ answer.
4x
c T= ___

2a
R=2(x+y) 2
d b What mistake has Jordan made?
c Work out the correct answer.
4 In each formula change the subject to the
letter given in brackets. 9 Make x the subject of the formula
p 3x 3 2x1
a V=__d 3 (d) b
V=____
(x) m=______
x
6 2
__


___ 1
3
c p=6x (x) d __
t=
3 Q9 hint First multiply both sides by x.
p (p)
5 Make x the subject of the Example 10 Exam-style question
formula A = 2x + px
Make y the subject of the formula
y
c=_____

 (4 marks)
5y

148 R = Reasoning P = Problem-solving 16 Problem-solving


9 Write as a single fraction in its simplest form.
17.2 Algebraic fractions x+1 _____ x+4 x+6
x2 _____
a _____
+ _____
b

3 5 3 7
1 Write as a single fraction in its simplest form. x6 ______ 3x+2
The first one has been started for you. c _____
+

5 2
a a _____ aa
a __ __=
=
5 2 52 10 Exam-style question
3c 2d 3 8 Write as a single fraction in its simplest form
b ______ c ___
____

5 9 2k 11k 4x+5 ______2x1
______

+
 (3 marks)
2 Write as a single fraction in its simplest form. 7 2
The first one has been started for you.
1 2 1 __ 1
5y 2 __________
3x 3 ____ 35y
3 x 11 Make a the subject of the formula __ ab=1.
a ____y


=

=
9x y3

9 x
9y 4 21y 2 The working has been started for you.
10x 2 _____ 12x 6 _____
b _____
5 c _____
__ 1 1
3y 20x 35y 20x 3
7 a __=1
b
3 Write as a single fraction in its simplest form. __ 1 1
a=1+__
2 __ 3 1 b
a __
aa b xy 2 ___
xy __ 1 1
a = __
+__ =
5x 2 15x 4 x x+5 b
c ____6 _____
__
_____
d

4y 8y 3 4 12
12 Scientists use the lens formula to solve
xy 2 problems involving light.
Q3b hint Write xy2 as ____

1 1 1 __ 1
The lens formula is __ =__ +
f u v
4 Write as a single fraction in its Example where f = focal length, u = object distance
simplest form. and v = image distance.
4y y 2y y Make f the subject of the formula.
a ___+__ b
___
+__
9 3 3 5
5y y
c ___
__
3 7 17.3 Simplifying algebraic
5 Write down the LCM of fractions
a 2x and 3x b 2x and 6x
c 5x and 6x d 7x and 3x 1 Simplify
1 1 a x+2
6 a Write ___ and ___as equivalent fractions a ___
b _______

3x 2x ab 5(x+2)
with denominator the LCM of 3x and 2x. x+5 (x+4)(x3)
1 1 c _______
___________
d
b Simplify ___+___ (x + 5)
2 (x3)(x8)
3x 2x
x(x5) x(x + 6) 2
7 Write as a single fraction in its simplest form. e ___________
________
f
2
1 1 1 1 7 1 (x5)(x+2) x (x+6)
a ___+___
b ___
___
____
___
c
7x 2x 3x 5x 10x 3x
Q1e hint You can only cancel whole brackets.
8 a Copy and complete.
x+5 ________ (x+5) _______ x+ 2 a Factorise x2 + 4x
_____
=
=

3 23 6 b Use your answer to part a to simplify
b Copy and complete. x 2+4x
_______
x7 ________ (x7) _______ x

_____
= =
x+4
2 32 6
c Use your answers to parts a and b to 3 Simplify fully
x+5 _____ x7 x2 5x 6x 2+9x 4x+8
workout _____ +
a ______ x
________
b
________
c
3 2 2x+3 3x 2+6x

R = Reasoning P = Problem-solving 17.3 Simplifying algebraic fractions 149


x 23x
4 R Simplify _______
x2 3


17.4 M
 ore algebraic
Ayesha says, (x 3) is a factor of the fractions
numerator and the denominator.
a Is Ayesha correct? Explain. 1 Write as a single fraction in its simplest form.
b Can the fraction be simplified? x+5 x+3 x4
Explain your answer. a (x 2) 2 _____
b __________
x2 x4 x+6
5 Simplify fully x+2 ______ 3 4 10
c _____

d _____
______

4(x+1) x 2+2x8 5 4x+8 x3 3x9
a __________
2
__________
b


x +7x+6 3(x2) 4x+12 _____ x+3 (x + 6) 2 _____
x+6
e _______


_______
f
x2 x+5 x+5 x1
6 Simplify fully Example
x 2+7x+12 2 a Factorise x2 16
a ___________ b Factorise x2 + 7x + 12
x 2+3x4
x 216 ___________ 3
x 27x+10 c Write _______
2
as a single
b ___________
2
6 x +7x+12
x +4x12
fraction in its simplest form.
x 216
c _______

3 Write as a single fraction in its simplest form.
(x + 4) 2
x 2+3x4 __________x 24
a __________
2
2
7 Exam-style question x 5x+6 x +5x6
x 2+12x+36 15x+20 3x 22x8
Simplify fully ____________

 (3 marks) b ___________ ___________



x 236 x +7x+10
2 x 2 x6
4 Write down the LCM of
8 Simplify fully a x and x + 1 b x + 1 and x + 4
2x 2 + x15 6x 2 + x15 c x + 5 and x + 7 d x + 2 and x 2
a ____________
_____________
b


3x +10x+3
2 3x 2+17x+20 e 2x + 1 and 2x 2
16x 21
c ____________
5 Simplify fully Example
16x 2+8x+1
1 1
a _____
+_____

9 Exam-style question x+1 x+4
2 5
3x 2+11x+10
_____________ b _____
+_____

Simplify fully
 (3 marks) x+2 x2
x 24x12
6 2 1 1
c _____
_____

______
d ______

x+3 x1 2x+2 2x+5
10 a Copy and complete.
4 x = ( ) 6 Exam-style question
b Simplify Write as a single fraction in its simplest form
4x
i _____

1
_____ 5
_____
 (3 marks)
x4 x3 x+4
16x2
ii _________
2
x x12 7 a Factorise
11 Simplify fully i 3x + 6 ii 5x + 10
25x 2 3x 227 b Write down the LCM of 3x + 6 and 5x + 10.
a _______ b
__________ 1 1
x 5x
2 x 26x+9 c Write ______ +_______

as a single fraction
3x+6 5x+10
8x 2+20x
c ____________

in its simplest form.
8x 2+18x5
12 Show that Q7b hint Look at the factorised form of each
expression:
(x 23x10)(x 22x3)(12x 2+20x) ___ 4x
________________________________

= a(x + y)b(x + y)LCM = ab(x + y)
(3x 24x15)(9x45)(x 2+3x+2) 9

150 R = Reasoning P = Problem-solving 17.3 Simplifying algebraic fractions


8 a Factorise x2 25 6 R a Work out the area of each shape. __
b Write _____
1 1
+_______

as a single fraction in Write your answers in the form a+b

3
x5 x 225 i ii 1 1 12
its simplest form. 71 3

9 Write as a single fraction in its simplest form. 42 3


1 1
a ___________ ______

2x 2+5x+3 2x+3
1 1 b Would the perimeter of each shape be
b __________ +_______

x 2+5x+4 3x+12 rational or irrational? Explain.
1 2
c ___________ +__________ 7 Rationalise the denominators.
x 2+7x+10 x 22x8
The first one has been started for you.
2 5 __ __ __ __ __
d _____ _______
4+
______
3 ___ 3 43+3
______________ 3

6x 36x 2 a __ __ = __ __ =
1 1 1



3

3
3

3
10 Write ___+_______ +__as a single fraction in __ __ __
3x 3(x+1) 9 5

2 2+

6 6

5
b ______
__ ______
c __

______
d __

its simplest form.

2

6

5

11 Show that 8 Exam-style question


___
___________ 1 1 x+9 9+48 __
2 +_______
=____________
Write _______
__ in the form a+b3,

x +9x+20 4x+16 A(x+5)(x+4) 3

and find the value of A. where a and b are integers. (3 marks)
__ __
9 R a Expand and simplify (4+7 )(47
)
17.5 Surds b Is your answer rational or irrational?
c How can you tell if your answer will be
1 Simplify___ rational or irrational?
a i ___
12 d Which of these will have rational answers
ii 75 when expanded?__ __
b Use your answers
___ ___ to part a to simplify i (5+ __3)(3__
3)
412+375 ii (5+ __
3)(5+ __
3)
iii (5+ 3)(53)
2 Simplify___ ___ Check by expanding the brackets.
a 2____
20+480 1
b 3 ___

___
1502__54 ____ e Rationalise the denominator of ______ __
5+

3
c 332+68200
Q9e hint Multiply the __numerator and
3 Factorise these expressions. ).
denominator by (53
The first one has __
been started for you.
___

a 45+9=3 +9=3( + ) 10 Rationalise the denominators. __
___
b 4+12 c
___
1550 d
___ ___
6354 Give your __
answers in the form abor
where a, b and c are rational.
abc
4 Expand and simplify 1 1 __ 2 __
a ________
b ______ ______
c
__
a
(1
3
__
+ 3) b
__ __
( 2+3)(2+ 2)

1+

5
3__

2 2+__


3
__ __ __ 5 __
______
______
3 2+
______
5
c (43)(2+3) d
(23)2 d e __
f __
__ __ 1+

7 4
3 7

5
e (4+6)2 f (5 7)2
11 a Solve x2 + 4x 3 = 0 by using the
5 Exam-style question quadratic formula.
__ b Solve the equation x2 + 6x + 3 = 0 by
Expand (32)2. __ completing the square.
Write your answer in the form a + bc,
c Solve the equation x2 10x + 5 = 0.
where a, b and c are integers. (2 marks)
Write all your answers in surd form.

R = Reasoning P = Problem-solving 17.5 Surds 151


17.6 S
 olving algebraic 17.7 Functions
fraction equations
1 a L = 2x and x = 4y. Write L in terms of y.
x
1 Solve these equations. b T=__and x=_15z. Write T in terms of z.
Give your answer as a simplified fraction. 2
c A = x2 and x = y 2. Write A in terms of y.
4 __ 3
a __x+x=14 12
2 f(x) = ___
x. Work out
5 1
b _____
_____ =3 a f(2) b f(_12) d
f(3) c f(36)
x+1 x+1
4 6
c 10=_____ _____
3 Rg(x) = 3x2. Robert says that g(4) = 144.
x2 x2
a Explain what Robert did wrong.
2 Solve these quadratic equations. b Work out g(4).
3 _____ x1 3 ______2x+5
a __
x= 2

__
b x= 4
4 h(x) = 4x2. Work out
3x+7 1 2x+1 __ 5 a h(4) b h(_12) d
h(2) c h(3)
c ______ __

=x d ______ = x

4 3
5 f(x) = x2 + 3x and g(x) = x3. Work out
Q2a hint First multiply both sides by the a f(1) + g(1) b f(3) g(2) c f(2) g(2)
LCM (2x) and simplify. Then multiply out the g(10)
bracket and solve by factorising. d _____
e
2f(16) f 2f(4) g(2)
f(2)
3 Solve this quadratic equation. Example 6 g(x) = 3x + 7. Work out the value of a when
5 2 a g(a) = 22 b g(a) = 9 c g(a) = 0
_____
+______

=1
x+3 2x1
7 f(x) = 2x2 + 1. Work out the values of a when
a f(a) = 33 b f(a) = 1
c f(a) = 11 d f(a) = 25
2 x
4 a Show that the equation _____
+______

=1
x+4 2x1 8 f(x) = x(x 2) and g(x) = (x + 1)(x + 3).
can be rearranged to give x2 x 2 = 0 Work out the values of a when
b Solve x2 x 2 = 0 a f(a) = 0 b g(a) = 0
c f(a) = 8 d g(a) = 8
5 Solve these quadratic equations.
3 4 2 4 9 f(x) = 3x + 2. Write out in full
a _____ +_____
=2 b _____
+_____
=3
x+2 x3 x+1 3x1 a f(x) + 3 b f(x) 8 c 3f(x)
3x 6 6 1 d 5f(x) e f(3x) f f(6x)
c ______
+______
=1 d _____ _____
=3
4x2 2x+6 x+2 x2
4 6 10 h(x) = 2x2 8. Write out in full
e _____
_____ =1 a h(x) 2 b 4h(x)
x3 x+1
c h(3x) d h(x)
6 Solve these quadratic equations.
Give your answers correct to 2 decimal places. 11 Rf(x) = x2 + 3 and g(x) = 3x 7. Work out
3 1 3 _____ 1 a gf(2) b gf(5) c fg(3) d fg(6)
a _____
+_____ __
=1 b xx+2 =4
x1 x+3
1 1 2 1 Q11a hint First work out f(2) and then
c _____
+_____
=3 d _____ _____ =5 substitute your answer into g(x).
x+4 x2 x+4 x+2

7 Exam-style question 12 Rf(x) = 2x + 4, g(x) = 5 x and h(x) = x2 3.


Work out
Find the exact solutions of
3
a gf(x) b fg(x) c hf(x)
x+__
x=10 (3 marks) d fh(x) e hg(x) f gh(x)

152 R = Reasoning P = Problem-solving 17.6 Solving algebraic fraction equations


13 Find the inverse of each function. Example 5 Give a counter example to prove that these
a x 2x 5 statements are not true.
x a No prime numbers are even.
b x __+3
4 b The square of a number is always greater
c x 4(x + 5) than the number itself.
d x 5(x 1) + 3 c The product of two numbers is always
14 Rf(x) = 3(x + 2) and g(x) = 3(x 2) greater than their sum.
a Find f1(x). b Find g1(x). d The sum of two square numbers is always
c Work out f1(x) + g1(x). even.
d If f1(a) + g1(a) = 1, work out the value of a. 6 R Prove that the sum of any two odd
numbers is always even.
17.8 Proof Q6 hint Let 2n be any even number.
Let 2n + 1 be any odd number.
1 Show that Example
a (x + 2)2 4x ; x2 + 4
7 R aThe nth odd number is 2n 1.
b x2 7x + 25 ; (x 5)2 + 3x Explain why the next odd number is 2n + 1.
c (x + 4)2 4 ; (x + 6)(x + 2) b Prove that the product of two consecutive
d 16 (x 1)2 ; (5 x)(3 + x) odd numbers is 1 less than a multiple of 4.
2 R a Show that (x 2)(x + 2) ; x2 4
8 R Prove that the product of any odd number
b Use your rule to work out and any even number is even.
i 48 52 ii 298 302
3 R The diagram shows a garden in the shape 9 R Given that 2(x a) = x + 8, where a is an
of a rectangle of length x + 7 and width x + 3. integer, show that x must be an even number.
x17 10 R a Work out
x12
i _13 _15 ii _15 _17 iii _12 _14
b Use your answers to part a to write down
x11 x13
the answer to _19 __ 1
11.
c Explain how you can quickly
calculate ___ 1
100 ___
1
.
102
a Write an expression for the area of the garden.
1 1
There is a patio in the centre of the garden. d i Simplify __
x_____

x+2
The patio is a rectangle of length x + 2 and
ii Explain how this proves your answer
width x + 1.
from part c.
b Write an expression for the area of the patio.
c Show that the area of the garden, 11 R Show that
excluding the patio, is 7x + 19. 1 1 A
_______ _______
=____________

x 2+4x x 22x x(x+4)(x2)
4 Exam-style question
and find the value of A.
The diagram shows a large rectangle of
length (5x 4)cm and width xcm. 12 R Prove that n2 n is even for all values of n.
A smaller rectangle of length 2xcm and
width 3cm is cut out and removed. 13 a Write an expression for the product of three
The area of the shape that is left is 40cm2. consecutive integers, n, n + 1 and n + 2.
5x 2 4 b Hence show that n3 + 3n2 + 2n is even.

14 Exam-style question
x
3 Prove algebraically that the difference
between the squares of any two consecutive
2x
odd integers is equal to 4 times the integer
Show that 5x2 10x 40 = 0. (3 marks) between them. (4 marks)

R = Reasoning P = Problem-solving 17.8 Proof 153


5 R Two photographs are mathematically
17 Problem-solving similar.
The area of the larger photograph is 260cm2.
Solve problems using these strategies where The area of the smaller photograph is 65cm2.
appropriate: If the length of the larger photograph is
20cm, what is the length of the smaller
Use pictures or lists
photograph?
Use smaller numbers
Use bar models 6 Exam-style question
Use x for the unknown
x 2+4x+3
Use a flow diagram Simplify fully __________
2 
(3 marks)
x +5x+6
Use arrow diagrams
Use geometric sketches
7 R Terri is asked to____
work out value of the
Use graphs ___ ___
+ 180
expression 20 48
, giving her
Use logical reasoning. answer in surd form. ____
1 R Anya gets a cab from the station. a Terri gets the answer 152.
The cost of the cab ride was 12.40. The cab Explain why she is not correct.
driver charged 4 for the first mile and then b Work out the correct answer.
30p for every tenth of a mile after that.
8 A rectangular hole is cut out of a rectangular
a What is the formula the cab driver used to
piece of card.
find the total (T) cost of the cab ride?
The dimensions are shown on the diagram.
Use x for one tenth of a mile.
b How far was the cab ride? (3x 2 2) cm

2 Rhian buys a house for 135000. The value (x 2 1) cm


of her house increases by 8% in the first (2x 1 1) cm
(x 1 5) cm
year and 6% in the second year, and then
decreases by 3% in the third year.
At the end of the third year, is Rhians house a Work out an expression in x for the
worth more or less than she originally paid shaded area.
for it? How much more or less? b If the shaded area is 283cm2, work out the
value of x.
3 R Clare and Emma are asked to rearrange
this formula to make d the subject: 9 R Two integers have a difference of 2.
2+9d Prove that the difference between their
b=______

squares is divisible by 4.
d
2+9d
Clares answer is d = ______

10 Rationalise the denominator and simplify
b
12 __
Emma says that Clares answer cannot be ______
correct. 3
5

__
a Why does Emma say this? Write your answer in the form a+bcwhere
b Emma correctly makes d the subject of the a, b and c are integers.
formula. What is Emmas answer?

4 Find the sizes of angles x, y and z.


x
y

z
73

112

154 R = Reasoning P = Problem-solving 17 Problem-solving


18 VECTORS AND GEOMETRIC
PROOF
18.1 Vectors and vector 7 Exam-style question

notation A is the point (2, 5) and B is the point (7, 0).



as a column vector. (1 mark)
a Write AB

1 On squared paper, draw and label these . (2 marks)
b Find the length of vector AB
vectors.
2
a a = ( ) b
3
c = ( )
b = ( ) c
1 2 8 RAB= ( ). B is the point (3, 4).
2 4 5 1
3 4 Work out the coordinates of A.
d AB = ( )
= ( ) e
CD
1 0
2 The point A is (3, 4), the Example 18.2 Vector arithmetic
point B is (2, 7) and the
point C is (2, 5). 1 R The points A, B, C and D are the vertices of
Write as column vectors a quadrilateral where A has coordinates (3, 2).
2 3 2
b
a AB c
BC AC AB= ( ), BC = ( ).
= ( ) and CD
3 1 3
3 Which of these vectors are equal? a Draw quadrilateral ABCD on squared
paper.

a b c d b Write AD as a column vector.
c What type of quadrilateral is ABCD?

d What do you notice about BC and AD
?

e f 2 R The points A, B, C and D are the vertices of


a rectangle.
4
A has coordinates (2, 1), AB= ( ) and
0
AD= ( ).
2
4 Find the magnitude of the vector AB= ( ).
0
7 3
Give your answer to 3 significant figures. a Draw rectangle ABCD on squared paper.
b Write as a column vector
5 Work out the magnitude of these vectors.
Where necessary, leave your answer as a surd. i CB

a a = ( ) b b = ( ) c c = ( )
5 3 1 ii BC
12 4 1 What do you notice?
2
= ( ) e
= (
12)
9
d AB CD c What do you notice about
9
33 i AB and DC
6 R In triangle ABC, AB= ( ) and AC
= ( ).
16
63 56 iI AD and BC
?
a Work out the length of the side AB of the 2
3 In quadrilateral ABCD, AB= ( ), BC
= ( ),
1
triangle. 3 3
2 1
b Show that triangle ABC is isosceles. ( ) and DA=
CD= ( ).
3 3
Q6 hint Draw a sketch of the triangle. What type of quadrilateral is ABCD?

R = Reasoning P = Problem-solving 18.2 Vector arithmetic 155


4 Exam-style question 9 a Find, by drawing, the sum of the vectors
a and b.
= ( )
3
P is the point (5, 6). PQ
1
a Find the coordinates of Q. (1 mark) a
R is the point (7, 4).
b
b Express PRas a column vector.(3 marks)
( )
3
RT=
5 b Copy and complete this vector addition.
c Calculate the length of PT. a + b = a+b

( 3) + (1) = ( )
Give your answer to 3 significant figures. 4 3
 (3 marks)

Q9a hint Use the triangle law of addition.


Q4 hint Sketch a diagram.
Move vector b to the end of vector a so that
the lines follow on. Draw and label the vector
5 The diagram shows the vectors a and b. a + b to complete the triangle.

a b
a
b

1
( ) and BC=
( ). Find AC
2
10 a AB= .
3 4
On squared paper draw vectors to represent
b a = ( ) and b = ( ). Find a + b.
3 2
a 2a b a c b d 3b e 2b 5 1
11 p = ( ) , q = ( ) and r = ( ).
2 1 0
6 The vectors a and b are shown on an
4 3 5
isometric grid.
a Work out
i p + q ii q + p
b Work out
a
i (p + q) + r ii p + (q + r)
b
12 p = ( ) and q = ( ) . Work out p q.
3 2
6 4

13 a = ( ), b = ( ) and c = ( ) .
Draw these vectors on an isometric grid. 2 1 4
a 2a b _
12b c
a d b 5 2 0
Write down the column vector for
4
( )
7 RAB= a a b a + b c a+b+c
6 d a b e bc
a Copy and complete to find the column

vector for 2AB.
18.3 More vector arithmetic
26) ( )

2AB= 2 ( )= (
4 24
=
6
4 1 5
b Write down the column vector for 1 RAB= ( ) and CD=
( ) , BC= ( ).
2 3 4
i 3AB ii 4AB iii _12AB

a Find the column vector for AD .
2 3
( ) and BC=
8 RAB= ( ) . C Draw a diagram to show this.
3 2 4
Write down the b Show that DB= ( ).
B 1
vector AC. 2 a = ( ) and b = ( ).
2 4
5 4
Work out the magnitude of
A
a a b 2b c a + b d ab

156 R = Reasoning P = Problem-solving 18.2 Vector arithmetic



3 In the quadrilateral ABCD, AB = a, BC
= b 7 Exam-style question

and CD = c. B
b
B a
a b C

A C A

= a and BC
ABC is a triangle with AB = b.
c a Express, in terms of a and b

i AC

D ii CA (2 marks)
Find in terms of a, b and c B
q
C

a AC
b AD
T
p
= b
4 OA
M is the midpoint of OA. A D
M
O A AC and BD are diagonals of rectangle ABCD.
AC and BD intersect at T.
b
in terms of p and q.(1 mark)
b Express DT

in terms of b.
a Write down OM
6
( )
OA= 8 RAC = a and BC
= b.
2
C
b Express as a column vector

ii OM
i AO

= b.
5 In the diagram AC= a and CM A B
M is the midpoint of CB.
in terms of a and b.
a Write AB
C _1
b = 2b?
b Where is the point S such that BS
a M
9 R ABCDEF is a regular octagon.

AB = k
A B A B

Write down in terms of a and/or b. H C



b
a CB
MA c AB
G D
6 JKLM is a parallelogram.
F E
= a and M
LM J = b.
L

a Explain why FE = k.
K
a BC = m, CD
= n and DE
= p.
b Find
M
J b i GF ii AH iii HG
c Find
a Explain why KJ = a.
b Find
i AD ii AE


i LK ii LJ ?
d What is HE

R = Reasoning P = Problem-solving 18.3 More vector arithmetic 157


2 3(y)+ ( )= ( )
10 R ABCD is a rectangle. x 2 5
M is the midpoint of DC. 5 10
Find (y).
x
AB = q and DA
= p.
q
3 e = ( ) and f = ( ).
A B 2 4
3 3
p Calculate g given that 2e g = f.

D M C 4 O is the origin (0, 0). A has coordinates (3, 6)


and B has coordinates (5, 1).
Write in terms of p and q Find as column vectors



a DC b
DM c

AM
d
BM
a OA b
AO c
OB d AB

11 R Here are five vectors.

= 2a 8b CD = a + 4b EF= 4a 16b
AB = a and BC
5 AB = b.
B b C
GH= 2a + 8b IJ = a 7b
a Three of these vectors are parallel.
a
Which three?
b Simplify
i 5p 6q + 2p 7q A D
ii 3(a 2b) + _12(3a + 4b)
in terms of a and b.
Express CA
12 R In parallelogram ABCD, AB = a and BC
= b.
M is the midpoint of AD. 6 R The points P, Q, R and S Example

B b C
have coordinates
(1, 3), (7, 5), (12, 15) and
a (24, 3) respectively.
O is the origin.
a Write down the position
A M D
vectors OP and OQ
.
Write in terms of a and b
b Write down as a column vector

a AM b
BM c
CM
i PQ ii RS
13 In triangle ABC, AB = a and AC
= b. c What do these results show about the
M is the midpoint of AB. lines PQ and RS?
N is the midpoint of AC.
B 7 Exam-style question
C P is the point (2, 9) and Q is the point (4, 7).
M
N as a column vector. (1 mark)
a Find PQ

= ( ).
A 5
R is the point such that QR
Write in terms of a and b 7
b Write down the coordinates of the
b
a BC
BM c

CN
d
MN point R. (2 marks)
X is the midpoint of PQ. O is the origin.

18.4 P
 arallel vectors and c Find OXas a column vector. (2 marks)
collinear points
Q7 hint In this type of vector question it can
1 a = ( ) , b = ( ) and a + c = b.
2 3
5 1 be helpful to draw a sketch.
Calculate c.

158 R = Reasoning P = Problem-solving 18.3 More vector arithmetic



8 R Point A has coordinates (2, 7), point B has in terms of a and b.
c Express MN
coordinates (1, 3) and point C has coordinates d Explain what the answers to parts b and c
(1, 6). show about AC and MN.

a Write ABas a column vector. 2 R In triangle ABO, A

b CD = 6AB.Find CD
. OA = a, and OB
= b. a
c Find the coordinates of D. The point X divides X
AB in the ratio 2:3 O
9 P a = ( ) and b = ( ).
1 4
3 2 Express in terms b
B
Find a vector c such that a + c is parallel to of a and b

a b. b
a AX
OX

10 R OABC is a quadrilateral in which 3 P In parallelogram ABCD, AB = a and BC
= b.
_1 B b C
OA= 2a, OB
= 2a + b and OC
= 2b.

a Find ABin terms of a and b. a M
What does this tell you about AB and OC
?

b Find BCin terms of a and b.
A D
What does this tell you about OA and BC
?
.
a Find in terms of a and/or b the vector CD
c What type of quadrilateral is OABC? M is the midpoint of CD.

.
b Find in terms of a and/or b the vector CM
11 P The points A, B and C have coordinates
(1, 5), (3, 12) and (5, 19) respectively. c Find in terms of a and/or b the vector AM.
a Find as column vectors 4 Exam-style question

i AB ii AC
OX= a + 2b and OY
= 2a + 3b
b What do these results show you about the Y
points A, B and C?
2a 1 3b
12 P The point P has coordinates (3, 2). O
The point Q has coordinates (7, 7).
The point R has coordinates (15, 17). 2a 1 2b
Show that points P, Q and R are collinear. X

in terms of a and b.
a Express the vector XY
18.5 S
 olving geometric Give your answer in its simplest form.
 (2 marks)
problems Z

Y
1 R In parallelogram ABCD the point M is
the midpoint of AB and the point N is the 2a 1 3b
midpoint of BC. O

AB = a and BC
= b.
2a 1 2b
B b C X
a N XYZ is a straight line.
M XY:YZ = 3:1.

in terms of a and b.
b Express the vector OZ
A D Give your answer in its simplest form.
 (3 marks)
a Express in terms of a and/or b

ii BN iii MB iv NB
i AM
Q4b hint XY:YZ = 3:1 YZ = __
XZ
in terms of a and b.
b Express AC

R = Reasoning P = Problem-solving 18.5 Solving geometric problems 159



5 Exam-style question 7 P In triangle ABC, AB = b B

The diagram shows a regular and AC = c.
Example b
hexagon ABCDEF with The point M is the midpoint N M
Q
centre O. of BC and Q is the point
_2
DA= 8a
EB= 8b such that AQ = 3AM.
A c C
A B a Find in terms of b and c

i AM ii AQ iii BQ
8a N is the midpoint of AB.
F O C
8b b Find in terms of b and c the vector CN .
c Show that the points C, Q and N are collinear.
E X D
8 P In the diagram OR= 9a, OP
= 4b and
a Express in terms of a and/or b
PQ= 3a.
i OA 3a Q

ii OB P

iii AB (3 marks)
4b
X is the midpoint of ED. R
9a
b Express AX in terms of a and/or b.
O
 (2 marks)
The point M is on PQ such that PM = 2a.
Y is the point on AB extended, such that _5
The point N is on OR such that ON = 9OR.
AY = 2AB
The point T is on MN such that MT = _ 23MN.
c Show that D, C and Y lie on a straight
line.(3 marks) a Find in terms of a and/or b the vector NM .

b Find in terms of a and/or b the vector OT .

c V is the point such that QV= 9a.
6 P OACB is a parallelogram with OA = a
Find in terms of a and b the vector OV .
and OB = b.
d Show that T lies on the line OV.
E is the point on AC such that AE = _13AC.
e When a = ( ) and b = ( )find the length
1 2
F is the point on BC such that BF = _13BC. 5 7
of QR.
A E C Give your answer to 3 significant figures.

a
F
18 Problem-solving
O b B
Solve problems using these strategies where
a Find in terms of a and/or b
appropriate:
i AB
Use pictures or lists

ii AE Use smaller numbers


iii OE Use bar models

iv OF Use x for the unknown
b Show that EF is parallel to AB. Use a flow diagram
Use arrow diagrams
Use geometric sketches
Use graphs
Use logical reasoning
Use problem-solving strategies and then
explain.

160 R = Reasoning P = Problem-solving 18.5 Solving geometric problems


y+4 ______ 5y2 7 A curve has the equation
1 Write the expression _____
+ as a
3 8 y = ax3 + bx2 + cx + d
single fraction in its simplest form. It crosses the x-axis at x = 3, x = 1 and x = 1.
What are the values of a, b, c and d?
2 R The diagram shows a circle with centre O.
A, B, C, D and E are points on the 8 Exam-style question
circumference.
a Find the resultant of the vectors ( )
4
Angle DOC = 112 and OA bisects angle DAC. 1
and ( ) .
2
A (1 mark)
5
E B b OABC is a parallelogram.
M is the midpoint of AB.
O N is the midpoint of BC.
112
OA = a and OC
= c
D C A M B

a
Work out the sizes of the angles at the points N
of the star shape.
O c C
3 R WXYZ is a parallelogram.

Find MN in terms of a and c. (2 marks)
X b W November 2012, Q12, A502/02
a

Y Z 9 R In triangle DEF, DE = EF, X is the midpoint



a Explain why Y Z = b. of DE and Y is the midpoint of DF.

b Describe YWin terms of a and b. DE= 2a and DF
= 3b.
E
c C is the point on YZ such that Y C = _ 12YZ.
X
CDis parallel to YW
.Describe where point
D is on this parallelogram.
D Y F
8 12 5
( ) , CD=
( ) , NP=
( ) and XY=
( ).
4
4 AB= Express in terms of a and/or b
6 8 9 7
Put the vectors in descending order of
a EF b DX
length.


c DY d DE +EF
5 RVector c is parallel to a + b. 10 R In the diagram, A is the midpoint of OP,
Vector d is parallel to a b. C is the midpoint of PQ and B is the point on
Vector e is parallel to vector d. OQ such that OB:BQ = 1:2
Find vectors c, d and e when a = ( )
3
4 = a and OB
OA = b.
and b = ( ).
5
3 P
3 3
( ) and BC=
( ).
A C
6 In triangle ABC, AB= a
4 1
a Draw triangle ABC on suitable grid paper. O Q
b B

b Work out AC . Work out these vectors, in terms of

c Use vector addition to show that AC is a and/or b.

equivalent to AB + BC
. a OP
b
OQ
c
PQ


d AB e
AC f
BC
Q6 hint Think of the triangle law of vector addition.

R = Reasoning P = Problem-solving 18 Problem-solving 161


19 PROPORTION AND GRAPHS
5 Exam-style question
19.1 Direct proportion y is directly proportional to x.
When x = 500, y = 25
1 R The tables show the prices paid for
a Find a formula for y in terms of x.
different quantities of US dollars from two
 (3 marks)
currency exchange websites during July.
b Calculate the value of y when x = 360.
Website A  (1 mark)
Sterling () 40 180 500 450 160 80 140 320
US dollars ($) 70 300 750 700 255 120 210 510
Website B
Sterling () 300 200 280 410 100 200 370 50
19.2 M
 ore direct
US dollars ($) 540 340 480 700 170 350 610 80
proportion
a Draw a scatter graph for both sets of
1 R The extension of a spring, d (in metres),
information on the same axes.
is directly proportional to the force, F (in
b Draw a line of best fit for each set of data. newtons, N), used to extend the spring.
c Write a formula for dollars, D, in terms of When F = 20N, d = 5m.
sterling, S, for
a Express F in terms of d.
i Website A
ii Website B. b Find F when d = 12.5m.
d Which currency exchange website offers c Find d when F = 65N.
better value for money when buying dollars? 2 R The table gives information about the
Explain your answer. total surface area, A1, of different 3D objects
2 y is directly proportional to x. Example and the area of their smallest face, A2.
y = 60 when x = 12 A1 (cm2) 26 52 97.5 130
a Express y in terms of x. A2 (cm2) 4 8 15 20
b Find y when x = 20 a Show that A1 is directly proportional to A2.
c Find x when y = 2.5 b Given that A1 = kA2, work out the value
ofk.
Q2a hint Start with the statement y ~ x,
then write the equation y = kx. Use the c Write a formula for A2 in terms of A1.
values of x and y to find the value of k. d Work out
i the value of A1 when A2 = 5
3 y is directly proportional to x. ii the value of A2 when A1 = 162.5
y = 46 when x = 6 3 R The distance, d (in km), covered by a long
a Write a formula for y in terms of x. distance runner is directly proportional to the
b Find y when x = 24 time taken, t (in hours).
c Find x when y = 161 The runner covers a distance of 42km in
4hours.
4 P y is directly proportional to x. a Find a formula for d in terms of t.
a y = 12 when x = 5 b Find the value of d when t = 8
Find x when y = 7.2
c Find the value of t when d = 7.7
b y = 16.2 when x = 9
d What happens to the distance travelled, d,
Find x when y = 15.3
when the time, t, is
c y = 17.4 when x = 0.6 i trebled
Find x when y = 58 ii divided by 3?

162 R = Reasoning P = Problem-solving 19.1 Direct proportion


4 R The amount, C (in ), a plumber charges is 10 P The volume, V (in cm3), of a sphere is
directly proportional to the time, t (in hours), directly proportional to the cube of its radius,
that the plumber works. A plumber earns r (in cm). Asphere with a radius of 5cm has a
247.50 when she works 5.5hours. volume of 523.5cm3.
a Sketch a graph of C against t. a Write a formula for V in terms of r.
b Write a formula for C in terms of t. b Calculate V when the radius is 20cm.
c Use your formula to work out how many 11 RThe y-coordinate of a point on a parabola
hours the plumber has worked when she is directly proportional to the square root of
earns 1035. the x-coordinate of that point.
5 y is proportional to the square of x. When x = 36, y = 24
When x = 2, y = 16 a Write a formula for y in terms of x.
a Write the statement of proportionality. b Find the value of y when x = 81.
b Write an equation using k. 12 P In an experiment, a 45 125 320
c Work out the value of k. measurements were
b 3 5 8
d Find y when x = 10 taken of a and b.
e Find x when y = 100 Which of these laws fits the results? __
6 P y is proportional to the cube of x. a ~ ba ~ b2a ~ b3a ~ b
When x = 5, y = 25
a Write a formula for y in terms of x.
b Find y when x = 2 19.3 Inverse proportion
c Find x when y = 25000
1 y is inversely proportional to x. Example
Q6a hint Write y = k and find the value When y = 9, x = 4
of k. a Write a formula for y in
terms of x.
7 y is proportional to the square root of x. b Calculate the value of y
When x = 16, y = 20 when x = 12
a Find a formula for y in terms of x. c Calculate the value of x when y = 2
b Find y when x = 64
2 R The average speed of a car, s (in km/h),
c Find x when y = 55 is inversely proportional to the time, t (in
8 Exam-style question hours), it takes to travel a fixed distance.
s = 60km/h when t = 0.2 hours
y is directly proportional to the cube of x.
a Write a formula for s in terms of t.
When x = 2, y = 48
b Work out the time of a journey when the
Find the value of y when x = 5. (4 marks)
average speed is 50km/h.
c Work out the average speed when the
9 R When an object accelerates steadily from
time of the journey is 1.5 hours.
rest, the distance, d (in metres), it moves
varies in direct proportion to the square of d What happens to the average speed when
the time, t(in seconds), it has been moving. the time of the journey doubles?
An object moves 176.4m in 6 seconds. 3 P As a balloon is blown up, the thickness
a Write a formula for d in terms of t. of its walls, t (in mm), decreases and its
b How far does an object move if it volume, V (in cm3), increases. V is inversely
accelerates like this for 10 seconds from proportional to t. When V is 15000cm3,
rest? t is 0.05mm.
c How many seconds has an object been a Write a formula for V in terms of t.
accelerating for if it has moved 1102.5m? b When the thickness of the wall of the
d What happens to the distance moved, d, if balloon is 0.03mm, the balloon will pop.
the time the object has been accelerating Is it possible to blow up this balloon to a
for is doubled? volume of 30000cm3?

R = Reasoning P = Problem-solving 19.3 Inverse proportion 163


4 R y is inversely proportional to x. 8 R A vineyard owner employs teams of
x 0.25 0.5 1 2 4 5 10 20 people to pick grapes.
y 48 24 12 6 3 2.4 1.2 0.6 Teams have different numbers of people.
The owner records the time it takes different
a Draw a graph of y against x. teams to pick the grapes from 12 rows of
What type of graph is this? vines.
k
b y=__
xwhere k is the constant of Number of people Time taken,
proportionality. Find k. in team, n t(hours)
c Work out x y for each pair of values in 2 3.5
the table. What do you notice?
3 1.8
5 R Which of these graphs shows variables in 12 0.5
inverse proportion?
6 1
A y B y
4 1.4
5 1.3
O x O x 8 0.8
9 0.8
C y 10 0.6
a Plot t against n.
O x b Draw a curve of best fit.
c Write a formula for estimating t in terms
of n.
6 R The length, l (in cm), and width, w (in cm), d Use your formula to estimate how long it
of a rectangle with a fixed area are inversely would take a team of 15 people to pick the
proportional. When the length of the grapes from 12 rows of vines.
rectangle is 4.5cm, the width is 2.2cm.
a Write a formula for l in terms of w. k
Q8c hint t=__Use a point on the curve of
b Copy and complete the table of values for
l and w. best fit to find the value of k.
Length, l (cm) Width, w (cm)
0.3 9 Exam-style question
19.8
p is inversely proportional to the square of q.
0.9 When q = 10, p = 7.5
2 Find the value of p when q = 5 (3 marks)
3 3.3
1.65
15 10 y is inversely proportional to the cube of x.
c Sketch a graph to show how l varies with w. When x = 3, y = 6
a Write a formula for y in terms of x.
7 P The graph shows two variables that are
b Calculate y when x = 2
inversely proportional to each other.
Find the values of p and q. c Calculate x when y = 0.75
y
11 y is inversely proportional to the square root
of x.
(3, 8)
When x = 4, y = 5
(4, p) a Write a formula for y in terms of x.
(q, 3) b Calculate y when x = 16
c Calculate x when y = 2
O x

164 R = Reasoning P = Problem-solving 19.3 Inverse proportion


12 R When 30 litres of water are poured into 3 R a Draw the graphs of
any cylinder, the depth, D (in cm), of the i y = 3.5x ii y = 4.5x
water is inversely proportional to the square b Predict where the graph of y = 4x would be.
of the radius, r (in cm), of the cylinder. Sketch it on the same axes.
When r = 30cm, D = 10.6cm c At which point do all the graphs intersect
a Write a formula for D in terms of r. the y-axis?
b Find the depth of the water when the 4 a Copy and complete the table of values for
radius of the cylinder is 15cm. y = 4x. Give the values correct to 2 d.p.
c Find the radius of the cylinder (to 1 decimal
place) when the depth is 60cm. x 3 2 1 0 1 2 3
d Cylinder P has radius x cm and is filled y
with water to a depth of dcm. This water b Draw the graph of y = 4x for 3 < x < 3
is poured into cylinder Q and fills it to c Use the graph to find an estimate for
a depth of 3dcm. What is the radius of i the value of y when x = 1.5
cylinder Q? Give your answer in terms of x. ii the value of x when y = 5
5 R The table gives information about the
population of the UK.
3d cm
d cm x Year 1751 1801 1851 1901 1951 2001 2014
UK
P Q population 5.8 7.8 15.3 30.1 38.7 49.1 64.1
(millions)
13 R The gravitational force between two
a Draw a graph of the data.
objects, F (in newtons, N), is inversely
Plot the year on the horizontal axis and
proportional to the square of the distance, d
population on the vertical axis.
(in metres), between them.
Draw in a trend curve of best fit.
A satellite orbiting the Earth is 4.2 107m
from the centre of the Earth. b Is this an example of exponential growth
The force between the satellite and the Earth or exponential decay?
is 60N. c Use your graph to estimate the number of
a Write a formula for F in terms of d. years it took for the population to double
from its value in 1901.
b The force between two objects is 16N.
What is the value of the force when the 6 Exam-style question
distance between the objects doubles?
The sketch shows a curve with Example
equation y = kax, where k
and a are constants and a . 0.
19.4 Exponential functions y
(4, 1500)
1 Find the value of x for each of these
equations.
a 3x = 27 b 4x = 256 c 10x = 1000000 (1, 12)

2 a Copy and complete the table of values O x


fory = 3x.
Give the values correct to 2 decimal places. The curve passes through the points (1, 12)
and (4, 1500).
x 4 3 2 1 0 1 2 3 4 Calculate the value of k and the
y value of a.(3 marks)
b Draw the graph of y = 3x for 4 < x < 4
c Use the graph to find an estimate for
Q6 hint Start by using the point (1, 12) to
i the value of y when x = 2.5 express k in terms of a.
ii the value of x when y = 30

R = Reasoning P = Problem-solving 19.4 Exponential functions 165


7 R The value, V (in ), of a motorbike
depreciates exponentially over time. 19.5 Non-linear graphs
The value of the motorbike on 1 January 2015
is 15000. 1 Water is poured into a curved Example
The value of the motorbike on 1 January 2017 vase at a constant rate. h is the
is 8400. height of water after time t.
The sketch graph shows how the value of the a Describe how the rate at
motorbike changes over time. which the height increases
t
changes over time.
b Which graph best describes
the relationship between
h and t? h

A B C D
O V h h h h

The equation of the graph is V = pqt where


t is the number of years after 1 January 2015,
and p and q are positive constants. t t t t
a Use the information to find the values of 2 R The graph shows the relationship between
pand q. the temperature of cooling water in a kettle,
b Use your values of q and p in the formula in C, and the time, in minutes.
V = pqt to estimate the value of the Temperature of water
motorbike on 1 January 2018.
c By what percentage (to 1 decimal place) 100
does the motorbike depreciate each year 90
from 2015 to 2018? 80
Temperature (C)

70
8 R A business is currently valued at
60
2.4million, and is growing at a rate of
50
15% a year.
40
The expected value, v (in millions), in
tyears time, is given by the formula 30
v = 2.4 1.15t. 20
a Use a table of values to draw the graph 10
ofv against t for the next 5 years. 0
0 5 10 15 20 25 30 35
b Use your graph to estimate
Time (minutes)
i the value of the business after
2.5years a What is the temperature of the water after
ii the time taken for the value to reach 20 minutes?
3million. b Describe the rate at which the water cools
down.
9 R 8000 is invested in a savings account c Calculate the drop in temperature
paying 5% compound interest a year. between 0 and 5 minutes.
a Write a formula for the value of the d Calculate the average rate of cooling
savings account (V) and the number of between 5 and 10 minutes.
years (t). e Zain says, The water cools at least three
b Draw a graph of V against t for the first times as quickly between 0 and 5 minutes
8years. as it does between 20 and 25 minutes.
c Use the graph to estimate when the Is Zain correct? Explain your answer.
investment will reach a value of f Compare the average rate of cooling
10000. over the first 15 minutes with the rate of
cooling at exactly 15 minutes.

166 R = Reasoning P = Problem-solving 19.4 Exponential functions


3 R The distancetime graph shows d Calculate the areas of your trapezia to
information about a plane accelerating. estimate the area under the graph of
Motion of plane y=_12x2+1 from x = 0 to x = 4.
6 R The distancetime graph shows
8000
Distance, d (metres) information about a 20km bike race between
7000
Dawn and Aarti.
6000
Dawn and Aartis bike race
5000
4000 20
3000 Dawn
18
2000 16

Distance (km)
1000 14
0 Aarti
12
0 10 20 30 40 50 60
Time, t (seconds) 10
8
a Estimate the speed of the plane 6
30seconds after it starts moving. 4
b What speed does the plane reach after 2
60seconds? 0
c Will the speed of the plane continue 0 5 10 15 20 25 30 35 40
to rise after 60 seconds following the Time (minutes)
relationship shown by the graph? a Describe the race between Dawn and Aarti.
Explain your answer.
b Compare Dawns speed for the first and
second half of the race.
4 R A man is cycling Motion of cyclist
at a speed of 12m/s. c Estimate the difference in their speeds
14 20minutes into the race.
Velocity, V (m/s)

Thevelocity 12
time graph gives 10 7 Exam-style question
information about 8
the motion of 6 The velocitytime graph describes the
the bicycle as he 3 acceleration of a truck.
2 Velocity, v, is measured in metres per second
decelerates to rest
0 (m/s) and time, t, is measured in seconds.
in 5 seconds. 0 1 2 3 4 5 6
a Copy the graph. Time, t (seconds) Acceleration of a truck
Draw a chord from t = 0 to t = 5.
30
b Calculate the average deceleration of the
25
Velocity (m/s)

bicycle over the 5 seconds.


c Estimate the deceleration at time 20
t = 4 seconds. 15
d Describe how the deceleration changes 10
over the 5 seconds. 5
e Draw a chord from t = 0 to t = 3. 0
0 2 4 6 8 10 12
Use the chord to form a trapezium.
Time (s)
Estimate the distance travelled between
t =0 and t = 3 by finding the area of the a Estimate the acceleration at t = 5.
trapezium.  (2 marks)
b Estimate the distance travelled between
5 a Draw the graph of y=_21x2+1 for 0 , x , 4 t = 2 and t = 6. (3 marks)
b Draw in a chord from x = 0 to x = 1 and use c The instantaneous acceleration at time T
it to make a trapezium under the graph. is equal to the average acceleration over
c Repeat with chords from x = 1 to x = 2, the first 10 seconds. Find an estimate for
x = 2 to x = 3 and x = 3 to x = 4. the value of T. (3 marks)

R = Reasoning P = Problem-solving 19.5 Non-linear graphs 167


Exam-style question
19.6 Translating graphs of 4
functions The graph of y = f(x) is shown Example
on the grid.
y
1 Draw a coordinate grid with 5 to 5 on both 10
axes. 8
graph G
a On the same set of axes draw the 6
y  f(x)
graphsof 4
i y = f(x) = x2 2
ii y = f(x) + 3 = x2 + 3
iii y = f(x 1) = (x 1)2 26 24 22 O 2 4 6 8 x
22
b The maximum point of y = f(x) is (0, 0).
24
Write the coordinates of the maximum
point of
i y = f(x) + 3 The graph G is a translation of the graph
ii y = f(x 1) of y = f(x).
c Describe the transformation that maps the Write down the equation of graph G.
 (1 mark)
graph of y = f(x) onto the graph of
i y = f(x) + 3
ii y = f(x 1) Q4 hint First see if the translation is to the
right or to the left or up or down and by how
Q1a hint Create a table of values for each many squares.
graph.

5 Exam-style question
2 Here is the graph of y = f(x) = x2.
y The graph of y = f(x) is shown on the grid.
1 y
4
24 23 22 21O 1 2 3 4 x 3
21
22 2
23 1
24
26 25 24 23 22 21 O 1 2 3 4 x
25 21
22
Copy the axes and sketch the graphs of 23
a y = f(x) + 2 24
b y = f(x) 4 25
c y = f(x 2) 26
d y = f(x + 3)
Copy the diagram and sketch the graph
3 Write the vector that translates y = f(x) of y = f(x 4) 1. (2 marks)
onto
a y = f(x) + 3
Q5 hint Make sure you translate the points
b y = f(x) 5 that have integer coordinates such as (2,1)
c y = f(x 5) and (0, 3) exactly the right number of
d y = f(x + 4) squares in the correct direction.
e y = f(x 5) + 3

168 R = Reasoning P = Problem-solving 19.6 Translating graphs of functions


6 Here is a sketch of y b Caz says, The graph of y = f(x) is a
y = f(x) = x 3 4 reflection of the graph of y = f(x) in the
a Draw sketches of the 2 line x = 0.
graphs The graph of y = f(x) is a reflection of the
24 22O 2 4 x graph of y = f(x) in the line y = 0.
i y = f(x) 1 22
ii y = f(x + 2) Is Caz correct?
24
b Write the coordinates Explain your answer.
of the point which
3 R The diagram shows the graph of y = f(x).
(0, 0) is mapped to for both graphs.
The turning point of the curve is A(3, 4).
7 Rf(x) = 3 4x y
a Draw the graph of y = f(x) 8
b Draw the graph of y = f(x 1) 6
c Write the algebraic equation of y = f(x 1) 4
1 2
8 Pf(x) = __
x
a Sketch the graph of y = f(x + 1) 2 24 22O 2 4 6 8 x
22
b Write the equation of each asymptote.
24

19.7 Reflecting and Write the coordinates of the turning point of


stretching graphs of the curve with equation
a y = f(x)
functions b y = f(x)
c y = f(x)
1 f(x) = 5 2x
a Copy and complete the table. 4 R Here is the graph y = f(x) = x3 1
x 2 1 0 1 2 y
3
f(x)
2
f(x)
1
f(x)
b On the same set of axes, draw the graphs of 23 22 21O 1 2 3 4 x
21
i y = f(x) ii y = f(x) iii y = f(x) 22
c Describe the transformation that maps 23
f(x) onto f(x). 24
d Describe the transformation that maps
f(x) onto f(x).
a Sketch a copy of the graph and on the
2 R The diagram shows the graph of y = f(x). same axes sketch the graphs of
y i f(x) ii f(x) iii f(x)
4 b Describe the transformation that maps
2 f(x) onto f(x).

24 22O 2 4 6 x 5 f(x) = 2 x2
22 a Copy and complete the table.
24
x 4 3 2 1 0 1 2 3 4
26
f(x)
28
3f(x)
a Sketch a copy of the graph. f(3x)
On the same axes sketch the graphs of b On the same set of axes draw the graphs of
i y = f(x) ii y = f(x) i y = f(x) ii y = 3f(x) iii y = f(3x)

R = Reasoning P = Problem-solving 19.7 Reflecting and stretching graphs of functions 169


6 Here is the graph of y = f(x) Example 9 R The diagram shows the graph of
y y = f(x) = 5_12x and the graphs of some
12 transformations of f(x)
10 y B
8 10
6 y  f(x)
4 5 A
2
O 15 x
26 24 22O 2 4 6 8 x 210 25 5 10
22
25 C
Draw the graphs of
a y = 2f(x) 210 D
b y = 3f(x)
c y = _12f(x) Match the function notation to the graphs.
7 Here is the graph of y = f(x) a f(2x)
b 2f(x)
y
5 c f(x)
4 d f(x)
3
10 R Here is a sketch of y = f(x) = (x 3)2 + 2
2
y
1
(0, 11)
23 22 21 O 1 2 3 x
21
22

Draw the graphs of


a y = f(3x) b y = f(_12x) (3, 2)
O x
8 Exam-style question
The graph of y = f(x) is shown on the grid.
The graph has a minimum point at (3, 2).
y
Itintersects the y-axis at (0, 11).
4
a Sketch the graph of y=f(_12x)
3
b Write down the minimum value of f(_12x)
2
c Write down the coordinates of the
1
minimum point of y = f(2x)
2 1 O 1 2 3 4 x d R Explain why the graphs of y = f(x),
1 y = f(_12x) and y = f(2x) all intersect the
2 yaxis at the same point.
3
4

Copy the diagram and sketch the graph


of y = 2f(x). (2 marks)

Q8 hint Calculate some y-values for y = 2f(x)


to make sure your graph passes through the
correct points.

170 R = Reasoning P = Problem-solving 19.7 Reflecting and stretching graphs of functions


5 Jennifer is building a ramp from her gate to
19 Problem-solving her doorstep.
The height of her step is 28.4cm and
Solve problems using these strategies where the distance from the gate to the step is
appropriate: 240.6cm.
Use pictures Both measurements are to the nearest mm.
Use smaller numbers What are the upper and lower bounds for
Use bar models angle x, to 3 d.p?
Use x for the unknown x
Use a flow diagram 28.4 cm
Use arrow diagrams 240.6 cm
Use geometric sketches
Use graphs 6 R Marcia and Judy are both asked to sketch
Use logical reasoning the graph of the exponential function y = 6x
Use problem-solving strategies and then
Marcia sketches this graph and explains that
explain.
the function shows exponential growth.
1 A 3 inch chord PQ is cut y
across a circle with centre O. O
1 in 8
The minimum distance P Q 7
3 in
from the centre of the circle
6
to the chord is 1 inch. What is the radius, PO,
of the circle? Give your answer to 1 d.p. 5
4
2 R A tap is dripping at a steady rate. 3
Acontainer is put underneath to catch the
2
drips. Use the graph of the water depth in
the container to sketch the shape of the 1
container.
24 23 22 21O 1 2 x
21
Depth (mm)

30
20 Judy sketches this graph and explains that
10 the function shows exponential decay.
0 y
5 10 15 20
Time (minutes)
8
7
3 R a Find a counter example to prove that 6
the statement n2 + 3 can never be divided 5
by 2, where both n and the answer are
4
integers is not true.
3
b What could you say about n to make the
statement true? 2
1
4 Lukes wages are calculated for every 45
minutes worked and are shown in the table. 22 21O 1 2 3 4 x
21
Minutes 45 90 135 180
Wages () 6.30 12.60 18.90 25.20 a Who is correct?
The wages earned (w) are directly proportional Explain why.
to the number of minutes worked (m). b If x = 5 what would the value of y be?
a Find a formula for w in terms of m.
b How many 45-minute blocks must Luke 7 p is inversely proportional to the cube of q.
work to earn 200? When q = 3, p = 5
c How much does Luke earn per hour? Find the value of p when q = 5

R = Reasoning P = Problem-solving 19 Problem-solving 171


8 Exam-style question 10 R If A shows y = f(x), what does B show?
a y is inversely proportional to x2. B A y
y = 8 when x = 3. 8
Work out an equation connecting x and 7
y. (3 marks) 6
b Work out the value of y when x = 12. 5
Give your answer as a fraction in its 4
simplest form. (2 marks)
3
2
9 R The points M, N and P have the 1
coordinates (1, 9), (7, 3) and (5, 5)
respectively. 24 23 22 21O 1 2 3 4 x
21
Prove that the three points are collinear.
22
Q9 hint Collinear means lie on the same
straight line.

172 R = Reasoning P = Problem-solving 19 Problem-solving

You might also like